Sie sind auf Seite 1von 68

1.MAGALLONA v. ERMITA G.R.

187167, August 16, 2011


Facts:
In 1961, Congress passed R.A. 3046 demarcating the maritime baselines of the Philippines
as an Archepelagic State pursuant to UNCLOS I of 9158, codifying the sovereignty of State
parties over their territorial sea. Then in 1968, it was amended by R.A. 5446, correcting
some errors in R.A. 3046 reserving the drawing of baselines around Sabah. In 2009, it was
again amended by R.A. 9522, to be compliant with the UNCLOS III of 1984. The
requirements complied with are: to shorten one baseline, to optimize the location of some
basepoints and classify KIG and Scarborough Shoal as ‘regime of islands’. Petitioner now
assails the constitutionality of the law for three main reasons: 1. it reduces the Philippine
maritime territory under Article 1; 2. it opens the country’s waters to innocent and sea lanes
passages hence undermining our sovereignty and security; and 3. treating KIG and
Scarborough as ‘regime of islands’ would weaken our claim over those territories. Issue:
Whether R.A. 9522 is constitutional? Ruling: 1. UNCLOS III has nothing to do with acquisition
or loss of territory. it is just a codified norm that regulates conduct of States. On the other
hand, RA 9522 is a baseline law to mark out basepoints along coasts, serving as geographic
starting points to measure. it merely notices the international community of the scope of our
maritime space. 2. If passages is the issue, domestically, the legislature can enact legislation
designating routes within the archipelagic waters to regulate innocent and sea lanes
passages. but in the absence of such, international law norms operate. the fact that for
archipelagic states, their waters are subject to both passages does not place them in lesser
footing vis a vis continental coastal states. Moreover, RIOP is a customary international law,
no modern state can invoke its sovereignty to forbid such passage. 3. On the KIG issue, RA
9522 merely followed the basepoints mapped by RA 3046 and in fact, it increased the Phils.’
total maritime space. Moreover, the itself commits the Phils.’ continues claim of sovereignty
and jurisdiction over KIG. If not, it would be a breach to 2 provisions of the UNCLOS III: Art.
47 (3): ‘drawing of basepoints shall not depart to any appreciable extent from the general
configuration of the archipelago’. Art 47 (2): the length of baselines shall not exceed 100
mm. KIG and SS are far from our baselines, if we draw to include them, we’ll breach the
rules: that it should follow the natural configuration of the archipelago.
PROF. MERLIN M. MAGALLONA, et.al v HON. EDUARDO ERMITA, IN HISCAPACITY AS
EXECUTIVE SECRETARY, et.al G.R. No. 187167, 16 July 2011, EN BANC (Carpio, J.) The
conversion of internal waters into archipelagic waters will not risk the Philippines because an
archipelagic State has sovereign power that extends to the waters enclosed by the
archipelagic baselines, regardless of their depth or distance from the coast.
R.A. 9522 was enacted by the Congress in March 2009 to comply with the terms of the
United Nations Convention on the Law of the Sea (UNCLOS III), which the Philippines ratified
on February 27, 1984. Such compliance shortened one baseline, optimized the location of
some base points around the Philippine archipelago and classified adjacent terr itories such
as the Kalayaan Island Ground (KIG) and the Scarborough Shoal as “regimes of islands”
whose islands generate their own applicable maritime zones .Petitioners, in their capacities
as “citizens, taxpayers or legislators” assail the constitutionality of R.A. 9522 with one of
their arguments contending that the law unconstitutionally “converts” internal waters into
archipelagic waters, thus subjecting these waters to the right of innocent and sea lanes
passage under UNCLOS III, including overflight. Petitioners have contended that these
passage rights will violate the Constitution as it shall expose Philippine internal waters to
nuclear and maritime pollution hazard.

ISSUE:
Whether or not R.A. 9522 is unconstitutional for converting internal waters into archipelagic
waters

HELD:
Petition DISMISSED. The Court finds R.A. 9522 constitutional and is consistent with the
Philippine’s national interest. Aside from being a vital step in safeguarding the country’s
maritime zones, the law also allows an internationally-recognized delimitation of the breadth
of the Philippine’s maritime zones and continental shelf. The Court also finds that the
N a t r e _ M i d t e r m S Y 2 0 1 9 - 2 0 2 0 _ 1 s t S e m e s t e r _ P a g e 1 | 68
conversion of internal waters into archipelagic waters will not risk the Philippines as affirmed
in the Article 49 of the UNCLOS III, an archipelagic State has sovereign power that extends
to the waters enclosed by the archipelagic baselines, regardless of their depth or distance
from the coast. It is further stated that the regime of archipelagic sea lanes passage will not
affect the status of its archipelagic waters or the exercise of sovereignty over waters and air
space, bed and subsoil and the resources therein.

2.Oposa vs Factoran
Natural and Environmental Laws; Constitutional Law: Intergenerational Responsibility GR
No. 101083; July 30 1993

FACTS:
A taxpayer’s class suit was filed by minors Juan Antonio Oposa, et al., representing their
generation and generations yet unborn, and represented by their parents against Fulgencio
Factoran Jr., Secretary of DENR. They prayed that judgment be rendered ordering the
defendant, his agents, representatives and other persons acting in his behalf to:

1. Cancel all existing Timber Licensing Agreements (TLA) in the country;


2. Cease and desist from receiving, accepting, processing, renewing, or appraising
new TLAs; and granting the plaintiffs “such other reliefs just and equitable under
the premises.” They alleged that they have a clear and constitutional right to a
balanced and healthful ecology and are entitled to protection by the State in its
capacity as parens patriae. Furthermore, they claim that the act of the
defendant in allowing TLA holders to cut and deforest the remaining forests
constitutes a misappropriation and/or impairment of the natural resources
property he holds in trust for the benefit of the plaintiff minors and succeeding
generations.

The defendant filed a motion to dismiss the complaint on the following grounds:

1. Plaintiffs have no cause of action against him;


2. The issues raised by the plaintiffs is a political question which properly
pertains to the legislative or executive branches of the government.

ISSUE:
Do the petitioner-minors have a cause of action in filing a class suit to “prevent the
misappropriation or impairment of Philippine rainforests?”

HELD:
Yes. Petitioner-minors assert that they represent their generation as well as generations to
come. The Supreme Court ruled that they can, for themselves, for others of their generation,
and for the succeeding generation, file a class suit. Their personality to sue in behalf of
succeeding generations is based on the concept of intergenerational responsibility insofar as
the right to a balanced and healthful ecology is concerned. Such a right considers the “rhythm
and harmony of nature” which indispensably include, inter alia, the judicious disposition,
utilization, management, renewal and conservation of the country’s forest, mineral, land,
waters, fisheries, wildlife, offshore areas and other natural resources to the end that their
exploration, development, and utilization be equitably accessible to the present as well as
the future generations. Needless to say, every generation has a responsibility to the next to
preserve that rhythm and harmony for the full enjoyment of a balanced and healthful
ecology. Put a little differently, the minor’s assertion of their right to a sound environment
constitutes at the same time, the performance of their obligation to ensure the protection of
that right for the generations to come.

Natre_Midterm SY2019-2020_1stSemester_ P a g e 2 | 68
3.CARINO V INSULAR GOVT

Mateo Cariño (plaintiff in error) vs. Insular Government of the Philippines (defendant in error)
212 US 449, 41 Phil Justice Holmes

How it reached the court: - Plaintiff applied for registration of a certain land. Initially it was
granted by the court, but the Government of the Philippines and the government of the
United states appealed to the Court of first instance of Benguet (they were taking the
property for public and military purposes. The CFI dismissed the application (for registration)
and this was affirmed by the Philippine Supreme Court. This was brought to the US Supreme
court by writ of error.

Facts:
Plaintiff, an Igorot, possessed the land for more than 30 years before the treaty of Paris. He
and his ancestors had held the land for years. The local community recognizes them as the
owners of the said land. His grandfather lived upon it and maintained fences around the
property. His father raised cattle on the property and he had inherited the land according to
Igorot custom. Although no title was issued to them from the Spanish Crown. He tried twice
to have it registered during the Spanish occupation but to no avail. In 1901 he filed a petition
alleging ownership of the land but he was only granted a possessory title.

Preliminary issues.
o Whether the mode of reaching the US supreme court was right (this was a writ of error;
some were saying that it should have been an appeal) – Holmes said that the mode was
correct. Writ of error was the general rule, appeal is the exception. He saw no reason not to
apply the general rule to this case.

o Another issue was that even if Cariño was able to have a title over the land, he could not
have it registered because Benguet was one of the excluded provinces in the Philippine
Commission’s act no. 926 (AN ACT PRESCRIBING RULES AND REGULATIONS GOVERNING
THE HOMESTEADING, SELLING, AND LEASING OF PORTIONS OF THE PUBLIC DOMAIN OF
THE PHILIPPINE ISLANDS…). But that law dealt with acquisition of new titles and perfecting
of titles begun under the Spanish law. Cariño argued that he could register the land under
Philippine Commissions Act no. 496 which covered the entire Philippine archipelago. Holmes
held that he could register the land if ownership can be maintained

Main issue: whether Cariño owns the land.

o Government’s argument: Spain had title to all the land in the Philippines except those
it saw fit to permit private titles to be acquired. That there was a decree issued by Spain that
required registration within a limited time. Cariño’s land wasn’t registered and so in effect it
became public land. USSC: Whatever the position of Spain was on the issue, it does not
follow that the US would view plaintiff to have lost all his rights to the land – this would
amount to a denial of native titles throughout Benguet just because Spain would not have
granted to anyone in the province the registration of their lands. Organic act of July 1, 1902
provides that all the property and rights acquired there by the US would be for the benefit
of the inhabitants thereof. This same statute made a bill of rights embodying the safeguards
of the constitution, it provides that “'no law shall be enacted in said islands which shall
deprive any person of life, liberty, or property without due process of law, or deny to any
person therein the equal protection of the laws”. It would be hard to believe that that “any
person” didn’t include the inhabitants of Benguet. Nor it meant “property” to refer only to
those lands which had become such under a ceremony (of registration) many of the people
of the land may have not even heard of. Although in sec. 14 of the organic act, it is said that
the Philippine commission may prescribe rules and regulations for perfecting titles to public
lands, it should be noted that this section refers to those cases where the land was admitted
to be public land. The US SC hesitates to suppose that it was intended to declare every native
who had not a paper title, a trespasser.
N a t r e _ M i d t e r m S Y 2 0 1 9 - 2 0 2 0 _ 1 s t S e m e s t e r _ P a g e 3 | 68
The question still remains: what property and rights did the US acquire? in cases like this
one, the presumption would and should be against the government. As far back as memory
goes, the land has been held by individuals under a claim of private ownership, it was never
public land. It would not be proper to just let the conqueror to dictate how to deal with the
Philippine tribes if it really meant to use the rights acquired by them “for the benefit of the
inhabitants thereof”. The natives were recognized by the Spanish laws to own some lands,
irrespective of any royal grant. They didn’t intend to turn all the inhabitants into trespassers.
Principle of prescription was admitted: that if they weren’t able to produce title deeds, it is
sufficient if they show ancient possession, as a valid title by prescription. Although there was
a decree in June 25, 1880 that required everyone to get a document of title or else lose his
land, it does not appear that it meant to apply to all but only those who wrongfully occupied
royal lands. IT doesn’t appear that the land of Cariño was considered as Royal land nor was
it considered to have been wrongfully occupied. Two articles of the same decree provided
that titles would be attributed to those who may prove possession for the necessary time.
There were indications that registration was expected but it didn’t mean that ownership
actually gained would be lost. The effect of the proof was not to confer title to them but to
establish it. o “Law and justice require that the applicant should be granted what he seeks
and should not be deprived of what, by the practice and belief of those among whom he
lived, was his property, through a refined interpretation of an almost forgotten law of Spain.
“Judgment reversed

4.YSMAEL V EXEC SECRETARY DIGEST


FACTS:
Soon after the change in government in 1986, Petitioner a letter to the Office of the
President, and to Minister Ernesto Maceda of the Ministry of Natural Resources [MNR],
seeking: (1) the reinstatement of its timber license agreement which was cancelled during
the Marcos administration; (2) the revocation of TLA No. 356 which was issued to Twin Peaks
Development and Realty Corporation without public bidding and in violation of forestry laws,
rules and regulations; and, (3) the issuance of an order allowing petitioner to take possession
of all logs found in the concession area. P alleged: it entered into a timber license agreement
with the DENR, wherein it was issued an exclusive license to cut, collect and remove timber
except prohibited species within a specified portion of public forest land located in the
municipality of Maddela, province of Nueva Vizcaya from October 12, 1965 until June 30,
1990; That on August 18, 1983, the Director of the Bureau of Forest Development
[hereinafter referred to as "Bureau"], issued a memorandum order stopping all logging
operations in Nueva Vizcaya and Quirino provinces, and cancelling the logging concession of
petitioner and nine other forest concessionaires, pursuant to presidential instructions and a
memorandum order of the Minister of Natural Resources That on August 25, 1983, petitioner
received a telegram from the Bureau, requesting him to stop all logging operations and to
pull out logging machineries and equipment in order to conserve remaining forests. That
after the cancellation of its timber license agreement, it immediately sent a letter addressed
to then President Ferdinand Marcos which sought reconsideration of the Bureau' s directive,
citing in support thereof its contributions to forest conservation and alleging that it was not
given the opportunity to be heard prior to the cancellation of its logging operation, but no
favorable action was taken on this letter; That barely one year thereafter, approximately
one-half or 26, 000 hectares of the area formerly covered by TLA No. 87 was re -awarded
to Twin Peaks Development and Realty Corporation under TLA No. 356 which was set to
expire on July 31, 2009, while the other half was allowed to be logged by Filipinas Loggers,
Inc. without the benefit of a formal award or license; and, That the latter entities were
controlled or owned by relatives or cronies of deposed President Ferdinand Marcos. The MNR
issued an order denying petitioner's request. It ruled that a timber license was not a contract
within the due process clause of the Constitution, but only a privilege which could be
withdrawn whenever public interest or welfare so demands, and that
petitioner was not discriminated against in view of the fact that it was among ten
concessionaires whose licenses were revoked in 1983. Moreover, emphasis was made of the
N a t r e _ M i d t e r m S Y 2 0 1 9 - 2 0 2 0 _ 1 s t S e m e s t e r _ P a g e 4 | 68
total ban of logging operations in the provinces of Nueva Ecija, Nueva Vizcaya, Quirino and
Ifugao reasoning that the Ministry imposed the ban because it realizes the great
responsibility it bears [sic] in respect to forests. It considers itself the trustee thereof. This
being the case, it has to ensure the availability of forest resources not only for the present,
but also for the future generations of Filipinos. P’s MR denied. Meanwhile, MNR issued an
administrative order lifting the logging ban in the province of Quirino. P then appealed to the
Office of the President which however denied it on the ground of lack of merit. It ruled that
the appeal of petitioner was prematurely filed, the matter not having been terminated in the
MNR. Petitioner's motion for reconsideration was denied. Hence, this petition for certiorari.

Issue: WON public respondent committed grave abuse of discretion amounting to lack or
excess of jurisdiction and WON TLA are contracts.

Held: NO. It is an established doctrine in this jurisdiction that the decisions and orders of
administrative agencies have upon their finality, the force and binding effect of a final
judgment within the purview of the doctrine of res judicata. These decisions and orders are
as conclusive upon the rights of the affected parties as though the same had been rendered
by a court of general jurisdiction. The rule of res judicata thus forbids the reopening of a
matter once determined by competent authority acting within their exclusive jurisdiction. As
gleaned from the record, petitioner did not avail of its remedies under the law, i.e. Section
8 of Pres. Dec. No. 705 as amended, for attacking the validity of these administrative actions
until after 1986. By the time petitioner sent its letter dated April 2, 1986 to the newly
appointed Minister of the MNR, requesting reconsideration of the above Bureau actions, these
were already settled matters as far as petitioner was concerned. Public respondents herein,
upon whose shoulders rests the task of implementing the policy to develop and conserve the
country's natural resources, have indicated an ongoing department evaluation of all timber
license agreements entered into, and permits or licenses issued, under the previous
dispensation. The ongoing administrative reassessment is apparently in response to the
renewed and growing global concern over the despoliation of forest lands and the utter
disregard of their crucial role in sustaining a balanced ecological system. The legitimacy of
such concern can hardly be disputed, most especially in this country. The Court takes judicial
notice of the profligate waste of the country's forest resources which has not only resulted
in the irreversible loss of flora and fauna peculiar to the region, but has produced even more
disastrous and lasting economic and social effects. The delicate balance of nature having
been upset, a vicious cycle of floods and droughts has been triggered and the supply of food
and energy resources required by the people seriously depleted. While there is a desire to
harness natural resources to amass profit and to meet the country's immediate financial
requirements, the more essential need to ensure future generations of Filipinos of their
survival in a viable environment demands effective and circumspect action from the
government to check further denudation of whatever remains of the forest lands. Nothing
less is expected of the government, in view of the clear constitutional command to maintain
a balanced and healthful ecology. Section 16 of Article II of the 1987 Constitution. A long
line of cases establish the basic rule that the courts will not interfere in matters which are
addressed to the sound discretion of government agencies entrusted with the regulation of
activities coming under the special technical knowledge and training of such agencies
Petitioner is precluded from availing of the benefits of a writ of certiorari in the present case
because he failed to file his petition within a reasonable period. In the case at bar, petitioner
waited for at least three years before it finally filed a petition for certiorari with the
Court attacking the validity of the assailed Bureau actions in 1983 and 1984. Considering
that petitioner, throughout the period of its inaction, was not deprived of the opportunity to
seek relief from the courts which were normally operating at the time, its delay constitutes
unreasonable and inexcusable neglect, tantamount to laches. Accordingly, the writ of
certiorari requiring the reversal of these orders will not lie. A cursory reading of the assailed
orders issued by public respondent Minister Maceda of the MNR, which were affirmed by the
Office of the President, will disclose public policy considerations which effectively forestall
judicial interference in the case at bar. Timber licenses, permits and license agreements are
the principal instruments by which the State regulates the utilization and disposition of forest
N a t r e _ M i d t e r m S Y 2 0 1 9 - 2 0 2 0 _ 1 s t S e m e s t e r _ P a g e 5 | 68
resources to the end that public welfare is promoted. And it can hardly be gainsaid that they
merely evidence a privilege granted by the State to qualified entities, and do not vest in the
latter a permanent or irrevocable right to the particular concession area and the forest
products therein. They may be validly amended, modified, replaced or rescinded by the Chief
Executive when national interests so require. Thus, they are not deemed contracts within
the purview of the due process of law clause Dismissed.

5.RUZOL V. SANDIGANBAYAN
FACTS: • Leovegildo R. Ruzol was the mayor of General Nakar, Quezon from 2001 to 2004.
• Earlier in his term, he organized a Multi-Sectoral Consultative Assembly composed of civil
society groups, public officials and concerned stakeholders with the end in view of regulating
and monitoring the transportation of salvaged forest products within the vicinity of General
Nakar. • At the organizational meeting for the assembly, the participants agreed that to
regulate the salvaged forests products, the Office of the Mayor, through Ruzol, shall issue a
permit to transport after payment of the corresponding fees to the municipal treasurer. •
From 2001 to 2004, two hundred twenty-one (221) permits to transport salvaged forest
products were issued to various recipients, of which forty-three (43) bore the signature of
Ruzol while the remaining one hundred seventy-eight (178) were signed by his coaccused
Guillermo T. Sabiduria (Sabiduria), then municipal administrator of General Nakar. • 221
Informations for violation of Art. 177 of the RPC or for Usurpation of Authority or Official
Functions were filed against Ruzol and Sabiduria. o Claimed that the authority to issue such
permits belonged to the DENR and not to the Office of the Mayor. • Ruzol’s defense: o As
Chief Executive of the municipality of General Nakar, Quezon, he is authorized to issue
permits to transport forest products pursuant to RA 7160 which give the LGU not only
express powers but also those powers that are necessarily implied from the powers expressly
granted as well as those that are necessary, appropriate or incidental to the LGU’s efficient
and effective governance. Invoked the General Welfare Clause (§16, LGC) o RA 7160 has
devolved certain functions and responsibilities of the DENR to the LGU. And the permits to
transport were issued pursuant to the devolved function to manage and control communal
forests with an area not exceeding fifty (50) square kilometers. o Under (a) Section 5, Article
X of the Constitution, (b) Section 129, Chapter I, Title One Book II of R.A. 7160, and (c)
Section 186, Article Five, Chapter 5, Tile One, Book II of R.A. 7160, the municipality is
granted the power to create its own sources of revenue and to levy fees in accordance
therewith. o The only kind of document the DENR issues relating to log, timber or lumber is
denominated “Certificate of Timber Origin” or CTO for logs and “Certificate of Lumber Origin”
or CLO for lumber. o No proof of conspiracy between the two accused. o The DENR directly
sanctioned and expressly authorized the issuance of the 221 Transport permits through the
Provincial Environment and natural Resources officer Rogelio Delgado Sr., in a Multi-Sectoral
Consultative Assembly. • Sandiganbayan: Acquitted Sabiduria but found Ruzol guilty as
charged. o Cited §5, PD 705 (Forestry Code): [The DENR] shall be responsible for the
protection, development, management, regeneration, and reforestation of forest lands; the
regulation and supervision of the operation of licensees, lessees and permittees for the taking
or use of forest products therefrom or the occupancy or use thereof… o Likewise invoked EO
192 (Reorganizing the DENR). o Finally, citing RA 7160, determined that since the authority
relative to salvaged forest products was not included in the above enumeration of devolved
functions, the correlative authority to issue transport permits remains with the DENR, and
thus cannot be exercised by the LGUs.

ISSUES/HELD:
1. WoN the authority to monitor and regulate the transportation of salvaged forest product
is solely with the DENR, and no one else. (NO)

2. WoN the permits to transport issued by Ruzol are valid. (NO)

3. WoN Ruzol is guilty of usurpation of official functions. (NO) RATIO:

Natre_Midterm SY2019-2020_1stSemester_ P a g e 6 | 68
1. The LGU also has, under the LGC of 1991, ample authority to promulgate rules, regulations
and ordinances to monitor and regulate salvaged forest products, provided that the
parameters set forth by law for their enactment have been faithfully complied with.
• While the DENR is, indeed, the primary government instrumentality charged with the
mandate of promulgating rules and regulations for the protection of the environment and
conservation of natural resources, it is not the only government instrumentality clothed
with such authority.
• While the law has designated DENR as the primary agency tasked to protect the
environment, it was not the intention of the law to arrogate unto the DENR the exclusive
prerogative of exercising this function. Whether in ordinary or in legal parlance, the word
“primary” can never be taken to be synonymous with “sole” or “exclusive.”
• The General Welfare Clause of the LGC states: Every local government unit shall exercise
the powers expressly granted, those necessarily implied therefrom, as well as powers
necessary, appropriate, or incidental for its efficient and effective governance, and those
which are essential to the promotion of the general welfare. Within their respective
territorial jurisdictions, local government units shall ensure and support, among other
things, the preservation and enrichment of culture, promote health and safety, enhance
the right of the people to a balanced ecology, encourage and support the development of
appropriate and self-reliant scientific and technological capabilities, improve public
morals, enhance economic prosperity and social justice, promote full employment among
their residents, maintain peace and order, and preserve the comfort and convenience of
their inhabitants.
• Pursuant to the aforequoted provision, municipal governments are clothed with authority
to enact such ordinances and issue such regulations as may be necessary to carry out
and discharge the responsibilities conferred upon them by law, and such as shall be
necessary and proper to provide for the health, safety, comfort and convenience, maintain
peace and order, improve public morals, promote the prosperity and general welfare of
the municipality and its inhabitants, and ensure the protection of property in the
municipality.
• There is a clear merit to the view that the monitoring and regulation of salvaged forest
products through the issuance of appropriate permits is a shared responsibility which may
be done either by DENR or by the LGUs or by both. o DAO 1992-30: LGUs shall share
with the national government, particularly the DENR, the responsibility in the sustainable
management and development of the environment and natural resources within their
territorial jurisdiction.

2. The Permits to Transport issued by Ruzol are invalid for his failure to comply with the
procedural requirements set forth by law for its enforcement.
• Ruzol insists that the permits partake of the nature of transport fees levied by the
municipality for the use of public roads.
• Ruzol is correct to a point.
o Nevertheless, We find that an enabling ordinance is necessary to confer the subject
permits with validity.
o As correctly held by the Sandiganbayan, the power to levy fees or charges under the
LGC is exercised by the Sangguniang Bayan through the enactment of an appropriate
ordinance wherein the terms, conditions and rates of the fees are prescribed.
• Although We recognize the LGU’s authority in the management and control of communal
forests within its territorial jurisdiction, We reiterate that this authority should be
exercised and enforced in accordance with the procedural parameters established by law
for its effective and efficient execution.
• §17, LGC provides that the LGU’s authority to manage and control communal forests
should be “pursuant to national policies and is subject to supervision, control and review
of DENR.”
• Before an area may be considered a communal forest, the following requirements must
be accomplished: (1) an identification of potential communal forest areas within the
geographic jurisdiction of the concerned city/municipality; (2) a forest land use plan
which shall indicate, among other things, the site and location of the communal forests;
N a t r e _ M i d t e r m S Y 2 0 1 9 - 2 0 2 0 _ 1 s t S e m e s t e r _ P a g e 7 | 68
(3) a request to the DENR Secretary through a resolution passed by the Sangguniang
Bayan concerned; and (4) an administrative order issued by DENR Secretary declaring
the identified area as a communal forest.
• In the present case, the records are bereft of any showing that these requirements were
complied with.

3. Razol’s guilt was not proven beyond reasonable doubt.


• Art. 177 of the RPC provides: Usurpation of authority or official functions. — Any person
who shall knowingly and falsely represent himself to be an officer, agent or representative
of any department or agency of the Philippine Government or of any foreign government,
or who, under pretense of official position, shall perform any act pertaining to any person
in authority or public officer of the Philippine Government or any foreign government, or
any agency thereof, without being lawfully entitled to do so, shall suffer the penalty of
prision correccional in its minimum and medium periods.
• There are two ways of committing the crime:
o First, by knowingly and falsely representing himself to be an officer, agent or
representative of any department or agency of the Philippine Government or of any
foreign government. (Usurpation of authority)
o Second, under pretense of official position, shall perform any act pertaining to any
person in authority or public officer of the Philippine Government or any foreign
government, or any agency thereof, without being lawfully entitled to do so.
(Usurpation of official functions)
• In the present case, Ruzol stands accused of usurpation of official functions for issuing
221 permits to transport salvaged forest products under the alleged “pretense of official
position and without being lawfully entitled to do so, such authority properly belonging to
the Department of Environment and Natural Resources.”
• However, contrary to the ruling of the Sandiganbayan, We find that a careful scrutiny of
the events surrounding this case failed to prove that Ruzol is guilty beyond reasonable
doubt of committing the crime of usurpation of official functions of the DENR.
O We note that this case of usurpation against Ruzol rests principally on the
prosecution’s theory that the DENR is the only government instrumentality that can
issue the permits to transport salvaged forest products.
o But erstwhile discussed at length, the DENR is not the sole government agency vested
with the authority to issue permits relevant to the transportation of salvaged forest
products, considering that, pursuant to the general welfare clause, LGUs may also
exercise such authority.
• Moreover, Ruzol acted in good faith.
o The conduct of a public consultation was a sign supporting Ruzol’s good intentions to
regulate and monitor the movement of salvaged forest products to prevent abuse and
occurrence of untoward illegal logging. DISPOSITION: Acquitted

Natre_Midterm SY2019-2020_1stSemester_ P a g e 8 | 68
6. CRISOSTOMO B. AQUINO vs. MUNICIPALITY OF MALAY AKLAN G.R. No. 211356;
September 29, 2014

DOCTRINE: Based on law and jurisprudence, the office of the mayor has quasi-judicial
powers to order the closing and demolition of establishments. This power granted by the
LGC, as earlier explained, we believe, is not the same power devolved in favor of the LGU
under Sec. 17 (b)(2)(ii), as above-quoted, which is subject to review by the DENR. The fact
that the building to be demolished is located within a forestland under the administration of
the DENR is of no moment, for what is involved herein, strictly speaking, is not an issue on
environmental protection, conservation of natural resources, and the maintenance of
ecological balance, but the legality or illegality of the structure. Rather than treating this as
an environmental issue then, focus should not be diverted from the root cause of this debacle
compliance. Facts: Petitioner is the president and chief executive officer of Boracay Island
West Cove Management Philippines, Inc. (Boracay West Cove). On January 7, 2010, the
company applied for a zoning compliance with the municipal government of Malay, Aklan.2
While the company was already operating a resort in the area, and the application sought
the issuance of a building permit covering the construction of a three-storey hotel over a
parcel of land measuring 998 sqm. located in Sitio Diniwid, Barangay Balagab, Boracay
Island, Malay, Aklan, which is covered by a Forest Land Use Agreement for Tourism Purposes
(FLAgT) issued by the Department of Environment and Natural Resources (DENR) in favor of
Boracay West Cove. Through a Decision on Zoning dated January 20, 2010, the Municipal
Zoning Administrator denied petitioner’s application on the ground that the proposed
construction site was within the “no build zone” demarcated in Municipal Ordinance 2000-
131 (Ordinance). Petitioner appealed the denial action to the Office of the Mayor but despite
follow up, no action was ever taken by the respondent mayor. A Cease and Desist Order was
issued by the municipal government, enjoining the expansion of the resort, and on June 7,
2011, the Office of the Mayor of Malay, Aklan issued the assailed EO 10, ordering the closure
and demolition of Boracay West Cove’s hotel. Petitioner filed a Petition for Certiorari with
prayer for injunctive relief with the CA Alleging that the order was issued and executed with
grave abuse of discretion.

PETITIONER’s CONTENTION: The hotel cannot summarily be abated because it is not a


nuisance per se, given the hundred million peso-worth of capital infused in the venture. And
the Municipality of Malay, Aklan should have first secured a court order before proceeding
with the demolition.

RESPONDENT’s CONTENTION: The demolition needed no court order because the


municipal mayor has the express power under the Local Government Code (LGC) to order
the removal of illegally constructed buildings. Issue: Whether or not a judicial proceeding be
conducted first before the LGU can order the closure and demolition of the property in
question. Held: Generally, LGUs have no power to declare a particular thing as a nuisance
unless such a thing is a nuisance per se. Despite the hotel’s classification as a nuisance per
accidens, however, we still find in this case that the LGU may nevertheless properly order
the hotel’s demolition. This is because, in the exercise of police power and the general welfare
clause, property rights of individuals may be subjected to restraints and burdens in order to
fulfill the objectives of the government. Otherwise stated, the government may enact
legislation that may interfere with personal liberty, property, lawful businesses and
occupations to promote the general welfare. Under the law, insofar as illegal constructions
are concerned, the mayor can, after satisfying the requirement of due notice and hearing,
order their closure and demolition. One such piece of legislation is the LGC, which authorizes
city and municipal governments, acting through their local chief executives, to issue
demolition orders. Under existing laws, the office of the mayor is given powers not only
relative to its function as the executive official of the town; it has also been endowed with
authority to hear issues involving property rights of individuals and to come out with an
effective order or resolution thereon. Pertinent herein is Sec. 444 (b) (3) (vi) of the LGC,
which empowered the mayor to order the closure and removal of illegally constructed
establishments for failing to secure the necessary permits.
N a t r e _ M i d t e r m S Y 2 0 1 9 - 2 0 2 0 _ 1 s t S e m e s t e r _ P a g e 9 | 68
7. G.R. No. L-36847 July 20, 1983SERAFIN B. YNGSON vs. THE HON. SECRETARY
OF AGRICULTURE and NATURAL RESOURCES, ANITA V. DEGONZALES and JOSE M.
LOPEZ
Facts:
A number of people, including the petitioner and private respondents, applied for a permit
to utilize 66 hectares of mangrove swamps for fishpond purposes, but were not granted, the
area still being considered to be communal forest. When the area was released for said
purpose, the three private parties in this case had conflicting claims on the same. Initially,
the Bureau of Fisheries awarded the whole area in favor of petitioner. However, the Sec. of
Agriculture and Natural Resources later ordered the division of the area into three portions,
one part for each of the petitioner and the private respondents. Not satisfied with the portion
received, petitioner appealed the order.

Issue: Whether or not petitioner is entitled to the whole of the area concerned.

Held:
No. It is elementary in the law governing the disposition of lands of the public domain that
until timber or forest lands are released as disposable and alienable neither the Bureau of
Lands nor the Bureau of Fisheries has authority to lease, grant, sell, or otherwise dispose of
these lands for homesteads, sales patents, leases for grazing or other purposes, fishpond
leases, and other modes of utilization. The Bureau of Fisheries has no jurisdiction to
administer and dispose of swamplands or mangrove lands forming part of the public domain
while such lands are still classified as forest land or timberland and not released for fishery
or other purposes. All the applications in this case were premature; therefore, not one of the
applicants can claim to have a preferential right over another. The interpretation by the
Office of the President was held to be an exercise of sound discretion which should not be
disturbed.

8.REPUBLIC VS. CA AND NAGUIT


FACTS:
Corazon Naguit filed a petition for registration of title which seeks judicial confirmation of
her imperfect title over a parcel of land in Nabas, Aklan. It was alleged that Naguit and her
predecessors-in-interest have occupied the land openly and in the concept of owner without
any objection from any private person or even the government until she filed her application
for registration. The MCTC rendered a decision confirming the title in the name of Naguit
upon failure of Rustico Angeles to appear during trial after filing his formal opposition to the
petition. The Solicitor General, representing the Republic of the Philippines, filed a motion
for reconsideration on the grounds that the property which is in open, continuous and
exclusive possession must first be alienable. Naguit could not have maintained a bona fide
claim of ownership since the subject land was declared as alienable and disposable only on
October 15, 1980. The alienable and disposable character of the land should have already
been established since June 12, 1945 or earlier.

ISSUE:
Whether or not it is necessary under Section 14 (1) of the Property Registration Decree that
the subject land be first classified as alienable and disposable before the applicant’s
possession under a bona fide claim of ownership could even start.

RULING:
Section 14 (1) merely requires that the property sought to be registered as already alienable
and disposable at the time the application for registration of title is filed. There are three
requirements for registration of title, (1) that the subject property is alienable and
disposable; (2) that the applicants and their predecessor-in-interest have been in open,
continuous, and exclusive possession and occupation, and; (3) that the possession is under
a bona fide claim of ownership since June 12, 1945. There must be a positive act of the
N a t r e _ M i d t e r m S Y 2 0 1 9 - 2 0 2 0 _ 1 s t S e m e s t e r _ P a g e 10 | 68
government through a statute or proclamation stating the intention of the State to abdicate
its exclusive prerogative over the property, thus, declaring the land as alienable and
disposable. However, if there has been none, it is presumed that the government is still
reserving the right to utilize the property and the possession of the land no matter how long
would not ripen into ownership through acquisitive prescription. To follow the Solicitor
General’s argument in the construction of Section 14 (1) would render the paragraph 1 of
the said provision inoperative for it would mean that all lands of public domain which were
not declared as alienable and disposable before June 12, 1945 would not be susceptible to
original registration, no matter the length of unchallenged possession by the occupant. In
effect, it precludes the government from enforcing the said provision as it decides to
reclassify lands as alienable and disposable. The land in question was found to be cocal in
nature, it having been planted with coconut trees now over fifty years old. The inherent
nature of the land but confirms its certification in 1980 as alienable, hence agricultural. There
is no impediment to the application of Section 14 (1) of the Property Registration Decree.
Naguit had the right to apply for registration owing to the continuous possession by her and
her predecessors-in-interest of the land since 1945.

REPUBLIC VS. CA AND NAGUIT


FACTS:
Respondent applies for registration of title to 4 parcels of land contending she is the owner
of the said land which she acquired from the LID Corporation which in turn acquired the
same from persons who have been in possession thereof for more than 30 years. The
Republic filed in opposition that said lands belong to the public domain and not subject to
private appropriation.

ISSUE:
Whether or not the land in dispute as a forest land belonging to public domain may be
appropriated as private property.

Ruling:
For a public forest land/reserves to be subject for private appropriation, it requires an
express and positive act of the government that it will become a part of alienable and
disposable agricultural lands of public domain. Occupation in the concept of an owner cannot
ripen into private ownership and be registered to as a title.

9. LAGUA VS CUSI, 160 SCRA 69 (1983)


FACTS:
A memorandum was issued preventing the passage of Plaintiff Lagua’s' hauling trucks loaded
with logs for the Japanese vessel on the national highway loading towards where the vessel
was berthed. In compliance with this directive, the security force of Defendant Eastcoast
closed the road to the use by plaintiff’s trucks and other equipments and effectively
prevented their passage thereof while the vehicles and trucks of other people were curiously
not disturbed and were allowed passess on the same road. It resulted that the loading of
logs on the M/S"Kyofuku Maru" was discontinued. When Plaintiffs Laguas were already
resuming the hauling operations of their logs towards the Japanese Vessel, again that same
road, only the day before ordered by the BFD to be opened for use and passage by plaintiffs,
was closed to them by Defendant Eastcoast's security men upon a radio message order of
Defendant Maglana. Even the vessel M/S "Kyofuku Maruwas" ordered by Defendant Maglana
to untie her anchor contrary to existing laws, rules and regulations of the Bureau of Customs
and the Philippine Coastguard. Given no recourse in the face of the blatant and illegal closure
of the road in defiance of BFD orders to the contrary by the Defendant East coast through
the order of Defendant Maglana, Plaintiff Laguas had to depart post paste to Mati, Davao
Oriental, from Baganga where the shipment and the road closure were made, to seek the
assistance of the PC thereat. The private respondents filed a motion to dismiss argued that
petitioner Daylinda Laguas has no capacity to sue as her name was not registered as an
N a t r e _ M i d t e r m S Y 2 0 1 9 - 2 0 2 0 _ 1 s t S e m e s t e r _ P a g e 11 | 68
"agent" or "dealer" of logs in the Bureau of Forestry. The Court agrees with the defendants
that under the law, the Bureau of Forest Development has the exclusive power to regulate
the use of logging road and to determine whether their use is in violation of laws.

ISSUE:
Whether or not the Bureau of Forest Development has the jurisdiction to determine first the
legality of closure of logging roads before the case be directed to the regular courts

HELD:
NO. P.D. No. 705 upon which the respondent court based its order does not vest any power
in the Bureau of Forest Development to determine whether or not the closure of a logging
road is legal or illegal and to make such determination a pre-requisite before an action for
damages may be maintained. Moreover, the complaint instituted by the petitioners is clearly
for damages based on the alleged illegal closure of the logging road. Whether or not such
closure was illegal is a matter to be established on the part of the petitioners and a matter
to be disproved by the private respondents. This should appropriately be threshed out in a
judicial proceeding. It is beyond the power and authority of the Bureau of Forest
Development to determine the unlawful closure of a passage way, much less award or deny
the payment of damages based on such closure. Not every activity inside a forest area is
subject to the jurisdiction of the Bureau of Forest Development. As we have held in Ateneo
de Manila University v. Court of appeals (145 SCRA 100, 110):

10. HEIRS OF AMUNATEGUI VS DIRECTOR OF FORESTRY, 126 SCRA 69 (1983)


FACTS:
There were two petitions for review on certiorari questioning the decision of the Court of
Appeals which declared the disputed property as forestland, not subject to titling in favor of
private persons, Borre and Amunategui. The Director of Forestry, through the Provincial
Fiscal of Capiz, also filed an opposition to the application for registration of title claiming that
the land was mangrove swamp which was still classified as forest land and part of the public
domain. Another oppositor, Emeterio Bereber filed his opposition insofar as a portion of Lot
No. 885containing 117,956 square meters was concerned and prayed that title to said portion
be confirmed and registered in his name.

Issue:
Whether or not the lot in question can be subject of registration and confirmation of title in
the name of the private person

HELD:
The opposition of the Director of Forestry was strengthened by the appellate court's finding
that timber licenses had to be issued to certain licensees and even Jose Amunategui himself
took the trouble to ask for a license to cut timber within the area. It was only
sometimein1950 that the property was converted into fishpond but only after a previous
warning from the District Forester that the same could not be done because it was classified
as "public forest ‖. A forested area classified as forest land of the public domain does not
lose such classification simply because loggers or settlers may have stripped it of its forest
cover. "Forestlands" do not have to be on mountains or in out of the way places. Swampy
areas covered by mangrove trees, nipa palms, and other trees growing in brackish or sea
water may also be classified as forest land. The possession of forest lands, no matter how
long, cannot ripen into private ownership. Therefore, the lot in question never ceased to be
classified as forestland of public domain.

Natre_Midterm SY2019-2020_1stSemester_ P a g e 12 | 68
11. MERIDA V PEOPLE (Natural Resources)
MERIDA V PEOPLE (DEFINITION OF TIMBER, AUTHORITY OF FOREST OFFICERS)
G.R. No. 158182
June 12, 2008

FACTS:
On 23 December 1998, Tansiongco learned that petitioner cut a narra tree in the Mayod
Property. Tansiongco reported the matter to Florencio Royo (Royo), the punong barangay of
Ipil. On 24 December 1998, 7 Royo summoned petitioner to a meeting with Tansiongco.
When confronted during the meeting about the felled narra tree, petitioner admitted cutting
the tree but claimed that he did so with the permission of one Vicar Calix (Calix) who,
according to petitioner, bought the Mayod Property from Tansiongco in October 1987 under
a pacto de retro sale. Petitioner showed to Royo Calix's written authorization signed by Calix's
wife.

On 11 January 1999, Tansiongco reported the tree-cutting to the Department of Environment


and Natural Resources (DENR) forester Thelmo S. Hernandez (Hernandez) in Sibuyan,
Romblon.

DECISION OF LOWER COURTS:


* DENR forester: ordered petitioner not to convert the felled tree trunk into lumber.
On 26 January 1999, Tansiongco informed Hernandez that petitioner had converted the narra
trunk into lumber. Hernandez, with other DENR employees and enforcement officers, went
to the Mayod Property and saw that the narra tree had been cut into six smaller pieces of
lumber. Hernandez took custody of the lumber, 9 deposited them for safekeeping with Royo,
and issued an apprehension receipt to petitioner. A larger portion of the felled tree remained
at the Mayod Property. The DENR subsequently conducted an investigation on the matter.

* RTC (upon complaint of Tansiongco): Petitioner was charged in the Regional Trial Court of
Romblon, Romblon, Branch 81 (trial court) with violation of Section 68 of PD 705, as
amended, for "cut[ting], gather[ing], collect[ing] and remov[ing]" a lone narra tree inside a
private land in Mayod, Ipil, Magdiwang, Romblon (Mayod Property) over which private
complainant Oscar M. Tansiongco (Tansiongco) claims ownership.

* CA: affirmed trial court.

ISSUES & RULINGS:


1) Whether the trial court acquired jurisdiction over Criminal Case No. 2207 even though it
was based on a complaint filed by Tansiongco and not by a DENR forest officer; and YES,
DENR has jurisdiction.
[NOTE: This dispositive no longer applicable since the Rules of Procedure for Environmental
cases requires complaint to be filed first with the DENR, but the preliminary investigation is
done by the prosecutor]

Section 80 of PD 705 provides in relevant parts:


SECTION 80. Arrest; Institution of criminal actions. - x x x x
Reports and complaints regarding the commission of any of the offenses defined in this
Chapter, not committed in the presence of any forest officer or employee, or any of the
deputized officers or officials, shall immediately be investigated by the forest officer assigned
in the area where the offense was allegedly committed, who shall thereupon receive the
evidence supporting the report or complaint.

If there is prima facie evidence to support the complaint or report, the investigating forest
officer shall file the necessary complaint with the appropriate official authorized by law to
conduct a preliminary investigation of criminal cases and file an information in Court.
(Emphasis supplied)

Natre_Midterm SY2019-2020_1stSemester_ P a g e 13 | 68
Here, it was not "forest officers or employees of the Bureau of Forest Development or any of
the deputized officers or officials" who reported to Hernandez the tree-cutting in the Mayod
Property but Tansiongco, a private citizen who claims ownership over the Mayod Property.
Thus, Hernandez cannot be faulted for not conducting an investigation to determine "if there
is prima facie evidence to support the complaint or report."

At any rate, Tansiongco was not precluded, either under Section 80 of PD 705 or the Revised
Rules, from filing a complaint before the Provincial Prosecutor for petitioner's alleged
violation of Section 68 of PD 705, as amended. For its part, the trial court correctly took
cognizance of Criminal Case No. 2207 as the case falls within its exclusive original
jurisdiction.

2) Whether petitioner is liable for violation of Section 68 of PD 705, as amended. YES.

Before his trial, petitioner consistently represented to the authorities that he cut a narra tree
in the Mayod Property and that he did so only with Calix's permission. However, when he
testified, petitioner denied cutting the tree in question. We sustain the lower courts' rulings
that petitioner's extrajudicial admissions bind him.

3) Is the narra tree timber? YES.

The closest this Court came to defining the term "timber" in Section 68 was to provide that
"timber," includes "lumber" or "processed log."

In other jurisdictions, timber is determined by compliance with specified dimensions or


certain "stand age" or "rotation age." In Mustang Lumber, Inc. v. Court of Appeals, this Court
was faced with a similar task of having to define a term in Section 68 of PD 705 - "lumber"

- to determine whether possession of lumber is punishable under that provision. In ruling in


the affirmative, we held that "lumber" should be taken in its ordinary or common usage
meaning to refer to "processed log or timber,"

We see no reason why, as in Mustang, the term "timber" under Section 68 cannot be taken
in its common acceptation as referring to "wood used for or suitable for building or for
carpentry or joinery." Indeed, tree saplings or tiny tree stems that are too small for use as
posts, paneling, beams, tables, or chairs cannot be considered timber.

Undoubtedly, the narra tree petitioner felled and converted to lumber was "timber" fit "for
building or for carpentry or joinery" and thus falls under the ambit of Section 68 of PD 705,
as amended.

Natre_Midterm SY2019-2020_1stSemester_ P a g e 14 | 68
Topic: Doctrine of Primary Jurisdiction
12. PROVIDENT TREE FARMS, INC. (PTFI) VS HON. BATARIO & AJ INTERNATIONL
CORP. (AJIC)
GR 92285 March 28, 1994
DOCTRINE:
Cases before the BOC must be fully fleshed out before it prior to elevating the issues to a
regular court in keeping with the exhaustion of administrative remedies. (primary
jurisdiction)

FACTS:
 PTFI is a Phil corporation engaged in industrial tree planting. It supplies to a local
match manufacturer solely for production of matches.
 There’s a state policy to encourage qualified persons to engage in industrial tree
plantation under Revised Forestry Code which provides a set of incentives to
corporations like PTFI and is a qualified ban against importation of wood.
 Respondent, AJIC, imported matches from Indonesia which the BOC released which
violates the Revised Forestry Code’s ban of importing wood and wood-derivated
products.

RTC:
 PTFI filed with the RTC of Manila a complaint for injunction and damages with prayer
for a TRO against Commissioner of Customs to prohibit the latter from importing
matches.
 AJIC moved to dismiss the complaint alleging that:
o The Commissioner of Customs and not the regular court has exclusive
jurisdiction to determine the legality of an importation.
o The release of importations had rendered injunction moot and academic.
o The prayer for damages has no basis as the Commissioner’s acts are in
accordance with law.
o The complaint for injunction cannot stand it being only a provisional relief and
not a principal remedy.
 PTFI opposed the motion to dismiss.
 AJIC’s motion to dismiss was denied.
 AJIC filed a motion for reconsideration and the Court reconsidered and dismissed the
case on the ground that it had no jurisdiction to determine what are legal or
illegal importations.

 PTFI seeks to set aside the order of respondent court and prays for the continuation of
the hearing of the case contending that what was brought before the trial court was a
civil case for injunction for the purpose of securing compliance with the provision of
the RFC.

ISSUE:
Whether or not the Bureau of Customs holds jurisdiction in the matter of wood product
importation.

HELD:
The only subject of this incentive is a ban against importation of wood and wood products
which is to be enforced by Bureau of Customs since it has under the Tariff and Customs Code
the exclusive original jurisdiction over seizure and forfeiture cases. To allow the regular court
to direct the Commissioner is clearly an interference with the exclusive jurisdiction of the
BOC.

PTFI’c correspondence with the BOC contesting the legality of match importations may
already take the nature of administrative proceedings the pendency of which would preclude
the court from interfering with it under the doctrine of primary jurisdiction.

Natre_Midterm SY2019-2020_1stSemester_ P a g e 15 | 68
In Presidential Commission on Good Government v. Peña, 20 the court held that —
. . . . under the "sense-making and expeditious doctrine of primary
jurisdiction . . . the courts cannot or will not determine a controversy involving
a question which is within the jurisdiction of an administrative tribunal, where
the question demands the exercise of sound administrative discretion requiring
the special knowledge, experience, and services of the administrative tribunal to
determine technical and intricate matters of fact, and a uniformity of ruling is
essential to comply with the purposes of the regulatory statute administered
(Pambujan Sur United Mine Workers v. Samar Mining Co., Inc., 94 Phil. 932, 941
[1954].)
In this era of clogged court dockets, the need for specialized administrative
boards or commissions with the special knowledge, experience and capability to
hear and determine promptly disputes on technical matters or essentially factual
matters, subject to judicial review in case of grave abuse of discretion, has
become well indispensable . . . .

The court cannot compel an agency to do a particular act or to enjoin such act which is within
its prerogative, except when in the exercise of its authority it gravely abuses or exceeds its
jurisdiction. In the case at bench, we have no occasion to rule on the issue of grave abuse
of discretion or excess of jurisdiction as it is not before us.

Thus, the order of the RTC was affirmed and the petition for review was denied.

13. MOMONGAN V OMIPON


At around 10:00 o'clock of November 14, 1992, police officers of the Municipality of
Hinunangan, Southern Leyte apprehended Dionisio Golpe while he was driving his truck
loaded with illegally cut lumber. The truck and logs were impounded. A complaint was filed
against Basilio Cabig, the alleged owner of the logs. After conducting the preliminary
investigation, respondent Judge Rafael B. Omipon found that a prima facie case exists
against Mr. Cabig but he ordered the release of the truck inasmuch as the owner/driver, Mr.
Golpe, was not charged in the complaint.

Regional Director Augustus L. Momongan of the Department of Environment and Natural


Resources filed the instant complaint against respondent Judge alleging that his order
releasing the truck used in the transport of illegally cut forest products violated Presidential
Decree 705, as amended by Executive Order No. 277, Section 68 and 68-A1and
Administrative Order No. 59, Series of 1990.2 Complainant claims that respondent Judge has
no authority to order the release of the truck despite the non-inclusion of Mr. Golpe in the
complaint. The truck should have been turned over to the Community Environment and
Natural Resources Office of San Juan, Southern Leyte for appropriate disposition as the same
falls under the administrative jurisdiction of the Department of Environment and Natural
Resources Office.

In his comment, respondent Judge explained that after conducting the preliminary
investigation, he found that Golpe, the owner of the truck, is principally engaged in the
hauling of sand and gravel and the delivery of hollow blocks. On his way home after delivering
hollow blocks in Barangay Sto. Niño II, he met his friend Cabig who requested him to load
sliced lumber and deliver the same at Brgy. Lungsod-daan, Hinundayan to be used for the
construction of a barangay high school building. They were apprehended when the truck had
a flat tire. After changing the tire, both the lumber and the truck were ordered deposited at
the police station of Hinunangan.

Respondent Judge observed that Golpe has a lesser participation in the crime of illegal
logging and, being a mere accessory, he might be utilized by the Acting Chief of Police as
prosecution witness against Cabig. More importantly, the fact that the complaint charged

Natre_Midterm SY2019-2020_1stSemester_ P a g e 16 | 68
only Cabig, respondent Judge, in the exercise of his sound discretion, ordered the release of
the truck owned by Golpe.

The Memorandum of the Office of the Court Administrator recommended that a formal
investigation be conducted. An excerpt from its Memorandum states:

We find the explanation of respondent unsatisfactory. While he is authorized to


conduct preliminary investigation in all cases of violations of P.D. 705, as
amended, otherwise known as the Revised Forestry Code of the Philippines, Sec.
68-A thereof provides that it is the Department Head or his duly authorized
representative who may order the confiscation and disposition of the forest
products illegally cut, gathered, removed, or possessed or abandoned, and all
conveyances used either by land, water or air in the commission of the offense
and to dispose of the same in accordance with pertinent laws, regulations or
policies on the matter.

There may be some facts that are not extant in the records which can only come
out during a formal investigation to better establish clear culpability or
exoneration over the respondent.

In view thereof, and to give respondent an opportunity to clear himself, it is


respectfully recommended that this matter be referred to Acting Executive Judge
Leandro T. Loyao, Jr., RTC, Branch 26, San Juan, Southern Leyte, for
investigation, report and recommendation within sixty days from receipt of the
records.3

In the Resolution of November 8, 1993, the Court resolved to refer the case to Acting
Executive Judge Leandro T. Loyao, Jr., RTC, Branch 26, San Juan, Southern Leyte, for
investigation, report and recommendation, within sixty (60) days from receipt of the
records.4

During the first two hearing dates, complainant was unable to attend but sent his
representatives, DENR lawyer Constantino Esber and legal assistant Romeo Gulong.
Respondent Judge appeared with his counsel. However, on the third hearing date,
respondent Judge failed to appear as he suffered a stroke and was hospitalized. Thereafter,
DENR counsel Esber manifested that their office has filed a motion for reinvestigation and
for the turnover of the jeep to the PNP and subsequently, to the DENR. He also manifested
that the complainant is submitting the administrative matter for resolution and
recommendation without adducing evidence against respondent. Respondent's counsel did
not object to complainant's manifestation. The counsel of both complainant and respondent
jointly agreed to submit the case for appropriate action.

The Investigating Judge's confidential report, in part, states:

In view of this development in the course of an intended investigation this investigator


could not elicit additional facts than are found in the records, whether inculpatory or
exculpatory. Respondent was given an opportunity to explain the unfavorable
circumstances against him but he was overtaken by a serious illness. So much was
expected from the complainant to supply the facts not extant in the records, but he lost
interest in substantiating his April 1993 report to the Supreme Court. In fact, he was
submitting this administrative matter for resolution without adducing evidence against
respondent.

Except for the 21 January 1994 motion for reinvestigation of DENR counsel Esber which
sought for the inclusion of jeep owner and driver Dionisio Golpe in the criminal information,
there is nothing new that can be added to the facts found by the Honorable Deputy Court

Natre_Midterm SY2019-2020_1stSemester_ P a g e 17 | 68
Administrator as reflected in his Memorandum for the Honorable Chief Justice dated 12
October 1993.

There being no actual investigation conducted, no additional facts could be reported and
consequently, there is no basis for a recommendation on the basis of facts.

This investigator can only recommend appropriate action by the Supreme Court on the
basis of the facts already extant in the records with a prayer for consideration of
respondent plight especially so since on account of this investigation his health has
deteriorated and may affect his efficiency output as a judge. Perhaps, allowing him to bow
out of the service with honor and corresponding benefits.5

During the pendency of this case, respondent Judge filed for disability retirement. His
application was approved but his pension was not released pending the outcome of this case.
We find respondent Judge's order to release the truck owned and driven by Mr. Dionisio
Golpe legally justifiable, hence, he is not subject to any disciplinary sanction.

According to the Revised Penal Code, Art. 45, first paragraph: "[E]very penalty imposed for
the commission of a felony shall carry with it the forfeiture of the proceeds of the crime and
the instrument or tools with which it was committed." However, this cannot be done if such
proceeds and instruments or tools "be the property of a third person not liable for offense."
In this case, the truck, though used to transport the illegally cut lumber, cannot be
confiscated and forfeited in the event accused therein be convicted because the truck
owner/driver, Mr. Dionisio Golpe was not indicted. Hence, there was no justification for
respondent Judge not to release the truck.

Complainant is correct in pointing out that based on Pres. Decree No. 705, Sec. 68-A and
Adm. Order No. 59, the DENR Secretary or his duly authorized representative has the power
to confiscate any illegally obtained or gathered forest products and all conveyances used in
the commission of the offense and to dispose of the same in accordance with pertinent laws.
However, as complainant himself likewise pointed out, this power is in relation to the
administrative jurisdiction of the DENR.

We do not find that when respondent Judge released the truck after he conducted the
preliminary investigation and satisfied himself that there was no reason to continue keeping
the truck, he violated Pres. Decree No. 705 and Adm. Order No. 59. The release of the truck
did not render nugatory the administrative authority of the DENR Secretary. The confiscation
proceedings under Adm. Order No. 596 is different from the confiscation under the Revised
Penal Code, which is an additional penalty imposed in the event of conviction. Despite the
order of release, the truck can be seized again either by filing a motion for reinvestigation
and motion to include the truck owner/driver, as co-accused, which complainant has done
as manifested before the lower court or by enforcing Adm. Order No. 59. Section 12 thereof
categorically states that "[t]he confiscation of the conveyance under these regulations shall
be without prejudice to any criminal action which shall be filed against the owner thereof or
any person who used the conveyance in the commission of the offense."

Petitioner is of the opinion that under the circumstances, respondent Judge should have
turned over the truck to the Community Environment and Natural Resources Office (CENRO)
of San Juan, Southern Leyte for appropriate disposition. No doubt, this would have simplified
matters and prevented the present situation from occurring wherein one government official
files a complaint against another. Under Sec. 4 of Adm. Order No. 59, if the apprehension is
not made by DENR field offices, deputized military personnel and officials of other agencies
apprehending illegal logs and other forest products and their conveyances shall notify the
nearest DENR field offices and turnover said forest products and conveyances for proper
action and disposition. A period of about two weeks lapsed from the time the seizure was
made before a complaint was filed. During this period, the apprehending policemen had
enough time to turn over the logs and the truck to the nearest DENR field office for proper
N a t r e _ M i d t e r m S Y 2 0 1 9 - 2 0 2 0 _ 1 s t S e m e s t e r _ P a g e 18 | 68
action and disposition since the duty to turn over the truck to the nearest DENR field office
rests on the officials apprehending the illegal logs. There being no mandatory duty on the
part of respondent Judge to turn over the truck, he should not be visited with disciplinary
sanction when he did not refer the same to the DENR field office in San Juan, Southern Leyte.
The Court takes this opportunity to enjoin the National Police, the DENR, the prosecutors,
and the members of the bench to coordinate with each other for a successful campaign
against illegal logging. It behooves all the concerned agencies to seriously strive for the
attainment of the constitutionally-declared policy to "protect and advance the right of the
people to a balanced and healthful ecology in accord with the rhythm and harmony of
nature"7 in order to preserve our natural resources for the benefit of the generations still to
come.

WHEREFORE, the complaint is DISMISSED.

14. INTERNATIONAL HARDWOOD AND VENEER COMPANY OF THE PHILIPPINES,


petitionerappellee, vs. UNIVERSITY OF THE PHILIPPINES and JOSE C. CAMPOS, JR.,
respondentsappellants
G.R. No. 52518. August 13, 1991
J. Davide, Jr.
FACTS:
Kind of action: a special civil action for declaratory relief with injunction Effects of cession
in full ownership of a land removed from public domain Hardwood is engaged in the
manufacture, processing and exportation of plywood and was, for said purpose, granted by
the Government an exclusive license for a period of 25 years expiring on February 1, 1985.
Sometime on September 25, 1961, during the effectivity of License Agreement President
Garcia the issued EO No. 791 thereby withdrawing from sale or settlement and
reserving for the College of Agriculture, University of the Philippines, as experiment
station for the proposed Dairy Research and production studies of this College, a certain
parcel of land of the Public domain situated partly in the municipalities of Paete and Pakil,
province of Laguna, and partly in the municipality of Infanta, Province of Quezon, subject to
private rights, if any there be, and to the condition that the disposition of timber and other
forest products found therein shall be subject to the forestry laws and regulations.

On or about June 18, 1964, during the effectivity of the License RA 3990 was enacted by
the Congress and approved by the President, which provides for a Central Experiment Station
for the University of the Philippines. Sec. 2 of the law provides; “the parcel of the public
domain consisting of three thousand hectares, more or less, located in the Municipality of
Paete, Province of Laguna, the precise boundaries of which are stated in EO 791, Series of
1961, is hereby ceded and transferred in full ownership to the University of the
Philippines, subject to any existing concessions, if any.”

On the strength of RA 3990, UP demanded verbally and in writing to Hardwood that (1)
Hardwood shall pay the forest charges under the License Agreement to UP and (2) UP
Personnel shall perform the selling of any timber felled or cut by plaintiff within the
boundaries of the Central Experiment Station.

The civil case was filed by petitioner Hardwood before the trial court on 28 June 1966.
Petitioner Hardwood (1) seeks a declaration that respondent University of the Philippines
(UP) does not have the right to supervise and regulate the cutting and removal of timber
and other forest products covered by the License Agreement, ceded in full ownership to the
UP by RA. 3990; (2) asks that respondents be enjoined from committing the acts complained
of; and (3) prays that respondents be required to pay petitioner the sum of P100,000.00 as
damages and costs of the suit.

Both parties submitted a joint stipulation of facts and prayed that the Court grant relief on
the following
N a t r e _ M i d t e r m S Y 2 0 1 9 - 2 0 2 0 _ 1 s t S e m e s t e r _ P a g e 19 | 68
ISSUES:
1) WoN Hardwood should pay forest charges due and payable under its Timber License
Agreement to UP or to the BIR?

2) WoN UP is entitled to supervise, through its duly appointed personnel, the logging,
felling and removal of timber within the Central Experiment Station area as described
in Republic Act No. 3990, and to scale the timber thus felled and cut?

RTC Ruling: Ruling in favor of Hardwood. The demands of UP have no legal basis. The
cession in full ownership of the tract of land under RA 3990 was expressly made ‘subject to
any existing concessions.’ Inasmuch as at the time of RA 3990, the Hardwood’s timber
concession over the tract of land was existing and would continue to exist until February 1,
1985, UP will acquire ‘full ownership’ and exclusive jurisdiction to control and administer the
property only after February 1, 1985.

Arguments of opposing sides before the SC:


UP HARDWOOD
A. Under RA 3990, UP may collect UP has not been granted by R.A. No. 3990 the
forest charges because the authority to collect forest charges or the
Timber License Agreement authority to supervise the operation by the
does not expressly provide that petitioner of the timber concession affected by
the forest charges shall be paid said Act.
to the BIR Legislative grants must be construed strictly in
favor of the public and most strongly against the
grantee, and nothing will be included in the grant
except that which is granted expressly or by clear
implication.
BIR- the duties incident to the measuring of
forest products and the collection of the charges
(Sec.262 of the Tax Code)
Bureau of Forestry-collection of reforestation
fees
B. Having been vested with The cession in full ownership of the land in
administrative jurisdiction over question was expressly made “subject to any
and being the owner of the concession, if any”, Hardwood’s concession
tract of land in question, UP would continue until 1 February 1985;
acquired full control and UP then would acquire full ownership and
benefit of the timber and other exclusive jurisdiction to control and administer
resources within the area the property only after 1 February 1985. The
position of UP is akin to that of a donee of a
parcel of land subject to usufruct.
C. Section 3 of R.A. No. 3990
expressly provides: “any
incidental receipts or income
therefrom shall pertain to the
general fund of the University
of the Philippines.”
D. R.A. No. 3990, UP is duty
bound to operate and maintain
a central experiment station,
since the law does not provide
for appropriations for such
purpose, it is clearly the
legislative intention that the
establishment and
maintenance thereof must be
Natre_Midterm SY2019-2020_1stSemester_ P a g e 20 | 68
financed by the earnings or
income from the area
E. Supervision of the License
Agreement in favor of
petitioner by UP was intended
by R.A. No. 3990
F. The two government agencies The rulings of the Commissioner of Internal
affected by R.A. No. 3990 have Revenue and the Acting Director of the Bureau of
issued specific rulings Forestry are patently incorrect; moreover, said
recognizing the authority of UP agencies do not have the power to interpret the
to collect royalties or charges law, which is primarily a function of the judiciary
and to supervise petitioner’s
logging operations.

SC Ruling:
1. Hardwood shall pay forest charges to the University of the Philippines.

When R.A. No. 3990 which established a central experiment station for the use of UP
in connection with its research and extension functions, the above “reserved” area was
“ceded and transferred in full ownership to the University of the Philippines subject
to any existing concessions, if any.”

When it ceded and transferred the property to UP, the Republic of the Philippines
completely removed it from the public domain and, more specifically, in respect
to the areas covered by the timber license of petitioner, removed and
segregated it from a public forest; it divested itself of its rights and title
thereto and relinquished and conveyed the same to UP; and made the latter
the absolute owner thereof, subject only to the existing concession.

That the law intended a transfer of the absolute ownership is unequivocally evidenced
by its use of the word “full” to describe it. Full means entire, complete, or possessing
all particulars, or not wanting in any essential quality. The proviso regarding existing
concessions refers to the timber license of petitioner. All that it means, however, is
that the right of petitioner as a timber licensee must not be affected, impaired or
diminished; it must be respected. But, insofar as the Republic of the Philippines
is concerned, all its rights as grantor of the license were effectively assigned,
ceded and conveyed to UP as a consequence of the above transfer of full
ownership. This is further borne out by Section 3 of R.A. No. 3990 which provides,
inter alia, that “any incidental receipts or income therefrom shall pertain to the general
fund of the University of the Philippines.”

Having been effectively segregated and removed from the public domain or
from a public forest and, in effect, converted into a registered private
woodland, the authority and jurisdiction of the Bureau of Forestry over it were
likewise terminated.

Hardwood, as licensee or grantee of the concession, has been given the license to cut,
collect, and remove timber from the area ceded and transferred to UP until 1 February
1985. However, it has the correlative duty and obligation to pay the forest charges, or
royalties, to the new owner, the UP, at the same rate as provided for in the Agreement.
The charges should not be paid anymore to the Republic of the Philippines
through the Bureau of Internal Revenue because of the very nature of the
transfer as aforestated. Consequently, even the Bureau of Internal Revenue
automatically lost its authority and jurisdiction to measure the timber cut
from the subject area and to collect forestry charges and other fees due
thereon.
N a t r e _ M i d t e r m S Y 2 0 1 9 - 2 0 2 0 _ 1 s t S e m e s t e r _ P a g e 21 | 68
2. Based on the nature of the transfer, it follows then that respondent UP is entitled to
supervise, through its duly appointed personnel, the logging, felling, and removal of
timber within the area covered by R.A. No. 3990.

15. TABAO V LILAGAN


FACT:
On February 24, 1998, a water craft M/L Hadja, from Bongao, Tawi-tawi, was docked at the
port area of Tacloban City with a load of 100 tons of tanbark. Robert Hernandez was the
consignee to said cargo. While the cargo was being unloaded, the NBI decided to verify the
shipment's accompanying documents where it was found to be irregular and incomplete.
Consequently, the NBI ordered the unloading of the cargo stopped. As a result, the tanbark,
the boat, and three cargo trucks were seized and impounded.

On March 5, 1998, NBI-EVRO 8 Regional Director Carlos S. Caabay filed a Criminal Complaint
for the violation of Section 68 (now Section 78) of P.D. 705, The Forestry Code of the
Philippines as amended, against the captain and crew of the M/L Hadja, Robert Hernandez,
Tandico Chion, Alejandro K. Bautista, a forster, and Marcial A. Dalimot, a Community
Environment and Natural Resources Officer of the DENR. Bautista and Dalimot were also
charged with violation of Section 3(e) of R.A. No. 3019 or the Anti-Graft and Corrupt Practices
Act, along with Habi A. Alih and Khonrad V. Mohammad of the CENRO-Bongao, Tawi-tawi.
The complaint was docketed as I.S. No. 98-296 at the Prosecutor's Office of Tacloban City.
On March 10, 1998, DENR took possession of the cargo, the boat and the three trucks,
through the previous direction of the complainant. Due notice were issued to the consignee,
Robert Hernandez and the NBI Regional Director.

On March 11, 1998, Hernandez filed in the RTC of Leyte a case for replevin to recover the
items seized by the DENR and was docketed as Civil Case No. 98-03-42.

On March 16, 1998, subpoenas were issued to the respondents in I.S. No. 98-296 and on
March 17, 1998, confiscation proceedings were conducted by the PENRO-Leyte, with both
Hernandez and his counsel present.

On March 19, 1998, herein respondent Judge Frisco T. Lilagan issued a writ of replevin and
directed Sheriff IV Leonardo V. Aguilar to take possession of the items seized by the DENR
and to deliver them to Hernandez after the expiration of five days. Respondent Sheriff served
a copy of the writ to the Philippine Coast Guard station in Tacloban City at around 5:45 p.m.
of March 19, 1998.

Thus, the filing of this Administrative complaint against respondent via a letter addressed to
the Chief Justice and dated April 13, 1998, by Atty. Tabao. Complainant avers that replevin
is not available when properties sought to be recovered are involved in criminal proceedings.
He also submits that respondent judge is either grossly ignorant of the law and jurisprudence
or purposely disregarded them. Complainant states that the respondent sheriff had the duty
to safeguard M/L Hadja and to prevent it from leaving the port of Tacloban City, after he had
served a writ of seizure therefor on the Philippine Coast Guard. According to the complainant,
on March 19, 1998, the vessel left the port of Tacloban City, either through respondent
sheriff's gross negligence or his direct connivance with interested parties. Moreover,
complainant pointed out that respondent sheriff released the seized tanbark to Hernandez
within the five-day period that he was supposed to keep it under the terms of the writ,
thereby effectively altering, suppressing, concealing or destroying the integrity of said
evidence.

Respondent judge claim that the charge of gross ignorance of the law was premature since
there is a pending motion to dismiss filed by the defendants in the replevin case. Further, he
claimed that he was unaware of the existence of I.S. No. 98-296 and upon learning of the
N a t r e _ M i d t e r m S Y 2 0 1 9 - 2 0 2 0 _ 1 s t S e m e s t e r _ P a g e 22 | 68
same, he issued an order dated March 25, 1998, suspending the transfer to Hernandez of
possession of the subject items, pending resolution of an urgent manifestation by the
complainant. Respondent judges stresses that the writ of replevin was issued in strict
compliance with the requirements laid down in Rule 60 of the Revised Rule of Court. He also
pointed out that no apprehension report was issued by the NBI regarding the shipment and
neither did the DENR issue a seizure report. Respondent sheriff submits that he served the
writ of replevin on the Coast Guard to prevent the departure of subject vessel since he does
not have the means to physically prevent the vessel from sailing. He further claimed that he
verified the status of the cargo with DENR and that it came from a legitimate source except
that the shipment documents were not in order. Respondent sheriff contends that it was his
ministerial duty to serve the writ of replevin, absent any instruction to the contrary.

The Office of the Court Administrator, in a report dated April 8, 1999, recommended that the
judge be fined in the amount of P15,000.00 for gross ignorance of the law and that the
charges against respondent sheriff be dismissed for lack of merit.

ISSUE:
Whether or not the respondent judge was grossly ignorant of the law and jurisprudence for
issuing the writ of replevin.

RULING:
The complaint for replevin states that the shipment of tanbark and the vessel on which it
was loaded were seized by the NBI for verification of supporting documents. It also stated
that the NBI turned over the seized items to the DENR "for official disposition and appropriate
action". These allegations would have been sufficient to alert the respondent judge that the
DENR had custody of the seized items and that administrative proceedings may have already
been commenced concerning the shipment.

Under the doctrine of primary jurisdiction, the courts cannot take cognizance of cases
pending before administrative agencies of special competence. Also, the plaintiff in the
replevin suit who seeks to recover the shipment from the DENR had not exhausted the
administrative remedies available to him. Prudent thing for the respondent judge to do was
to dismiss the replevin outright.

Under Section 78-A of the Revised Forestry Code, the DENR secretary or his representatives
may order the confiscation of forest products illegally cut, gathered, removed, possessed or
abandoned, including the conveyances involved in the offense.

It was declared by the Court in Paat vs. Court of Appeals the that enforcement of forestry
laws, rules and regulations and the protection, development and management of forest lands
fall within the primary and special responsibilities of the DENR. The DENR should be given
free hand unperturbed by judicial intrusion to determine a controversy which is well within
its jurisdiction. The court held that the assumption of the trial court of the replevin suit
constitutes an unjustified encroachment into the domain of the administrative agency's
prerogative. The doctrine of primary jurisdiction does not warrant a court to arrogate unto
itself the authority to resolve a controversy the jurisdiction over which is initially lodged
within an administrative body of special competence.

The respondent judge's act of taking cognizance of the subject replevin suit clearly
demonstrates ignorance of the law. He has fallen short of the standard set forth in Canon 1
Rule 1.01 of the Code of Judicial Conduct, that a judge must be an embodiment of
competence, integrity and independence. To measure up to this standard, justices are
expected to keep abreast of all laws and prevailing jurisprudence. Failure to follow basic legal
commands constitutes gross ignorance of the law from which no one may be excused, not
even a judge.

Natre_Midterm SY2019-2020_1stSemester_ P a g e 23 | 68
On the charges against respondent sheriff, the Court agreed with the OCA that they should
be dismissed. Respondent sheriff merely complied with his material duty to serve the writ
with reasonable celerity and to execute it promptly in accordance with the mandates.
Respondent Judge Frisco T. Lilagan was found liable for gross ignorance of the law and is
accordingly ordered to pay a fine of 10,000. 00, with a warning that a repetition of the same
or similar offense will be dealt more severely. The complaint against respondent Sheriff IV
Leonardo V. Aguilar is dismissed for lack of merit.

FULL CASE
This is an administrative complaint filed by Atty. Leo C. Tabao, Assistant City Prosecutor of
Tacloban, in his capacity as Regional Chairman of the Region 8 Special Task Force on
Environment and Natural Resources, against (1) Judge Frisco T. Lilagan, presiding judge of
the Leyte Regional Trial Court, Branch 34, for gross ignorance of the law, gross abuse of
judicial authority, and willful disobedience to settled jurisprudence; and (2) Sheriff IV
Leonardo V. Aguilar of the Leyte RTC, Office of the Clerk of Court, for gross irregularity in
the performance of official duties, giving unwarranted benefits to a private individual,
violation of Section 1(b) and (c) of P.D. No. 1829, and conduct prejudicial to the best interest
of the service.

The records of this case reveal the following facts.


On February 24, 1998, a water craft registered under the name M/L Hadija, from Bongao,
Tawi-tawi, was docked at the port area of Tacloban City with a load of around 100 tons of
tanbark. Due to previous irregular and illegal shipments of tanbark from Bongao, agents of
the National Bureau of Investigation in Region 8 (NBI-EVRO #8) decided to verify the
shipments accompanying documents as the M/L Hadija was unloading its cargo to its
consignee, a certain Robert Hernandez.

The NBI agents found the documents irregular and incomplete, and consequently they
ordered the unloading of the cargo stopped. The tanbark, the boat M/L Hadija, and three
cargo trucks were seized and impounded.

On March 5, 1998, NBI-EVRO #8 Regional Director Carlos S. Caabay filed a criminal


complaint for violation of Section 68 (now Section 78) of P.D. No. 705, 1 the Forestry Reform
Code of the Philippines (as amended), against the captain and crew of the M/L Hadija, Robert
Hernandez, Tandico Chion, Alejandro K. Bautista, and Marcial A. Dalimot. Bautista was a
forester while Dalimot was a Community Environment and Natural Resources Officer
(CENRO) of the Department of Environment and Natural Resources (DENR) office in Tacloban
City. Bautista and Dalimot were, thus, also charged with violation of Section 3(e) of R.A. No.
3019 or the Anti-Graft and Corrupt Practices Act, 2 along with Habi A. Alih and Khonrad V.
Mohammad of the CENRO-Bongao, Tawi-tawi. The complaint was docketed as I.S. No. 98-
296 at the Prosecutors Office of Tacloban City.

In an order dated March 6, 1998, 3 complainant directed the seizure by the DENR of the M/L
Hadija, its cargo, and the three trucks pending preliminary investigation of the case. DENR
thus took possession of the aforesaid items on March 10, 1998, with notice to the consignee
Robert Hernandez and the NBI Regional Director.

On March 11, 1998, Hernandez filed in the Regional Trial Court of Leyte a case for replevin
to recover the items seized by the DENR. The case was raffled off to Branch 34 of said court
and docketed as Civil Case No. 98-03-42.

On March 16, 1998, subpoenas were issued to the respondents in I.S. No. 98-296. On March
17, 1998, confiscation proceedings were conducted by the Provincial Environment and
Natural Resources Office (PENRO)-Leyte, with both Hernandez and his counsel present.
On March 19, 1998, herein respondent Judge Frisco T. Lilagan issued a writ of replevin and
directed respondent Sheriff IV Leonardo V. Aguilar to take possession of the items seized by
the DENR and to deliver them to Hernandez after the expiration of five days. 4 Respondent
N a t r e _ M i d t e r m S Y 2 0 1 9 - 2 0 2 0 _ 1 s t S e m e s t e r _ P a g e 24 | 68
sheriff served a copy of the writ to the Philippine Coast Guard station in Tacloban City at
around 5:45 p.m. of March 19, 1998.

Thus, the filing of this administrative complaint against respondents via a letter addressed
to the Chief Justice and dated April 13, 1998, by Atty. Tabao.

Complainant avers that replevin is not available where the properties sought to be recovered
are involved in criminal proceedings for illegal logging. He points out that this is a well-
settled issue and cites several decisions 5 of this Court and the Court of Appeals on the
matter. He argues that respondent judge should have known of the existing jurisprudence
on this issue, particularly since they are subject to mandatory judicial notice per Section 1,
Rule 129 of the Revised Rules of Court.

Complainant submits that respondent judge is either grossly ignorant of the law and
jurisprudence or purposely disregarded them. But he avers that it is respondent judges duty
to keep abreast of developments in law and jurisprudence.

Complainant claims that respondent judge cannot claim ignorance of the proceedings in I.S.
No. 98-296 for the following reasons: (1) the defendants in the replevin case were all DENR
officers, which should have alerted respondent judge to the possibility that the items sought
to be recovered were being held by the defendants in their official capacities; and (2) the
complaint for replevin itself states that the items were intercepted by the NBI for verification
of supporting documents, which should have made respondent judge suspect that the same
were being held by authority of law.

As regards respondent sheriff Leonardo V. Aguilar, complainant states that it was incumbent
upon Aguilar to safeguard the M/L Hadija and prevent it from leaving the port of Tacloban
City, after he had served a writ of seizure therefor on the Philippine Coast Guard. However,
on March 19, 1998, the vessel left the port of Tacloban City, either through respondent
sheriffs gross negligence or his direct connivance with interested parties, according to
complainant. As of the time of the filing of the complaint, according to complainant, the
whereabouts of the vessel and its crew were unknown.

Moreover, complainant points out that respondent sheriff released the seized tanbark to
Hernandez on March 20 and 21, 1998, or within the five-day period that he was supposed
to keep it under the terms of the writ. Complainant argues that the tanbark formed part of
the peoples evidence in the criminal complaint against Hernandez and the others. By his act,
respondent sheriff effectively altered, suppressed, concealed, or destroyed the integrity of
said evidence. For this act, complainant contends that respondent sheriff may be held liable
under Section 1(b) of P.D. 1829, Penalizing Obstruction of Apprehension and Prosecution of
Criminal Offenders. 6 Respondent sheriffs acts also constitute gross irregularity in the
performance of his duty as a court employee.

Complainant notes that respondent sheriff was absent from his office from March 20 to March
24, 1998. This period included the dates he was supposed to have released the tanbark to
Hernandez. Complainant contends that respondent sheriff not only unlawfully released the
tanbark, he also made it appear that he was not physically present when such act was done.
In separate indorsements dated September 9, 1998, then Court Administrator Alfredo L.
Benipayo referred this administrative matter to both respondents for comment.

In his comment dated October 12, 1998, 7 respondent judge calls the attention of the Office
of the Court Administrator to a pending motion to dismiss filed by the defendants in the
replevin case that effectively prevented him from commenting on the issue. The discussions
that would have to be included in the comment, he says, would also resolve the pending
motion to dismiss. Respondent judge contends that complainant should have been prudent
enough to wait for the resolution of the motion to dismiss before filing the instant
administrative case.
N a t r e _ M i d t e r m S Y 2 0 1 9 - 2 0 2 0 _ 1 s t S e m e s t e r _ P a g e 25 | 68
Respondent judge claims that he was unaware of the existence of I.S. No. 98-296. He only
learned of the criminal case from an urgent manifestation dated March 20, 1998, filed by
complainant. He argues that he issued an order dated March 25, 1998, suspending the
transfer to Hernandez of possession of the subject items, pending resolution of the urgent
manifestation.

Respondent judge stresses that the writ of replevin was issued in strict compliance with the
requirements laid down in Rule 60 of the Revised Rules of Court. He also points out that said
writ was issued provisionally and was not intended to be the final disposition of the replevin
case.

Respondent judge avers that the charge of gross ignorance of the law is premature since he
has not made a ruling yet on the motion to dismiss filed in the replevin case. He contends
that it was too much to ask from him to take note of the fact that the defendants in said
case were officials of DENR and make assumptions based on such fact. Moreover, respondent
judge submits that while the complaint alleged that the cargo of tanbark was intercepted by
the NBI, it also alleged that the consignee thereof produced documents to prove that the
shipment was legal.

In conclusion, respondent judge points out that no apprehension report was issued by the
NBI regarding the shipment. Neither did the DENR issue a seizure report. Respondent judge
contends that the validity of the seizure of the subject items by the DENR is a matter that
will have to be resolved in relation to the motion to dismiss.

For his part, respondent sheriff submits 8 that he served the writ of replevin on the Coast
Guard precisely to prevent the departure of the subject vessel, since he does not have the
means to physically prevent said vessel from sailing. The Coast Guard commander should
have examined the vessel and its crew after being served the writ, to determine whether or
not they were engaged in any illegal activity.

Respondent sheriff narrates that no cargo was on board the vessel when he served the writ
on the Coast Guard. He verified the cargos status with DENR, which furnished him a copy of
a fax transmission stating that the tanbark came from legitimate sources except that the
shipment documents were not in order. 9 Respondent sheriff contends that it was his
ministerial duty to serve the writ of replevin, absent any instruction to the contrary. He
argues further that since the items subject of the writ are in the custody of the court and
could be disposed of only through court order, there could not be any unwarranted benefit
to a private individual as claimed by complainant.

Noting that the questioned shipment of tanbark was not covered by either an NBI
apprehension report or a DENR seizure report, respondent sheriff contends that complainant
should have taken steps to protect the integrity of the shipment instead of heaping blame
upon others for his own negligence. Respondent sheriff avers that it was not his intention to
obstruct the apprehension and prosecution of criminal offenders, contrary to complainants
claim.

Respondent sheriff refutes complainants claim that he was absent from his office from March
20 to March 24, 1998, and alleges that it was complainant who was absent from court
hearings on several occasions, in violation of his duty as a prosecutor.

Respondent submitted two supplemental comments dated October 30, 1998, 10 and May 3,
1999, 11 (1) reiterating his contention that the tanbark seized by the DENR and subject of
the replevin case had been found to come from a legitimate source, per an order signed by
the Regional Director (Region 8) of the DENR, 12 and (2) informing the OCA that the main
replevin case was dismissed per an order of respondent judge dated November 27, 1998. 13

Natre_Midterm SY2019-2020_1stSemester_ P a g e 26 | 68
As required by resolution of the Court dated January 24, 2001, the parties herein separately
manifested that they are willing to have the present case resolved based on the record on
hand.

We note that in its report dated April 8, 1999, the OCA, after reviewing the case,
recommended that respondent judge be fined in the amount of P15,000.00 for gross
ignorance of the law. At the same time, the OCA recommended that the charges against
respondent sheriff be dismissed for lack of merit.

The recommendation of the OCA is well taken, except for the amount of the fine to be
imposed on said respondent judge.

The complaint for replevin itself states that the shipment of tanbark as well as the vessel on
which it was loaded were seized by the NBI for verification of supporting documents. 14 It
also states that the NBI turned over the seized items to the DENR for official disposition and
appropriate action. 15 A copy of the document evidencing the turnover to DENR was attached
to the complaint as Annex D. 16 To our mind, these allegations would have been sufficient to
alert respondent judge that the DENR has custody of the seized items and that administrative
proceedings may have already been commenced concerning the shipment. Under the
doctrine of primary jurisdiction, courts cannot take cognizance of cases pending before
administrative agencies of special competence. 17 Note, too, that the plaintiff in the replevin
suit who seeks to recover the shipment from the DENR had not exhausted the administrative
remedies available to him. 18 The prudent thing for respondent judge to have done was to
dismiss the replevin suit outright.

Under Section 78-A of the Revised Forestry Code, the DENR secretary or his authorized
representatives may order the confiscation of forest products illegally cut, gathered,
removed, or possessed or abandoned, including the conveyances used in the commission of
the offense.

In this regard, we declared in Paat v. Court of Appeals:


the enforcement of forestry laws, rules and regulations and the protection, development and
management of forest lands fall within the primary and special responsibilities of the
Department of Environment and Natural Resources. By the very nature of its function, the
DENR should be given a free hand unperturbed by judicial intrusion to determine a
controversy which is well within its jurisdiction. The assumption by the trial court, therefore,
of the replevin suit filed by private respondents constitutes an unjustified encroachment into
the domain of the administrative agency’s prerogative. The doctrine of primary jurisdiction
does not warrant a court to arrogate unto itself the authority to resolve a controversy the
jurisdiction over which is initially lodged with an administrative body of special competence.
xxx19

Respondent judges act of taking cognizance of the subject replevin suit clearly demonstrates
ignorance of the law. He has fallen short of the standard set forth in Canon 1, Rule 1.01 of
the Code of Judicial Conduct, that a judge must be the embodiment of competence, integrity,
and independence. To measure up to this standard, judges are expected to keep abreast of
all laws and prevailing jurisprudence. 20 Judges are duty bound to have more than just a
cursory acquaintance with laws and jurisprudence. Failure to follow basic legal commands
constitutes gross ignorance of the law from which no one may be excused, not even a
judge. 21

We find, however, that respondent judge had already vacated the Writ of Seizure he issued
on March 19, 1998, in a subsequent Order dated November 27, 1998, dismissing the Civil
Complaint for replevin filed by Robert Hernandez against the Regional Director of the DENR
and other officers. He also directed in said order the sheriff to return to CENRO, Tacloban
City, all the chattels confiscated by virtue of the Writ of Seizure.22

Natre_Midterm SY2019-2020_1stSemester_ P a g e 27 | 68
Further, we find that Sheriff Aguilar in his Final Return of the Writ, dated December 15, 1998,
had already delivered to CENRO the 102 tons and 120 kilos of tanbark duly receipted by
CENRO representative Marcial A. Dalimot on the same date. 23

The OCA recommends that respondent judge be fined in the amount of P15,000.00. Under
the circumstances, considering that this is the first complaint against him, we deem a fine
of P10,000.00 to be sufficient.

Regarding the charges against respondent sheriff, we agree with the OCA that they should
be dismissed. Respondent sheriff merely complied with his ministerial duty to serve the writ
with reasonable celerity and to execute it promptly in accordance with its mandates. 24

WHEREFORE, respondent Judge Frisco T. Lilagan is hereby found liable for gross ignorance
of the law and is accordingly ordered to pay a FINE of P10,000.00, with a WARNING that a
repetition of the same or a similar offense will be dealt with more severely. The complaint
against respondent Sheriff IV Leonardo V. Aguilar is DISMISSED for lack of merit.
SO ORDERED

16. TAN VS DIR OF FORESTRY


Facts:
On April 15, 1963, petitioner-appellant Wenceslao Vinzon Tan won the bidding for the license of logging
operations on a public forest land situated in Olongapo, Zambales. Secretary of Agriculture and Natural
Resources Benjamin Gozon issued General Memorandum, Order No. 46 which states that “The Director
of Forestry is hereby authorized to grant a new ordinary timber licenses where the area covered thereby
is not more than 3,000 hectares each and that the extension of an ordinary timber license for areas not
exceeding 5,000 hectares.”

Thereafter Jose Y. Feliciano was appointed as Acting Secretary of Agriculture and Natural Resources. Upon
assumption of office on December 19,1963 he promulgated General Memorandum Order No. 46 which
took effect on the same day. Such General Memorandum is a revocation of General Memorandum No.
46 which states that “Issuance of new licenses, including amendments shall be signed by the secretary
of Agriculture and Natural Resources.” On the same day the Acting Director of Forestry signed the timber
license of the petitioner without the approval of the Secretary of Agriculture and Natural Resources and
was released by the Office of the Director of Forestry on January 6, 1964.On February 19, 1964. Ravago
Commercial wrote a letter to the Secretary of Agriculture and Natural Resources praying that the license
issued to petitioner be cancelled or revoked on the ground that the grant was irregular, anomalous and
contrary to the existing forest rules, laws and regulations. On March 9, 1964, Feliciano promulgated an
order declaring Ordinary Timber License NO. 20-64 issued to petitioner by the Director of Forestry without
authority is void ab initio. Petitioner moved for reconsideration of the order but Feliciano denied the
motion. On the basis of the denial for his motion to reconsideration, petitioner filed the instant case before
the court a quo, a petition for certiorari prohibiting and mandamus with preliminary prohibitory injunction.
Petitioner claims that respondent unlawfully, illegally and arbitrarily acted in excess of
their jurisdiction with grave abuse of discretion by revoking his license without a valid cause,
by denying him of equal protection of law, by depriving him of his constitutional right to property without
due process of law and by impairing the obligation of contract. The court a quo, from the evidence
received, declared that the petition did not state a sufficient cause of action and dismissed the same.
Petitioner, having his motion for reconsideration denied, appealed directly to the Supreme Court.

Issue:
1.Whether or not the timber license issued to the petitioner is void abinitio.
2.Whether or not petitioner has exhausted all his administrative remedies.
3.Whether or not petitioner’s action is a suit against the State.
4.Whether or not the revocation of the petitioner’s timber license by the respondent is a valid exercise of
his power as Secretary of Agriculture and Natural Resources.
5.Whether or not petitioner’s timber license can still be revoked even if it is valid.

Natre_Midterm SY2019-2020_1stSemester_ P a g e 28 | 68
Ruling:
1.Yes. The timber license issued to the petitioner is void ab initio.
Petitioner’s timber license was signed and released without the
authority of the by then Acting Director Estanislao Bernal of Forestry.

At the time it was released, the Acting Director of Forestry had no more authority to grant a
license. The authority delegated to him was contained in General Memorandum Order No. 46 which was
revoked by General Order No. 60 which removes the authority of the Director of Forestry to grant license.

2. No. Petitioner has not exhausted all his administrative remedies. Petitioner did not appeal the order of
the respondent Secretary of Agriculture and Natural Resources to the President of the Philippines who
issued Executive Proclamation No. 238 withdrawing the area from private exploitation, and establishing
it as the Olongapo Watershed Forest Reserve. Considering the President has the power to review on
appeal the orders or acts of the respondents, the failure of the petitioner to take that appeal is failure on
his part to exhaust his administrative remedies.

3. Yes. Petitioner’s action is a suit against the State.


Under the doctrine of State immunity, a suit against the State cannot prosper unless the State has given
its consent. Both the Secretary of Agriculture and Natural Resources and Director of Forestry acted in
their capacity as officers of the State and representatives of the sovereign authority discharging
governmental powers. Petitioner’s action is an attempt to circumvent the rule establishing State
exemption from suit. The promotion of public welfare and the protection of the inhabitants near the public
forest are property, rights and interest of the State. Accordingly, it may not be circumvented by directing
the action against the officer of the State instead of against the State itself.

4.Yes. The revocation of the petitioner’s timber license by the respondent is a valid exercise of his power
as Secretary of Agriculture and Natural Resources. The Director of Forestry who issued the timber license
of the petitioner is under the supervision of the Head or Secretary of the Agriculture and Natural
Resources. The power and control of the Department Heads over bureaus and offices includes the power
to modify, reverse or set aside acts of subordinate officials. Accordingly, respondent as Secretary of
Agriculture and Natural Resources has the authority to revoke, on valid grounds the timber license issued
by the Director of Forestry.

5.Yes. Petitioner’s timber license can still be revoked even if it is valid.


Paragraph 27 of the rules and regulations included in the ordinary timber license states that the terms
and conditions of the license are subject to change at the discretion of the Director of Forestry. A timber
license is not a contract within the purview of the due process of law but a privilege which can be
withdrawn whenever dictated by the public interest or welfare.

17. PEOPLE VS. QUE


Facts
• SPO1 Corpus, a member of the Provincial Task Force on Illegal Logging received an info
that a 10-wheeler truck had illegally obtained lumbers. So together with his team went on a
patrol. They followed the truck and apprehended it
• 3 persons, including Que were inside the truck. The truck contained coconut slabs qith
sawn lumbers in between them
• Corpuz asked for the necessary evidences to prove the legality of the origin of the
materials. But Que only showed a certification from CENRO (Community Environmental and
Natural Resources Office
• Que was charges in RTC with violation og Sec 68 of PD 705 as amended by EO 277
• Trial court found Que guilty and sentenced him to reclusion perpetua and also confiscated
the seized lumbers

Issue
• Whether there are no existing forest laws and regulations which required certain legal
documents for possession of timber and other forest products.
N a t r e _ M i d t e r m S Y 2 0 1 9 - 2 0 2 0 _ 1 s t S e m e s t e r _ P a g e 29 | 68
• Whether the law only penalizes possession of illegal forest products and that the possessor
cannot be held liable if he proves that the cutting, gathering, collecting or removal of such
forest products is legal

Held
• The interpretation of the appellant with “existing forest laws and regulations” refer to laws
and regulations” to refer to those laws and regulations which were already in effect at the
time of enactment of EO 277, would be strained and would render the law inutile. The phrase
should be construed to refer to laws and regulations existing at the time of possession of
timber or other forest products. Section 3 of the Administrative Order provides that the
movement of logs, lumber, non-timber forest products and wood based or wood based shall
be covered with the appropriate Certificates of Origin. The transport of lumber shall be
accompanied by CLO (Certificate of Lumber Origin). o Therefore, the accused was given
permit by DENR to transport one (1) truckload of coconut slabs only between March 7 to 11,
1994. The accused was apprehended on March 8, 1994 aboard his truck which was loaded
not only with coconut slabs but with chainsawn lumber as well. Admittedly, the lumber could
not be seen from the outside. The lumber were placed in the middle and not visible unless
the coconut slabs which were placed on the top, sides and rear of the truck were removed
• No, because there are 2 distinct and separate offenses punished under Section 68 of P.D.
705 o There are 2 distinct and separate offenses punished under Sec 68 of PD 705 § Cutting,
gathering, collecting and removing timber or other forest products from any forest land, or
timber from alienable or disposable public land, or from private land without any authority;
and § Possession of timber or other forest products without the legal documents required
under existing forest laws and regulations. o In the first offense, one can raise as a defense
the legality of the acts of cutting, gathering, collecting, or removing timber or other forest
products by presenting the authorization issued by DENR. Second, s=t is immaterial whether
the cutting, gathering, collecting and removal of the forest products is legal or not. Mere
possession of forest products without the proper documents consummates the crime.
Whether or not the lumber comes from a legal source is immaterial because E.O. 277
considers the mere possession of timber or other forest products without the proper legal
documents as malum prohibitum.

18. FACTORAN VS CA
FACTS:
On August 9, 1988 two police officers of Marikina Police Station, Sub-Station III, intercepted
a six-wheeler truck carrying 4,000 board feet of narra lumber as it was cruising along Marcos
Highway. They apprehended the truck driver, private respondent Jesus Sy, and brought the
truck and its cargo to the Personnel Investigation Committee/Special Actions and
Investigation Division (PIC/SAID) of DENR Office in Quezon City. There, petitioner Atty.
Vicente Robles of the PIC/SAID investigated them, and discovered the discrepancies in the
documentation of the narra lumber.

What were declared in the documents were narra flitches, while the cargo of the truck
consisted of narra lumber. In the documents, the plate numbers of the truck supposed to
carry the cargo bear the numbers BAX-404, PEC-492 or NSN-267, while the plate of the
truck apprehended is NVT-881. Considering that the cargo is lumber, the transport should
have been accompanied by a Certificate of Lumber Origin, scale sheet of said lumber and
not by a Certificate of Timber Origin. The Log Sale Purchase Agreement presented is between
DSM Golden Cup International as the Seller and Bonamy Enterprises as the buyer/consignee
and not with Lily Francisco Lumber Hardware.

These are in violation of Bureau of Forestry Development (BFD) Circular No. 10 which
requires possession or transportation of lumber to be supported by the following documents:

1. Certificate of Lumber Origin (CLO) which shall be issued only by the District
Forester, or in his absence, the Assistant District Forester;
N a t r e _ M i d t e r m S Y 2 0 1 9 - 2 0 2 0 _ 1 s t S e m e s t e r _ P a g e 30 | 68
2. Sales Invoice;
3. Delivery Receipt; and
4. Tally Sheets.

Such omission is punishable under Sec. 68 of Presidential Decree (P.D.) No. 705 otherwise
known as the Revised Forestry Code. Thus, petitioner Atty. Robles issued a temporary
seizure order and seizure receipt for the narra lumber and the six-wheeler truck.

On January 20, 1989, petitioner Fulgencio S. Factoran, then Secretary of Environment and
Natural Resources issued an order for the confiscation of the narra lumber and the six-
wheeler truck Private respondents neither asked for reconsideration of nor appealed the said
order to the Office of the President. Consequently, the narra lumber and six-wheeler truck
were forfeited in favor of the government and were later on advertised to be sold at a public
auction on March 20, 1989.

On March 17, 1989, private respondents filed a complaint with prayer for the issuance of the
writs of replevin and preliminary injunction and/or temporary restraining order for the
recovery of the confiscated items, and to enjoin the panned auction sale of the subject narra
lumber, respectively.

On the same day, the trial court issued an order directing the parties to desist from
proceeding with the planned auction sale and setting the hearing for the issuance of the writ
of preliminary injunction on March 27, 1989.

On March 20, 1989, private respondents filed and Ex-Parte motion for Release and Return
of Goods and Documents (Replevin) supported by an Affidavit for Issuance of Writ of Replevin
and Preliminary Injunction and a Replevin Bond in the amount of P180,000.00. The trial
court granted the writ of replevin on the same day and directed the petitioners "to deliver
the xxx [n]arra lumber, original documents and truck with plate no. NJT 881 to the custody
of the plaintiffs and/or their representatives x x x".

On March 22, 1989, the trial court issued a writ of seizure. However, petitioners refused to
comply therewith. Sheriff David G. Brodett of Branch 80 of the RTC of Quezon City, reported
that the petitioners prevented him from removing the subject properties from the DENR
compound and transferring them to the Mobile Unit compound of the Quezon City Police
Force. He then agreed to a constructive possession of the properties. On that same day,
petitioners filed a Manifestation stating their intention to file a counterbond under Rule 60 of
the Rules of Court to stay the execution of the writ of seizure and to post a cash bond in the
amount of P180,000.00. The trial court did not oblige the petitioners for they failed to serve
a copy of the Manifestation on the private respondents. Petitioners then made immediately
the required service and tendered the cash counterbond but it was refused, petitioners'
Manifestation having already been set for hearing on March 30, 1989.

On March 27, 1989, petitioners made another attempt to post a counterbond but was also
denied for the same reason. On the same day, private respondents filed a motion to declare
petitioners in contempt for disobeying the writ of seizure. The trial court gave petitioners 24
hours to answer the motion. Hearing was scheduled on March 30, 1989. On March 29, 1989,
petitioners filed with the Court of Appeals a Petition for Certiorari, Prohibition and/or
Mandamus to annul the orders of the trial court dated March 20, 1989 and March 27, 1989.
On March 30, 1989, the Court of Appeals granted petitioners temporary relief in the form of
a temporary restraining order (TRO).

On September 11, 1989, the Court of Appeals converted the TRO into a writ of preliminary
injunction upon filing by petitioners of a bond in the amount of P180,000.00. On March 30,
1990, the Court of Appeals lifted the writ of preliminary injunction and dismissed the petition.
It declared that the complaint for replevin filed by the private respondents complied with the
requirements of an affidavit and bond under Sec. 1 and 2 of Rule 60 of the Revised Rules of
N a t r e _ M i d t e r m S Y 2 0 1 9 - 2 0 2 0 _ 1 s t S e m e s t e r _ P a g e 31 | 68
Court, issuance of the writ of replevin was mandatory. As for the contempt charges against
the petitioners, the Court of Appeals believed that the same were sufficiently based on a
written charge by private respondents and the reports submitted by the Sheriff. On April 25,
1990, petitioners filed a motion for reconsideration of the foregoing decision but it was
subsequently denied by the Court of Appeals in its Resolution dated May 18, 1990.

Hence this petition.

ISSUE:
Whether or not the RTC was correct in the issuance of a writ of replevin and the Court of
Appeals in dismissing the petition and lifting the preliminary injunction.

RULING:
Pursuant to Sec. 8 of P.D. No. 705, all actions and decision of the Director are subject to
review, motu propio or upon appeal of any person aggrieved thereby, by the Department
Head whose decision shall be final and executory after the lapse of 30 days from the receipt
by the aggrieved party of said decision unless appealed to the President. The decision of the
Department Head may not be reviewed by the courts except through a special civil action
for certiorari or prohibition.

It was observed by the Court that herein respondents never appealed the confiscation order
of the petitioner Secretary to the Office of the President. The doctrine of exhaustion of
administrative remedies is basic. Courts, for reasons of law, comity and convenience, should
not entertain suits unless the available administrative remedies have first been resorted to
and proper authorities have been given an appropriate opportunity to act and correct their
alleged errors, if any, committed in the administrative forum.

It was pointed out by the Court in Paat vs. Court of Appeals that the enforcement of forestry
laws, rules and regulations and the protection, development and management of forest land
fall within the primary and special responsibilities of the DENR. It held that assumption of
the trial court of a replevin suit constitutes an encroachment into the domain of the
administrative agency's prerogative. The doctrine of preliminary jurisdiction does not warrant
a court to arrogate unto itself the authority to resolve a controversy the jurisdiction over
which is initially lodged with an administrative body of special competence.

However, herein petitioners did not a motion to dismiss on the ground of non-exhaustion of
administrative remedies. Thus, it is deemed waived. Nonetheless, the Court finds the petition
impressed with merit.

First. A writ of replevin does not issue as a matter of course upon the applicant's filing of a
bond and affidavit, as the Court of Appeals has wrongly put it. The mere filing of an affidavit,
sans allegations therein that satisfy the requirements of Section 2 Rule 60 of the Revised
Rules of Court, cannot justify the issuance of a writ of replevin. Wrongful detention of the
properties sought in an action for replevin must be satisfactory established. If only
mechanistic averment thereof is offered, the writ should not be issued. In the case at bar,
the taking of the subject property was within the administrative authority of the Secretary
as provided by Section 68-A of P.D. No. 705. Thus, it is not wrongful and does not warrant
the issuance of a writ of replevin prayed for by the private respondents.

Second. By virtue of the confiscation order by petitioner Secretary, the subject properties of
private respondents were held in custodia legis and hence, beyond the reach of replevin.
Property lawfully taken by virtue of legal process is deemed to be in custodia legis. So basic
is this doctrine that it found inclusion in the 1997 amendments introduced to the Rules of
Civil Procedure.

Natre_Midterm SY2019-2020_1stSemester_ P a g e 32 | 68
Third. Petitioner Secretary's authority to confiscate forest products under Section 68-A of
P.D. No. 705 is distinct and independent of the confiscation of forest products in a criminal
action provided for in Section 68 of P.D. No. 705.

Fourth. Section 80 of P.D. No. 705 which requires the delivery of the seized forest products
within six (6) hours from the time of the seizure to the appropriate official designated by law
to conduct preliminary investigations applies only to criminal prosecutions provided for in
Section 68 and not to administrative confiscation provided for in Section 68-A.

Fifth. Nothing in the records supports private respondents' allegation that their right to due
process was violated as no investigation was conducted prior to confiscation of their
properties.

Finally, the writ or seizure and the writ of replevin was issued by the trial court in grave
abuse of its discretion. Thus, disobedience thereto cannot constitute indirect contempt of
court which presupposes that the court order thereby violated was valid and legal. Without
a lawful order being issued, no contempt of court could be committed.

The instant petition is granted. The decision of the Court of Appeals dated March 30, 1990
and its Resolution dated May 18, 1990 were set aside. Respondent presiding judge of the
RTC of Quezon City was permanently enjoined from enforcing the Orders dated March 20,
1989 and March 22, 1989, or if said orders had already been issued, said respondent judge
was directed to render judgement of forfeiture of replevin bond filed by private respondents.
Finally, the said respondent judge is hereby permanently enjoined from further acting on the
Motion for Contempt filed by private respondents against petitioners.

19. MUSTANG LUMBER, INC. V. CA; DENR Sec. Factoran; SAID Chief Robles | Davide,
Jr., J
G.R. No. 104988 | June 18, 1996

Facts: SAID organized a team of foresters and policemen to conduct surveillance in the
lumberyard of Mustang Lumber. They saw the truck of petitioner loaded with lauan and
almaciga lumber of assorted sizes and dimensions. The team seized the truck including its
cargo and impounded them at the DENR compound in Quezon City because the driver was
not able to provide the required invoices. They were not able to enter the lumberyard
because of the owner’s refusal. The team secured a search warrant from Judge Osorio of
RTC Valenzuela. By virtue, thereof, they seized truckloads of shorts, slabs and lumber from
the lumberyard of Mustang Lumber. The other remaining stockpiles were placed under
administrative seizure because the petitioner failed to produce upon demand the required
legal documents to prove the legitimacy of their source and origin.

Sec. Factoran ordered the suspension of Mustang Lumber lumber-dealer permit. He also
ordered to confiscate in favor of the government to be disposed of in accordance with law
the seized lumber inside petitioner’s lumberyard.

Mustang Lumber filed a petition for certiorari and prohibition against DENR Sec and SAID
Chief with the RTC questioning the seizure without any search and seizure order from the
judge and the orders of the DENR Sec. for lack of prior notice and hearing (due process).

Trial Court decision (Civil Case): The warrantless search and seizure of the petitioner's
truck was valid. Search of a moving vehicle is one of the exceptions where warrantless search
and seizure is justified. The seizure of large volume of lumber was a continuation of that
made the previous day and was still pursuant to or by virtue of the search warrant issued by
Judge Osorio whose validity the petitioner did not even question. Even if the search warrant
did not specifically mention almaciga, supa, and lauan lumber and shorts, their seizure was
valid because it was contraband or prohibited articles.
N a t r e _ M i d t e r m S Y 2 0 1 9 - 2 0 2 0 _ 1 s t S e m e s t e r _ P a g e 33 | 68
Chief Robles filed with the DOJ a complaint against Ri Chuy Po, Mustang Lumber’s President
and General Manager. An Information was filed by DOJ with the RTC charging Ri Chuy Po for
violating Sec. 68 of PD No. 705

Ri Chuy Po defense in Criminal Case: lumber, as opposed to timber, is not penalized in


Sec. 68 of PD No. 705. Even if lumber is included, it cannot be used against him because it
was seized illegally. Also, the pending case in CA regarding the legality of the seizure raises
a prejudicial question.

Prosecution’s opposition: Lumber is included in Sec. 68 of PD No. 705 and possession of


such without the required documents is penalized. Also, exclusion of lumber from 68 would
defeat the purpose of the law which is to halt illegal logging

Trial Court decision (Criminal Case): Granted the motion to quash and dismissed the
case on the ground that possession of lumber without the required documents was not a
crime.
People filed a petition for certiorari with the SC contending that respondent judge acted with
grave abuse of discretion when she granted the motion to quash and dismissed the case.

CA decision: Dismissed the appeal of Mustang Lumber for lack of merit and affirmed the
trial court’s ruling. They were not able to show the required legal documents for the
possession of lumber.

Held: The word lumber does not appear in Sec. 68 of PD No. 705 however lumber is included
in the term timber. Lumber is a processed forest raw material. The Revised Forestry Code
uses the term lumber in its common usage. Even Webster Dictionary defines lumber as a
timber or logs after being prepared for the market. Simply put lumber is a processed timber.
Judge Dizon-Capulong committed grave abuse of discretion in dismissing the criminal case

The seizure the truck and its cargo was a valid exercise of the power vested upon a forest
officer or employee by Section 80 of P.D. No. 705, as amended by P.D. No. 1775. Search of
a moving vehicle is one of the five doctrinally accepted exceptions to the constitutional
mandate that no search or seizure shall be made except by virtue of a warrant issued by a
judge after personally determining the existence of probable cause. The other exceptions
are (1) search as an incident to a lawful arrest, (2) seizure of evidence in plain view, (3)
customs searches, and (4) consented warrantless search

They are presumably trifling attempts to block the serious efforts of the DENR to enforce the
decree, efforts which deserve the commendation of the public in light of the urgent need to
take firm and decisive action against despoilers of our forests whose continuous destruction
only ensures to the generations to come, if not the present, an inheritance of parched earth
incapable of sustaining life. The Government must not tire in its vigilance to protect the
environment by prosecuting without fear or favor any person who dares to violate our laws
for the utilization and protection of our forests.

Dispositive portion:

WHEREFORE, judgment is hereby rendered

1. (a) GRANTING the petition in G.R. No. 106424; (b) SETTING ASIDE and ANNULLING,
for having been rendered with grave abuse of discretion, the challenged orders of 16
August 1991 and 18 October 1991 of respondent Judge Teresita Dizon-Capulong,
Branch 172, Regional Trial Court of Valenzuela, Metro Manila, in Criminal Case No.
324-V-91, entitled People of the Philippines vs. Ri Chuy Po; (c) REINSTATING the
information in the said criminal case; and (d) DIRECTING the respondent Judge on her
successor to hear and decide the case with purposeful dispatch; and
N a t r e _ M i d t e r m S Y 2 0 1 9 - 2 0 2 0 _ 1 s t S e m e s t e r _ P a g e 34 | 68
2. DENYING the petitions in G.R. No. 104988 and in G. R. No. 123784 for utter failure of
the petitioner to show that the respondent Court of Appeals committed any reversible
error in the challenged decisions of 29 November 1991 in CA-G.R. SP No. 25510 in the
FIRST CIVIL CASE and of 31 July 1995 in CA-G.R. SP No. 33778 in the SECOND CIVIL
CASE.
Costs against the petitioner in each of these three cases.

SO ORDERED.

20. TAOPA v. PEOPLE


FACTS:
the Community Environment and Natural Resources Office of Virac, Catanduanes seized a
truck loaded with illegally-cut lumber (113 pieces of lumber of Philippine Mahogany Group
and Apitong species without any authority and/or legal documents as required under existing
forest laws and regulations, prejudicial to the public interest.) and arrested its driver, Placido
Cuison. The lumber was covered with bundles of abaca fiber to prevent detection. On
investigation, Cuison pointed to petitioner Amado Taopa and a certain Rufino Ogalesco as
the owners of the seized lumber.

Taopa, Ogalesco and Cuison were charged with violating Section 68 of Presidential Decree
(PD) No. 705 as amended, in the RTC Virac, Catanduanes. Taopa, Ogalesco and Cuison
pleaded not guilty on arraignment. After trial on the merits, RTC found them guilty as
charged beyond reasonable doubt.

Only Taopa and Cuison appealed to CA, Cuison was acquitted but Taopa's conviction was
affirmed.4

The dispositive portion of the CA decision read:


In this petition, Taopa seeks his acquittal from the charges against him alleging that the
prosecution failed to prove that he was one of the owners of the seized lumber as he was
not in the truck when the lumber was seized.

ISSUE:
WON taopa is guilty of violating Section 68 of PD No. 705, as amended?

HELD:
YES. Petition is denied. CA decision affirmed with modification. Petitioner Amado Taopa is
hereby found GUILTY beyond reasonable doubt for violation of Section 68 of PD No. 705, as
amended, and sentenced to suffer the indeterminate penalty of imprisonment from 10 years
and one day of prision mayor, as minimum, to 20 years of reclusion temporal as maximum,
with the accessory penalties provided for by law.

Both RTC and CA gave no consideration to Taopa's alibi because Cusion's testimony proved
Taopa's active participation in the transport of the seized lumber

RTC and CA found that the truck was loaded with the cargo in front of Taopa's house and
that Taopa and Ogalesco were accompanying the truck driven by Cuison up to where the
truck and lumber were seized. These facts proved Taopa's (and Ogalesco's) exercise of
dominion and control over the lumber loaded in the truck. The acts of Taopa (and of his co-
accused Ogalesco) constituted possession of timber or other forest products without the
required legal documents.

The mere fact that Taopa and Ogalesco ran away at the mere sight of the police was likewise
largely indicative of guilt. Court is convinced that Taopa and Ogalesco were owners of the
seized lumber.

Natre_Midterm SY2019-2020_1stSemester_ P a g e 35 | 68
However, Court disagree with RTC and CA as to the penalty imposed on Taopa. Section 68
of PD 705, as amended,7 refers to Articles 309 and 310 of the Revised Penal Code (RPC) for
the penalties to be imposed on violators. Violation of Section 68 of PD 705, as amended, is
punished as qualified theft.8

The law treats cutting, gathering, collecting and possessing timber or other forest products
without license as an offense as grave as an equivalent to the felony of qualified theft.

Articles 309 and 310 read: Art. 309. Penalties. - Any person guilty of theft shall be punished
by:

1. The penalty of prision mayor in its minimum and medium periods, if the value of the thing
stolen is more 12,000 pesos but does not exceed 22,000 pesos; but if the value of the thing
stolen exceeds the latter amount, the penalty shall be the maximum period of the one
prescribed in this paragraph, and one year for each additional ten thousand pesos, but the
total of the penalty which may be imposed shall not exceed twenty years. In such cases, and
in connection with the accessory penalties which may be imposed and for the purpose of the
other provisions of this Code, the penalty shall be termed prision mayor or reclusion
temporal, as the case may be. (emphasis supplied)

2. xxx Art. 310. Qualified theft. - The crime of theft shall be punished by the penalties next
higher by two degrees than those respectively specified in the next preceding articles xxx
(emphasis supplied).

The actual market value of the 113 pieces of seized lumber was P67,630.9 Following Article
310 in relation to Article 309, the imposable penalty should be reclusion temporal in its
medium and maximum periods or a period ranging from 14 years, eight months and one day
to 20 years plus an additional period of four years for the excess of P47,630.

The minimum term of the indeterminate sentence10 imposable on Taopa shall be the penalty
next lower to that prescribed in the RPC. In this case, the minimum term shall be anywhere
between 10 years and one day to 14 years and eight months or prision mayor in its maximum
period to reclusion temporal in its minimum period.

The maximum term shall be the sum of the additional four years and the medium period11
of reclusion temporal in its medium and maximum periods or 16 years, five months and 11
days to 18 years, two months and 21 days of reclusion temporal. The maximum term
therefore may be anywhere between 16 years, five months and 11 days of reclusion temporal
to 22 years, two months and 21 days of reclusion perpetua.

21. Galo vs. Monge, G. R. No. 170308


Facts:
On July 20, 1994, Monge(petitioner) and Potencio were found by the barangay tannods in
possession of and transporting 3 pieces of mahogany lumber in Iriga City. Monge and
Potencio were not able to show any documents or the requisite permit from DENR. The trial
court found Monge guilty of violation of Section 68 of PD 705, as amended by E.O. No. 277
while Potencio was discharged because he was used as a state witness.

Aggrieved, petitioner elevated the case to CA where he challenged the discharge of Potencio
as a state witness on the ground that there is no absolute necessity for his testimony. Monge
contested that it was Potencio who owned the lumbers and not him, that he was only hired
by Potencio to transport the lumbers to a sawmill. The appellate court dismissed his petition,
hence, he filed a review on certiorari.

Natre_Midterm SY2019-2020_1stSemester_ P a g e 36 | 68
Issue:
Whether or not Monge was guilty of the offense charged?

Held:
Yes.
The contention of Monge is unavailing.
Under Section 68 of PD 705,as amended by E.O. No. 277, criminalizes two distinct and
separate offences namely;
a. Cutting, gathering, collecting and removing of timber from alienable or disposable
public land, or timber from alienable or disposable public land, or from private land without
any authority; and
b. The possession of timber or other forest products without legal documents required
under the existing laws and regulations.
In the first offense, the legality of the acts of cutting, gathering, collecting or removing
timber or other forest products may be proven by the authorization duly issued by the DENR.
The second offense, however, it is immaterial whether or not the cutting, gathering,
collecting and removal of forest products are legal precisely because mere possession of
forest products without the requisite documents consummates the crime.
Petitioner cannot take refuge in his denial of ownership over the pieces of lumber found
in his possession nor his claim that he was merely hired by Potencio to provide the latter
with assistance in transporting the said lumber. PD 705 is a penal statute that punishes acts
essentially malum prohibitum. In other words, mere possession of timber or other forest
products without the proper legal documents, even absent malice or criminal intent is illegal.

22. PALLADA vs. PEOPLE G.R. No. 131270. March 17, 2000
FACTS:
DENR officers, assisted by the PNP, raided the warehouse of the Valencia Golden Harvest
Corporation. The company is engaged in rice milling and trading. They found a large stockpile
of lumber of varying sizes cut by a chain saw. As proof that the company had acquired the
lumber by purchase, petitioner produced two receipts issued by R.L. Rivero Lumberyard of
Maramag, Bukidnon, dated March 6 and 17, 1992. The DENR officers did not, however, give
credit to the receipts considering that R. L. Rivero Lumberyard's permit to operate had long
been suspended. What is more, the pieces of lumber were cut by chain saw and thus could
not have come from a licensed sawmill operator. Accordingly, all the lumber in the warehouse
had been seized and the petitioner was charged with violation of §68 of P.D. No. 705, as
amended.

RTC convicted the petitioner. The trial court did not give credence to the Certificates of
Timber Origin presented by petitioner since the lumber held by the company should be
covered by Certificates of Lumber Origin. His conviction was affirmed by the Court of Appeals.
Hence, petitioner then filed a petition for review before the Supreme Court.

ISSUES:
1. Whether separate certificates of origin should be issued for lumber and timber. 2. Whether
the presence of erasures in the certificate of timber origin render them valueless as evidence.

HELD:
Different certificates of origin are required for timber, lumber and non-timber forest
products. The issuance of a separate certificate of origin for lumber is required in order to
"pinpoint accountability and responsibility for shipment of lumber . . . and to have uniformity
in documenting the origin thereof."

Even assuming that a Certificate of Timber Origin could serve as a substitute for Certificate
of Lumber Origin, the trial court and the Court of Appeals were justified in convicting
petitioner, considering the numerous irregularities and defects found in the documents
presented by the latter. The irregularities and discrepancies make the documents in which
N a t r e _ M i d t e r m S Y 2 0 1 9 - 2 0 2 0 _ 1 s t S e m e s t e r _ P a g e 37 | 68
they are found not only questionable but invalid and, thus, justified the trial court in giving
no credence to the same. The presence of such glaring irregularities negates the presumption
that the CTOs were regularly executed by the DENR officials concerned. WHEREFORE, the
decision of the Court of Appeals is AFFIRMED with the MODIFICATION as to the penalty.

23. ALVAREZ VS PICOP


G.R. No. 162243 November 29, 2006
CHICO-NAZARIO, J.:
FACTS:
PICOP was granted Timber License Agreement (TLA) No. 43 covering an area of 75,545
hectares in Surigao del Sur, Agusan del Sur, Compostela Valley, and Davao Oriental, to
"terminate on April 25, 2002." On 23 December 1999, DENR promulgated DENR
Administrative Order (DAO) No. 99-53 mandating the conversion of the TLA into IFMA that
is primarily aimed at sustaining the raw materials for the continuous operation of the
integrated wood processing plant of the company. Pursuant to this, PICOP signified its
intention to convert its TLA No. 43 into an Integrated Forest Management Agreement (IFMA).

A Performance Evaluation Team was created to conduct performance evaluation indicating


violations by PICOP of such as the non-submission of its five-year forest protection plan and
seven-year reforestation plan as required by the DENR rules and regulations. Also there are
alleged unpaid and overdue forest charges of PICOP. It was the position of the DENR
members that PICOP’s application for the IFMA conversion should undergo the process as
provided in DAO No. 99-53. PICOP representative Atty. Caingat, however, claimed that "the
conversion of TLA No. 43 into IFMA has already been completed" and indicated that they had
"no choice except to decline participation in the ongoing meeting and bring our issues to the
proper public and legal forum."

Accordingly, the Secretary of DENR claims that further processing of PICOP’s application for
the conversion of TLA No. 43 cannot proceed until PICOP complies with the requirements.
Insisting that the conversion of its TLA No. 43 had been completed, PICOP filed a Petition for
Mandamus against then DENR Secretary Alvarez before the RTC of Quezon City. The RTC
rendered a Decision granting PICOP’s Petition for Mandamus. On appeal, CA affirmed the
Decision of the RTC. Hence, this petition by the DENR assailing that the mandamus filed by
PICOP should be out rightly dismissed on the ground that RTC has no jurisdiction over the
subject matter of the case since the acts questioned herein are related to the licensing
regulation and management of forest resources. Furthermore, RTC is prohibited to issue
TRO, writs of preliminary injunction and preliminary mandatory injunction as provided by
PD605 and RA8975.

ISSUE: WON THE TRIAL COURT HAD JURISDICTION TO TAKE COGNIZANCE OF THIS CASE
BECAUSE THE SUBJECT MATTER THEREOF PERTAINS TO THE EXCLUSIVE ADMINISTRATIVE
DOMAIN OF THE DENR SECRETARY

HELD: YES. The Petition filed before the trial court was one for mandamus with prayer for
the issuance of a writ of preliminary prohibitory and mandatory injunction with damages.
Specifically, it sought to compel the DENR Secretary to: (1) sign, execute and deliver the
IFMA documents to PICOP; (2) issue the corresponding IFMA number assignment; and (3)
approve the harvesting of timber by PICOP from the area of TLA No. 43. In its petition for
mandamus, [PICOP] asserted that "DENR Secretary Alvarez acted with grave abuse of
discretion or in excess of his jurisdiction in refusing to perform his ministerial duty to sign,
execute and deliver the IFMA contract and to issue the corresponding IFMA number to it."
What is at stake is not the scope of the DENR jurisdiction but the manner by which it exercises
or refuses to exercise that jurisdiction.

The courts have the duty and power to strike down any official act or omission tainted with
grave abuse of discretion. The 1987 Constitution is explicit in providing that judicial power
N a t r e _ M i d t e r m S Y 2 0 1 9 - 2 0 2 0 _ 1 s t S e m e s t e r _ P a g e 38 | 68
includes not only the duty of the courts of justice to settle actual controversies involving
rights which are legally demandable and enforceable, but also to determine whether or not
there has been grave abuse of discretion amounting to lack or in excess of jurisdiction on
the part of any branch or instrumentality of the government. Since PICOP alleges grave
abuse of discretion on the part of the DENR Secretary, it behooves the court to determine
the same. An outright dismissal of the case would have prevented such determination.

It is argued that PICOP’s immediate resort to the trial court was precipitate based on the
doctrine of exhaustion of administrative remedies. This holds no water. The doctrine of
exhaustion of administrative remedies is disregarded when there are circumstances
indicating the urgency of judicial intervention, which are averred to be extant in this case,
citing PICOP’s employment of a sizable number of workers and its payment of millions in
taxes to the government. Moreover, contrary to [the DENR Secretary’s] claim, the approval
of an application for IFMA conversion is not purely discretionary on the part of the DENR
Secretary since the approval of an IFMA conversion depends upon compliance with the
requirements provided under DAO No. 99-53. Even assuming, arguendo, that the approval
of an IFMA conversion involves the exercise of discretion by the DENR Secretary, the writ of
mandamus may be issued to compel the proper exercise of that discretion where it is shown
that there was grave abuse of discretion, manifest injustice, or palpable excess of authority.

DENR Sec is wrong in invoking the provisions of PD605 and RA8975. These statutes merely
proscribe the issuance of temporary restraining orders and writs of preliminary injunction
and preliminary mandatory injunction. They cannot, under pain of violating the Constitution,
deprive the courts of authority to take cognizance of the issues raised in the principal action,
as long as such action and the relief sought are within their jurisdiction.

Hence, the outright dismissal of the mandamus prayed for by DENR herein is hereby denied.

24. SOLEDAD DY v. CA, GR No. 121587, 1999-03-09


Facts:
On May 31, 1993, the Mayor of Butuan City issued Executive Order No. 93-01 creating Task
Force Kalikasan to combat "illegal logging, log smuggling or possession of and/or transport
of illegally cut or produced logs, lumber, flitches and other forest products" in that... city.
On July 1, 1993, the members of the task force received confidential information that two
truckloads of illegally cut lumber would be brought to Butuan City from the Ampayon-
Taguibe-Tiniwisan area.

Forester Resurreccion Maxilom of the DENR issued a temporary seizure order and a seizure
receipt for the two vehicles and their cargo consisting of several pieces of lumber of different
sizes and dimensions, but Lucero, the caretaker of the compound where they were seized,
refused to accept them. The seized lumber and vehicles were then taken to the City
motorpool and placed in the custody of respondent Lausa.

For lack of claimants, DENR Regional Technical Director Raoul Geollegue recommended to
the Secretary on July 29, 1993 the forfeiture of the lumber and the two vehicles.
On October 20, 1993, more than two months after the lumber had been forfeited, petitioner,
claiming to be the owner of the lumber, filed a suit for replevin in the Regional Trial Court of
Butuan City (Branch 5) for its recovery.

Before the court could act on his motion, he moved to dismiss and/or quash the writ of
replevin on the ground that the lumber in question, having been seized and forfeited by the
DENR... pursuant to P.D. No. 705, as amended (Revised Forestry Code), was under its
custody and, therefore, resort should first be made to the DENR.

Natre_Midterm SY2019-2020_1stSemester_ P a g e 39 | 68
Issues:
WITH DUE RESPECT RESPONDENT COURT OF APPEALS ERRED IN RULING THAT THE
VERIFICATION MADE BY LORENCIO DY AND NOT BY PETITIONER SOLEDAD Y. DY WAS
INSUFFICIENT TO JUSTIFY THE ISSUANCE OF THE REPLEVIN WRIT.

THE RESPONDENT COURT OF APPEALS ERRED IN RULING THAT A COUNTERBOND IN


REPLEVIN WHICH IS EFFECTIVE FOR ONLY ONE YEAR IS VALID TO CAUSE THE RETURN OF
THE PROPERTY TO DEFENDANT.

THE RESPONDENT COURT OF APPEALS ERRED IN GIVING DUE COURSE TO PRIVATE


RESPONDENT'S PETITION FOR CERTIORARI.

The appeal is without merit. The threshold question is whether the Regional Trial Court could
in fact take cognizance of the replevin suit, considering that the object was the recovery of
lumber seized and forfeited by law enforcement agents of the DENR pursuant to P.D. No.
705 (Revised Forestry Code), as amended by Executive Order No. 277.

Ruling:
The rule is that a party must exhaust all administrative remedies before he can resort to the
courts. Hence, if a remedy within the administrative machinery can still be resorted to by
giving the administrative officer concerned every opportunity to decide on a matter that
comes within his jurisdiction then such... remedy should be exhausted first before a court's
judicial power can be sought. As petitioner clearly failed to exhaust available administrative
remedies, the Court of Appeals correctly set aside the assailed orders of the trial court
granting petitioner's application for a replevin writ and denying private respondent's motion
to dismiss. Having been... forfeited pursuant to P.D. No. 705, as amended, the lumber
properly came under the custody of the DENR and all actions seeking to recover possession
thereof should be directed to that agency.

WHEREFORE, the decision of the Court of Appeals, dated January 19, 1995, and its
Resolution, dated July 26, 1995, in CA-G.R. SP 33099 are AFFIRMED with the modification
that the complaint for recovery of personal property is DISMISSED.

Principles:
Section 8 of P.D. No. 705, as amended, provides:
SEC. 8. Review. ¾ All actions and decisions of the Director are subject to review, motu propio
or upon appeal of any person aggrieved thereby, by the Department Head whose decision
shall be final and executory after the lapse of thirty (30) days from... receipt by the aggrieved
party of said decision, unless appealed to the President in accordance with Executive Order
No. 19, series of 1966. The Decision of the Department Head may not be reviewed by the
courts except through a special civil action for certiorari or... prohibition.

25. TIGOY vs. CA Case Digest


RODOLFO TIGOY vs. COURT OF APPEALSG.R. No. 144640. June 26, 2006
FACTS:
Nestor Ong, who had been engaged in the trucking business in Iligan City since 1986, was
allegedly introduced by his friend Gamad Muntod to Lolong Bertodazo who signified his intent
to rent the trucks of Ong to transport construction materials from Larapan, Lanao del Norte
to Dipolog City. A Contract to Transport was supposedly entered into between Ong and
Bertodazo. In the evening of October 3, 1993, Ong allegedly ordered Nestor Sumagang and
petitioner Rodolfo Tigoy to bring the two trucks to Lolong Bertodazo in Larapan, Lanao del
Norte. He instructed the two drivers to leave the trucks in Larapan for the loading of the
construction materials by Lolong Bertodazo. Thus, after meeting with Bertodazo, Sumagang
and petitioner Tigoy allegedly went home to return to Larapan at four o'clock in the morning
the next day. When they arrived, the trucks had been laden with bags of cement and were
half-covered with canvas. That same morning of October 4, 1993, the Ozamis City police
N a t r e _ M i d t e r m S Y 2 0 1 9 - 2 0 2 0 _ 1 s t S e m e s t e r _ P a g e 40 | 68
received a report that two trucks, a blue and green loaded with cement, did not stop at the
checkpoint. Thus, some police officers boarded their patrol vehicle to intercept the two
trucks. Upon inspection, the police officers discovered piles of sawn lumber beneath the
cement bags in both trucks. The police officers inquired if the drivers had a permit for the
lumber but the latter could not produce any. After an investigation was held by the police
and the DENR office in the city, an Information was filed against Nestor Ong, Sumagang,
Lolong Bertodazo and petitioner Tigoy for possession of forest products without legal permit
in violation of Section 68 of Presidential Decree 705, as amended by Executive Order No.
277, Series of 1987, in relation to Article 309 and 310 of the Revised Penal Code. Ong and
petitioner Tigoy entered pleas of not guilty during the arraignment. After trial, the Regional
Trial Court found both Ong and Tigoy guilty. On appeal, Ong was acquitted while Tigoy’s
conviction was upheld.

ISSUE:
Is Tigoy guilty of possession of forest products without permit?

HELD:
Yes. There are two ways of violating the said Section 68: 1) by cutting, gathering and/or
collecting timber or other forest products without a license; and, 2) by possessing timber or
other forest products without the required legal documents. Petitioner was charged with and
convicted of transporting lumber without a permit which is punishable under Section 68 of
the Code. The appellant, Sumagang and the rest of their companions were apprehended by
the police officers in flagrante delicto as they were transporting the subject lumber from
Larapan to Dipolog City. Tigoy contends that he did not know that the truck was loaded with
timber without the necessary permit. However, the circumstances show otherwise. Why
would the drivers refuse to stop when required? Did they fear inspection of their cargo? Why
would "S.O.P." (which in street parlance is grease money) be offered to facilitate the passage
of the trucks? The only logical answer to all these questions is that the drivers knew that
they were carrying contraband lumber. In offenses considered as mala prohibita or when the
doing of an act is prohibited by a special law such as in the present case, the commission of
the prohibited act is the crime itself. It is sufficient that the offender has the intent to
perpetrate the act prohibited by the special law, and that it is done knowingly and
consciously. Direct proof of previous agreement to commit an offense is not necessary to
prove conspiracy. Conspiracy may be proven by circumstantial evidence. It may be deduced
from the mode, method and manner by which the offense is perpetrated, or inferred from
the acts of the accused when such acts point to a joint purpose and design, concerted action
and community of interest. It is not even required that the participants have an agreement
for an appreciable period to commence it.

Natre_Midterm SY2019-2020_1stSemester_ P a g e 41 | 68
26. AQUINO v. PEOPLE GR No. 165448 July 27, 2009
FACTS:
The Teacherâs Camp filed with the DENR an application to cut down 14 dead Benguet pine
trees within their area in Baguio City. The trees were to be used for the repairs of Teachers
Camp. After the inspection of the trees to be cut, the Executive Director of the DENR issued
a permit allowing the cutting of 14 trees. Thereafter, a group of forest rangers received
information that pine trees were being cut without proper authority at the Teacherâs Camp.
They went to the site where they found petitioner Aquino, a forest ranger from CENRO,
another forest ranger, two supervisors, and two sawyers. The forest rangers found 23 tree
stumps, out of which only 12 were covered by the permit. An information was then filed
against the five individuals for cutting without permit the nine (9) pine trees in conspiracy.
The trial court ruled that despite the existence of a permit, the trees cut exceeded the allowed
number of the trees authorized to be cut and that the cutting of trees went beyond the period
stated in the permit. Nonetheless, all of the accused have been acquitted in the trial court
and on appeal, except for the petitioner. Petitionerâs defense was that he was merely sent
to supervise the cutting of trees at the Teacherâs Camp and he was not aware of the trees
covered by the permit. However, he still supervised the cutting of trees without procuring a
copy of the vicinity map used in the inspection of the trees to be cut. He claimed that he
could not prevent the overcutting of trees because he was just alone and that he feared one
of the sawyers, Santiago.

ISSUE: WoN petitioner is guilty beyond reasonable doubt of violation of Section 68 of PD


705

HELD: NO. Section 68 of PD 705 punishes anyone who shall cut, gather, collect or remove
timber or other forest products from any forest land, or timber from alienable or disposable
public land, or from private land, without any authority. In this case, petitioner was charged
by CENRO to supervise the implementation of the permit. He was not the one who cut,
gathered, collected or removed the pine trees within the contemplation of Section 68 of PD
705. He was not in possession of the cut trees because the lumber was used by Teachers
Camp for repairs. Petitioner could not likewise be convicted of conspiracy to commit the
offense because all his co-accused were acquitted of the charges against them. Petitioner
may have been remiss in his duties when he failed to restrain the sawyers from cutting trees
more than what was covered by the permit. As the CA ruled, petitioner could have informed
his superiors if he was really intimidated by Santiago. If at all, this could only make petitioner
administratively liable for his acts. It is not enough to convict him under Section 68 of PD
705.

Natre_Midterm SY2019-2020_1stSemester_ P a g e 42 | 68
27. FIRST DIVISION
[G.R. No. L-46772. February 13, 1992.]
THE PEOPLE OF THE PHILIPPINES, petitioner, vs. COURT OF FIRST INSTANCE OF
QUEZON (BRANCH VII), GODOFREDO ARROZAL AND LUIS FLORES, respondents.

MEDIALDEA, J p:

Doctrine:
When an accused invokes in a motion to quash the ground that the facts charged do not
constitute an offense (Rule 117, Sec. 2[a] Rules of Court), the sufficiency of the Information
hinges on the question of whether the facts alleged, if hypothetically admitted, meet the
essential elements of the offense defined in the law.
***The elements of the crime of qualified theft of logs are: 1) That the accused cut,
gathered, collected or removed timber or other forest products; 2) that the timber of other
forest products cut, gathered, collected or removed belongs to the government or to any
private individual; and 3) that the cutting, gathering, collecting or removing was without
authority under a license agreement, lease, license, or permit granted by the state.

Facts:
This petition seeks the annulment of the order of the CFI of Quezon dismissing the
information filed therein.

The private respondents were charged with the crime of qualified theft of logs, defined and
punished under Section 68 of Presidential Decree No. 705, otherwise known as the Revised
Forestry Code of the Philippines, in an information which read:

On March 23, 1977, the named accused filed a motion to quash the information on two (2)
grounds, to wit: (1) that the facts charged do not constitute an offense; and, (2) that the
information does not conform substantially to the prescribed form. The Trial court dismissed
the information on the grounds invoked and the reconsideration sought was denied.

Hence this petition.


Issue:
WoN the information charged an offense.

Held:
YES. The Court agree with the petitioner that the information substantially alleged all the
elements of the crime of qualified theft of logs as described in Section 68 of P.D. 705. While
it was admitted that the information did not precisely allege that the taking of the logs in
question was "without the consent of the state," nevertheless, said information expressly
stated that the accused "illegally cut, gather, take, steal and carry away therefrom, without
the consent of said owner and without any authority under a license agreement, lease,
license or permit, sixty (60) logs of different species. . .." Since only the state can grant the
lease, license, license agreement or permit for utilization of forest resources, including
timber, then the allegation in the information that the asportation of the logs was "without
any authority" under a license agreement, lease, license or permit, is tantamount to alleging
that the taking of the logs was without the consent of the state.

When an accused invokes in a motion to quash the ground that the facts charged do not
constitute an offense (Rule 117, Sec. 2[a] Rules of Court), the sufficiency of the Information
hinges on the question of whether the facts alleged, if hypothetically admitted, meet the
essential elements of the offense defined in the law.

The failure of the information to allege that the logs taken were owned by the state is not
fatal. The fact that only the state can grant a license agreement, license or lease does not
make the state the owner of all the logs and timber products produced in the Philippines
including those produced in private woodlands. While it is only the state which can grant a
N a t r e _ M i d t e r m S Y 2 0 1 9 - 2 0 2 0 _ 1 s t S e m e s t e r _ P a g e 43 | 68
license or authority to cut, gather, collect or remove forest products it does not follow that
all forest products belong to the state. In the just cited case, private ownership of forest
products grown in private lands is retained under the principle in civil law that ownership of
the land includes everything found on its surface.

Ownership is not an essential element of the offense as defined in Section 60 of P.D. No.
705. Thus, the failure of the information to allege the true owner of the forest products is
not material, it was sufficient that it alleged that the taking was without any authority or
license from the government.

Dispositive Portion:
ACCORDINGLY, the petition is GRANTED. The questioned order of the trial court dismissing
the information is SET ASIDE. Criminal Case No. 1591 is reinstated.

PHILIPPINE MINING ACT CASES


28. Atok Big Wedge Mining Co. v. Intermediate Appellate Court
Facts:
Private respondents are in an open, notorious and exclusive possession of the land; Petitioner
Atok alleges otherwise due to supposed mining claim

Held:
Mere recording of a mining claim, without performing annual work obligation, does not
convert the land into mineral land. The recording only operates as reservation to the
registrant exclusive rights to undertake mining activities. Thus, if no minerals are extracted
therefrom, the land is not mineral land and registration is not precluded by such recorded
claim.

The respondent appellate court, on its part, correctly considered inadequate, however, the
mere recording of petitioners mining claims in the Mining Recorder of Benguet and the
corresponding, albeit religious, payment of annual assessment fees therefor, to vest in
petitioner ownership rights over the land in question. Truly, under Executive Order No. 141,
the payment of annual assessment fees is only proof of compliance with the charges imposed
by law and does not constitute proof of actual assessment work on the mining land
concerned.

Under the Philippine Bill of 1902, the mining claim holder, upon locating and recording of his
claim, has the right to acquire for himself all mineral deposits found within his claim to the
exclusion of everyone, including the Government. Such rights are necessarily possessory as
they are essentially utilitarian and exploitative. Such rights accruing to the mining claim
locator are personal to him in the sense that no conclusion as to the nature of the land may
definitively be made based solely on the fact that a mining claim has been recorded as
regards a particular land. However, insofar as his rights are exclusive and no other person
may undertake mining activities on a recorded mining claim, unless the same has been
abandoned or the works thereon not done, the mining locators rights are also protected
against adverse mining claims of third persons. He also has the right to immediately or
eventually secure a patent on his mining claim and in the event that he postpones securing
a patent, his rights to exclusive possession and exploitation of his mining claim subsist for
as long as he complies with the continuing requirement of annually performing work or
undertaking improvements at the mine site. Insofar as the Philippine Bill of 1902 does not
provide a specific time within which the mining claim holder must secure a patent, his rights
to possession and use of the mining land appear to be unconditional, the option not at all to
secure a patent being available to him in the absence of a deadline or ultimatum therefor.
The Philippine Bill of 1902, however, did not foreclose a subsequent act on the part of the
State to limit the time within which the said patent must be secured under threat of forfeiture
of rights provided for under the Philippine Bill of 1902. Thus, in the sense that the rights of
a mining claim holder may in the future be curtailed by failure to obtain a patent, especially
N a t r e _ M i d t e r m S Y 2 0 1 9 - 2 0 2 0 _ 1 s t S e m e s t e r _ P a g e 44 | 68
if we recall that Section 36 of the said Bill itself foretold the subsequent promulgation of
regulations regarding mining claims, such rights cannot also be said to be truly unconditional
or absolute. The process of recording mining claims could not have been intended to be the
operative act of classifying lands into mineral lands. The recording of a mining claim only
operates to reserve to the registrant exclusive rights to undertake mining activities upon the
land subject of the claim. The power to classify lands into mineral lands could not have been
intended under the Philippine Bill of 1902 to be vested in just anyone who records a mining
claim. In fact, this strengthens our holding that the rights of a mining claimant are confined
to possessing the land for purposes of extracting therefrom minerals in exclusion of any or
all other persons whose claims are subsequent to the original mining locator. Thus, if no
minerals are extracted therefrom, notwithstanding the recording of the claim, the
land is not mineral land and registration thereof is not precluded by such recorded
claim. Thus, in the case at bench, the mining claimant, who had failed to comply with the
annual minimum labor requirement, could not, all the more, be expected to have extracted
minerals from the mining location. Utter lack of proof of even its potential deposits on the
part of the petitioner, thus, does not surprise us at all.

Thus, it can be said (1) that the rights under the Philippine Bill of 1902 of a mining claim
holder over his claim has been made subject by the said Bill itself to the strict requirement
that he actually performs work or undertakes improvements on the mine every year and
does not merely file his affidavit of annual assessment, which requirement was correctly
identified and declared in E.O. No. 141; and (2) that the same rights have been terminated
by P.D. No. 1214, a police power enactment, under which non-application for mining lease
amounts to waiver of all rights under the Philippine Bill of 1902 and application for mining
lease amounts to waiver of the right under said Bill to apply for patent. In the light of these
substantial conditions upon the rights of a mining claim holder under the Philippine Bill of
1902, there should remain no doubt now that such rights were not, in the first place, absolute
or in the nature of ownership, and neither were they intended to be so.

29. Republic vs. CA and De La Rosa


Republic of the Philippines, Benguet & Atok vs. Court of Appeals & De La Rosa
G.R. No. L-43938, April 15, 1988
Cruz, J.:
FACTS: These consolidated cases arose from the application for registration of a parcel of
land filed on February 11, 1965, by Jose de la Rosa on his own behalf and on behalf of his
three children, Victoria, Benjamin and Eduardo. The land, situated in Tuding, Itogon, Benguet
Province, was divided into 9 lots and covered by plan Psu-225009. According to the
application, Lots 1-5 were sold to Jose de la Rosa and Lots 6-9 to his children by Mamaya
Balbalio and Jaime Alberto, respectively, in 1964.

The application was separately opposed by Benguet Consolidated, Inc. as to Lots 1-5, Atok
Big Wedge Corporation, as to Portions of Lots 1-5 and all of Lots 6-9, and by the Republic of
the Philippines, through the Bureau of Forestry Development, as to lots 1-9. In support of
the application, both Balbalio and Alberto testified that they had acquired the subject land
by virtue of prescription; Balbalio claimed to have received Lots 1-5 from her father shortly
after the Liberation. Benguet opposed on the ground that the “June Bug” mineral claim
covering Lots 1-5 was sold to it on September 22, 1934, by the successors-in-interest of
James Kelly, who located the claim in September 1909 and recorded it on October 14, 1909.
From the date of its purchase, Benguet had been in actual, continuous and exclusive
possession of the land in concept of owner, as evidenced by its construction of adits, its
affidavits of annual assessment, its geological mappings, geological samplings and trench
side cuts, and its payment of taxes on the land.

For its part, Atok alleged that a portion of Lots 1-5 and all of Lots 6-9 were covered by the
Emma and Fredia mineral claims located by Harrison and Reynolds on December 25, 1930,
and recorded on January 2, 1931, in the office of the mining recorder of Baguio. These claims
were purchased from these locators on November 2, 1931, by Atok, which has since then
N a t r e _ M i d t e r m S Y 2 0 1 9 - 2 0 2 0 _ 1 s t S e m e s t e r _ P a g e 45 | 68
been in open, continuous and exclusive possession of the said lots as evidenced by its annual
assessment work on the claims, such as the boring of tunnels, and its payment of annual
taxes thereon.

The Bureau of Forestry Development also interposed its objection, arguing that the land
sought to be registered was covered by the Central Cordillera Forest Reserve under
Proclamation No. 217 dated February 16, 1929. Moreover, by reason of its nature, it was not
subject to alienation under the Constitutions of 1935 and 1973.

The trial court denied the application, holding that the applicants had failed to prove their
claim of possession and ownership of the land sought to be registered. The applicants
appealed to the respondent court, which reversed the trial court and recognized the claims
of the applicant, but subject to the rights of Benguet and Atok respecting their mining claims.
In other words, the Court of Appeals affirmed the surface rights of the de la Rosas over the
land while at the same time reserving the sub-surface rights of Benguet and Atok by virtue
of their mining claims. Both Benguet and Atok have appealed to this Court, invoking their
superior right of ownership.

ISSUE: WHETHER OR NOT APPLICANTS ALL SURNAMED DELA ROSA HAVE SUPERIOR
RIGHTS OF OWNERSHIP OVER THE SURFACE RIGHTS OVER THE LAND IN QUESTION WHILE
OPPOSITORS BENGUET CONSOLIDATED, INC. AND ATOK BIG WEDGE MINING COMPANY
ARE RESERVED OF THEIR SUB-SURFACE RIGHTS BY VIRTUE OF THEIR MINING CLAIM AS
DECIDED BY THE RESPONDENT COURT.

HELD: NO. Our holding is that Benguet and Atok have exclusive rights to the property in
question by virtue of their respective mining claims which they validly acquired before the
Constitution of 1935 prohibited the alienation of all lands of the public domain except
agricultural lands, subject to vested rights existing at the time of its adoption. The land was
not and could not have been transferred to the private respondents by virtue of acquisitive
prescription, nor could its use be shared simultaneously by them and the mining companies
for agricultural and mineral purposes. It is true that the subject property was considered
forest land and included in the Central Cordillera Forest Reserve, but this did not impair the
rights already vested in Benguet and Atok at that time. Such rights were not affected either
by the stricture in the Commonwealth Constitution against the alienation of all lands of the
public domain except those agricultural in nature for this was made subject to existing rights.
The perfection of the mining claim converted the property to mineral land and under the
laws then in force removed it from the public domain. By such act, the locators acquired
exclusive rights over the land, against even the government, without need of any further act
such as the purchase of the land or the obtention of a patent over it. As the land had become
the private property of the locators, they had the right to transfer the same, as they did, to
Benguet and Atok. The Court of Appeals justified this by saying there is “no conflict of
interest” between the owners of the surface rights and the owners of the sub-surface rights.
This is rather doctrine, for it is a well-known principle that the owner of piece of land
has rights not only to its surface but also to everything underneath and the airspace
above it up to a reasonable height. Under the aforesaid ruling, the land is classified as
mineral underneath and agricultural on the surface, subject to separate claims of title. This
is also difficult to understand, especially in its practical application.

The Regalian doctrine which, as its name implies, is intended for the benefit of the State,
not of private persons. The rule simply reserves to the State all minerals that may be found
in public and even private land devoted to "agricultural, industrial, commercial, residential
or (for) any purpose other than mining." Thus, if a person is the owner of agricultural land
in which minerals are discovered, his ownership of such land does not give him the right to
extract or utilize the said minerals without the permission of the State to which such minerals
belong.

Natre_Midterm SY2019-2020_1stSemester_ P a g e 46 | 68
The flaw in the reasoning of the respondent court is in supposing that the rights over the
land could be used for both mining and non-mining purposes simultaneously. The correct
interpretation is that once minerals are discovered in the land, whatever the use
to which it is being devoted at the time, such use may be discontinued by the State
to enable it to extract the minerals therein in the exercise of its sovereign
prerogative. The land is thus converted to mineral land and may not be used by any private
party, including the registered owner thereof, for any other purpose that will impede the
mining operations to be undertaken therein. The Regalian doctrine then extends not only to
land but also to “all natural wealth that may be found in the bowels of the earth.”

1. MARCOS B. COMILANG vs. HON. GENEROSO A. BUENDIA


G.R. No. L-24757 October 25, 1967
Facts:
 Nicolas Comilang staked a mining claim known as the "Bua Fraction Mineral Claim"
over a parcel of land in Tuding, Benguet, Mountain Province, with an area of 76,809
square meters, more or less. He stopped the exploration but continue to live in the
house built on a portion of the land with his wife and other relatives. In 1918, Macario
Comilang and his relatives also settled on a portion of the land with an area of about
one (1) hectare, for residential and agricultural purposes.
 Surface rights over the area embraced in the original Bua Fraction Mineral Claim of
Nicolas Comilang soon became the subject of litigation in an action to quiet title filed
in RTC of Baguio by the other heirs against Macario claiming that they bought the
rights and interest of Nicolas Comilang in the old mining claim.
 The court dismissed both claims of ownership of petitioner and respondent declaring
the area as a public land, but recognized the possession of Macario Comilang over 1½
hectares which was declared for taxation purposes but later on levied and sold at a
public auction to satisfy a money judgment obtained by spouses Jose Coloma and
Eugenia Rumbaoa against Macario filed in the RTC of Baguio. The spouses were the
purchasers in the auction and the certificate of sale was issued in favor of them.
 In the meantime, an application for lode patent covering the Bua Fraction Mineral Claim
was filed with the Bureau of Mines. Abdon Delenela and his co-heirs filed their
opposition to the application. Pending application, Delenela filed an action for the
determination of their rights on the land in the RTC of Baguio to which the said court
awarded one-half in undivided share in the mineral claim in favor of Marcos and the
other half also in undivided share in favor of Abdon Delenela and co-heirs. Delenela,
with the knowledge and conformity of Marcos Comilang, redeemed and bought from
the Coloma spouses, the latter's rights, title, interest and claim to the 1-1/2 hectares
of land acquired under the certificate of sale. A writ of possession was issued in their
favor.
 In a petition for certiorari with PI filed in the RTC of Baguio, the wife of Marcos
questioned the power of municipal court to issue said writ of possession on two grounds
(1) that conjugal property had been levied upon and sold in the execution sale, and
her share therein is affected; and (2) that there can be no severance of surface rights
over a mineral claim located under the Philippine Bill of 1902, and petitioner argued
that the sheriff could not have validly sold the surface rights in the execution sale of
June 1, 1957. The court rendered a decision in said case, holding that the writ of
possession issued by the respondent Municipal Judge was within his competence and
jurisdiction. On appeal, the decision became final.
 For a second time, a petition for certiorari and mandamus with PI was instituted by
Marcos Comilang in the RTC of Baguio City seeking the annulment of the order granting
the alias writ of possession in favor of Delenela and Perez, and again the Court of First
Instance of Baguio threw out the petition in its order dated October 22, 1964. Hence
the petition.

Issue: whether or not the final certificate of sale conveying the land described in Tax
Declaration No. 4771 to the purchasers in the execution sale is not a valid disposition of a
N a t r e _ M i d t e r m S Y 2 0 1 9 - 2 0 2 0 _ 1 s t S e m e s t e r _ P a g e 47 | 68
portion of the public domain, and especially in view of the subsequent issuance of a mineral
lode patent over the Bua Mineral Claim by the Director of Mines (Patent issued on November
7, 1966) whereby full ownership not only of the minerals therein but also of the surface
ground have been conveyed to the patentee thereof, and, therefore, the Municipal Court of
Baguio City may no longer eject them from the land.

Held: No. The 1-½ hectares portions of the Bua Fraction Mineral Claim described in Tax
Declaration No. 4771 in the name of herein appellant was levied upon and sold at public
auction to satisfy the money judgment against him in Civil Case No. 1433 of the Municipal
Court of Baguio City, and the corresponding certificate of sale was issued in favor of the
judgment creditors. Interest acquired under like certificates of sale alone has been described
as more than a lien on the property, more than an equitable estate, an inchoate legal title
to the property.

The validity of that sale was questioned when the Municipal Court ordered the eviction of
appellant from the land sold on execution, and the Supreme Court declared in L-18897 that
the sale was valid. The sale operated to divest appellant of his rights to the land which vested
in the purchasers at the auction sale. The parties herein subsequently litigated their rights
to the mineral claim in Civil Case No. 735 of the Court of First Instance of Baguio City, and
on the basis of their amicable agreement (appellant was a party in the case), the court
declared the Bua Mineral Claim co-ownership property of the parties thereto "except the
improvements existing thereon".

There is no room for doubt, therefore, that the right to possess or own the surface ground
is separate and distinct from the mineral rights over the same land. And when the application
for lode patent to the mineral claim was prosecuted in the Bureau of Mines, the said
application could not have legally included the surface ground sold to another in the
execution sale. Consequently, we have to declare that the patent procured thereunder, at
least with respect to the 1-½ hectares sold in execution pertains only to the mineral right
and does not include the surface ground of the land in question.

2. APEX MINING CO., INC. V. SOUTHEAST MINDANAO GOLD MINING CORP. (2006)
Facts:
The case involves the “Diwalwal Gold Rush Area” (Diwalwal), a rich tract of mineral land
located inside the Agusan-Davao-Surigao Forest Reserve in Davao del Norte and Davao
Oriental. Since the early 1980s, Diwalwal has been stormed by conflicts brought about by
numerous mining claims over it. On March 10, 1986, Marcopper Mining Corporation (MMC)
was granted an Exploration Permit (EP 133) by the Bureau of Mines and Geo-Sciences (BMG).
A long battle ensued between Apex and MMC with the latter seeking the cancellation of the
mining claims of Apex on the ground that such mining claims were within a forest reservation
(Agusan-Davao-Surigao Forest Reserve) and thus the acquisition on mining rights should
have been through an application for a permit to prospect with the BFD and not through
registration of a DOL with the BMG. When it reached the SC in 1991, the Court ruled against
Apex holding that the area is a forest reserve and thus it should have applied for a permit to
prospect with the BFD. On February 16 1994, MMC assigned all its rights to EP 133 to
Southeast Mindanao Gold Mining Corporation (SEM), a domestic corporation which is alleged
to be a 100%-owned subsidiary of MMC. Subsequently, BMG registered SEM’s Mineral
Production Sharing Agreement (MPSA) application and the Deed of Assignment. Several
oppositions were filed. The Panel of Arbitrators created by the DENR upheld the validity of
EP 133. During the pendency of the case, DENR AO No. 2002-18 was issued declaring an
emergency situation in the Diwalwal Gold Rush Area and ordering the stoppage of all mining
operations therein.

Natre_Midterm SY2019-2020_1stSemester_ P a g e 48 | 68
Issues:
1. W/N EP 133 and its subsequent transfer to SEM is valid.
2. W/N the DENR Secretary has authority to issue DAO 66 declaring 729 hectares of the
areas covered by the Agusan-Davao-Surigao Forest Reserve as non-forest lands and open
to small scale mining purposes.
3. Who (among petitioners Apex and Balite) has priority right over Diwalwal?

Held/Ratio:
1. INVALID. One of the terms and conditions of EP 133 is: “That this permit shall be for the
exclusive use and benefit of the permittee or his duly authorized agents and shall be used
for mineral exploration purposes only and for no other purpose.” While it may be true that
SEM is a 100% subsidiary corporation of MMC, there is no showing that the former is the
duly authorized agent of the latter. As such, the assignment is null and void as it directly
contravenes the terms and conditions of the grant of EP 133. a. The Deed of Assignment
was a total abdication of MMC’s rights over the permit. It is not a mere grant of authority to
SEM as agent. b. Reason for the stipulation. Exploration permits are strictly granted to
entities or individuals possessing the resources and capability to undertake mining
operations. Without such a condition, non-qualified entities or individuals could circumvent
the strict requirements under the law by the simple expediency of acquiring the permit from
the original permittee. c. Separate personality. The fact that SEM is a 100% subsidiary of
MMC does not automatically make it an agent of MMC. A corporation is an artificial being
invested by law with a personality separate and distinct from persons composing it as well
as from that of any other legal entity to which it may be related. Absent any clear proof to
the contrary, SEM is a separate and distinct entity from MMC. d. Doctrine of piercing the
corporate veil inapplicable. Only in cases where the corporate fiction was used as a shield
for fraud, illegality or inequity may the veil be pierced and removed. The doctrine of piercing
the corporate veil cannot therefore be used as a vehicle to commit prohibited acts. The
assignment of the permit in favor of SEM is utilized to circumvent the condition of non-
transferability of the exploration permit. To allow SEM to avail itself of this doctrine and to
approve the validity of the assignment is tantamount to sanctioning an illegal act which is
what the doctrine precisely seeks to forestall. e. PD 463 requires approval of Secretary of
DENR. Also, PD 463 (Mineral Resources Development Decree), which is the governing law
when the assignment was executed, explicitly requires that the transfer or assignment of
mining rights, including the right to explore a mining area, must be with the prior approval
of the Secretary of DENR. Such is not present in this case. f. EP 133 expired by non-renewal.
Although EP 133 was extended for 12 months until July 6, 1994, MMC never renewed its
permit prior and after its expiration. With the expiration of EP 133 on July 6, 1994, MMC lost
any right to the Diwalwal Gold Rush Area. SEM, on the other hand, has not acquired any
right to the said area because the transfer of EP 133 in its favor is invalid. Hence, both MMC
and SEM have not acquired any vested right over the area covered by EP 133.

2. NO. The DENR Secretary has no power to convert forest reserves into non-forest reserves.
Such power is vested with the President. The DENR Secretary may only recommend to the
President which forest reservations are to be withdrawn from the coverage thereof. Thus,
DAO No. 66 is null and void for having been issued in excess of the DENR Secretary’s
authority.

3. (Since it’s been held that neither MMC nor SEM has any right over Diwalwal, it is thus
necessary to make a determination of the existing right of the remaining claimants,
petitioners Apex and Balite, in the dispute.) The issue on who has priority right over Diwalwal
is deemed overtaken by the issuance of Proclamation 297 and DAO No. 2002-18, both being
constitutionally-sanctioned acts of the Executive Branch. Mining operations in the Diwalwal
Mineral Reservation are now, therefore, within the full control of the State through the
executive branch. Pursuant to Sec. 5 of RA 7942, the State can either: (1) directly undertake
the exploration, development and utilization of the area or (2) opt to award mining
operations in the mineral reservation to private entities including petitioners Apex and Balite,

Natre_Midterm SY2019-2020_1stSemester_ P a g e 49 | 68
if it wishes. The exercise of this prerogative lies with the Executive Department over which
courts will not interfere.

3. SOUTHEAST MINDANAO GOLD MINING CORPORATION v. BALITE PORTAL


MINING COOPERATIVE
G.R. No. 135190 April 3, 2002
FACTS:
The instant case involves a rich tract of mineral land situated in the Agusan-Davao-Surigao
Forest Reserve known as the “Diwalwal Gold Rush Area.” Located at Mt. Diwata in the
municipalities of Monkayo and Cateel in Davao Del Norte, the land has been embroiled in
controversy since the mid-80’s due to the scramble over gold deposits found within its
bowels.

On March 10, 1988, Marcopper Mining Corporation (Marcopper) was granted Exploration
Permit No. 133 (EP No. 133) over 4,491 hectares of land, which included the hotly-contested
Diwalwal area.

Not long thereafter, Congress enacted on June 27, 1991 Republic Act No. 7076, or the
People’s Small-Scale Mining Act. The law established a People’s Small-Scale Mining Program
to be implemented by the Secretary of the DENR and created the Provincial Mining
Regulatory Board (PMRB) under the DENR Secretary’s direct supervision and control. The
statute also authorized the PMRB to declare and set aside small-scale mining areas subject
to review by the DENR Secretary and award mining contracts to small-scale miners under
certain conditions.

DENR Secretary Fulgencio S. Factoran issued Department Administrative Order (DAO) No.
66, declaring 729 hectares of the Diwalwal area as non-forest land open to small-scale
mining. The issuance was made pursuant to the powers vested in the DENR Secretary by
Proclamation No. 369, which established the Agusan-Davao-Surigao Forest Reserve.
On April 1, 1997, Provincial Mining Regulatory Board of Davao passed Resolution No. 26,
Series of 1997, authorizing the issuance of ore transport permits (OTPs) to small-scale
miners operating in the Diwalwal mines.

Thus, on May 30, 1997, petitioner filed a complaint for damages before the Regional Trial
Court of Makati City, Branch 61, against the DENR Secretary and PMRB-Davao. SEM alleged
that the illegal issuance of the OTPs allowed the extraction and hauling of P60,000.00 worth
of gold ore per truckload from SEM's mining claim.

Petitioner then filed a special civil action for certiorari, prohibition and mandamus before the
Court of Appeals against PMRB-Davao, the DENR Secretary and Balite Communal Portal
Mining Cooperative (BCPMC), which represented all the OTP grantees. It prayed for the
nullification of the above-quoted Memorandum Order No. 97-03 on the ground that the
“direct state utilization” espoused therein would effectively impair its vested rights under EP
No. 133.

The Court of Appeals dismissed the petition. It ruled that the DENR Secretary did not abuse
his discretion in issuing Memorandum Order No. 97-03 since the same was merely a directive
to conduct studies on the various options available to the government for solving the
Diwalwal conflict.

ISSUE: Whether or not the Court of Appeals erred when it concluded that the assailed
memorandum order did not adopt the “direct state utilization scheme” in resolving the
Diwalwal dispute.

Natre_Midterm SY2019-2020_1stSemester_ P a g e 50 | 68
RULING:
No. The challenged MO 97-03 did not conclusively adopt "direct state utilization" as a policy
in resolving the Diwalwal dispute. The terms of the memorandum clearly indicate that what
was directed thereunder was merely a study of this option and nothing else. Contrary to
petitioner's contention, it did not grant any management/operating or profit-sharing
agreement to small-scale miners or to any party, for that matter, but simply instructed the
DENR officials concerned to undertake studies to determine its feasibility.

Further, there is no merit in petitioner's assertion that MO 97-03 sanctions violation of mining
laws by allowing illegal miners to enter into mining agreements with the State, and it must
likewise be pointed out that under no circumstances may petitioner's rights under EP No.
133 be regarded as total and absolute.

Petitioner's first assigned error is baseless and premised on tentative assumptions. Petitioner
cannot claim any absolute right to the Diwalwal mines pending resolution of the Consolidated
Mines cases, much less ask us to assume, at this point, that respondent BCMC and the other
mining firms are illegal miners. These factual issues are to be properly threshed out in CA
G.R. SP Nos. 61215 and 61216, which have yet to be decided by the Court of Appeals. Any
objection raised against MO 97-03 is likewise premature at this point, inasmuch as it merely
ordered a study of an option which the State is authorized by law to undertake.

There is no need to rule on the matter of the OTPs, considering that the grounds invoked by
petitioner for invalidating the same are inextricably linked to the issues raised in the
Consolidated Mines cases.

4. MINERS ASSOCIATION OF THE PHILIPPINES, INC., petitioner, vs.


HON. FULGENCIO S. FACTORAN, JR., Secretary of Environment and Natural
Resources, and JOEL D. MUYCO, Director of Mines and Geosciences
Bureau, respondents
G.R. No. 98332 January 16, 1995 ROMERO, J.:
Facts:
Former President Cory Aquino, exercising legislative power, issued EO211 prescribing the
interim procedures in the processing and approval of exploration, development and
utilization of minerals. To implement the act, Secretary of DENR promulgated AO No. 57 and
82. On 25 July 1987 the then President Aquino issued EO279 authorizing the DENR
SECRETARY to negotiate and conclude joint venture, co-production, production sharing,
development and those agreements involving technical or financial assistance by foreign
owned corporations for large scale EDU.

Pursuant to Sec 6 of EO 279, the DENR issued AO No.57 which provides that all existing
mining leases or agreements which were granted AFTER the affectivity of the 1987
Constitution, except small scale mining leases and those pertaining to sand and gravel and
quarry resources covering an area of 20 hectares or less, shall be converted into production
sharing agreement within one year from the effectivity of the guidelines.

On 20 November 1990, Sec of DENR issued EO No. 82 laying down the Procedural Guidelines
on the award of mineral Production sharing agreement. This order provides the person or
entities required to submit a LETTER OF INTENT and MINERAL PRODUCTION SHARING
AGREEMENT within 2 years from the effectivity of AO No.57 or until 17 July 1991. Failure to
do so within the prescribed period shall cause the abandonment of mining, quarry, gravel
and sand.

Natre_Midterm SY2019-2020_1stSemester_ P a g e 51 | 68
The Miners Association Inc assailed the validity of the above-mentioned issuances and
alleged the following:
1. Sec of DENR issued both AO57 and 82 in excess of their rule-making power under Section
6 of EO279.
2. The orders violate the non-impairment of contracts provision under the bill of rights on
the ground the AO57 unduly pre-terminates existing mining leases and other mining
agreements and automatically converts them into production-sharing agreements within one
year from its effectivity date.
3. AO No.82 declares that failure to submit the LETTER OF INTENT and Mineral Production
Sharing Agreement within 2 years from the date of effectivity of said guidelines shall cause
an abandonment of their mining, quarry, sand and gravel permits.
On 13 November 1991, Continental Marble Corp sought to intervene in the cases alleging
that the TRO issued by the Court, the DENR Regional Office in San Fernando Pampanga
refused to renew its Mines Temporary Permit and claimed further that its rights and interest
are prejudicially affected by AO No.57 and 82.

Issue:
1. Whether AO57 and 82 have the effect of abrogating the existing mining laws and unduly
preterminate the existing mining leases and agreements.

Held: No.
PD No. 463, as amended, pertains to the old system of EDU of natural resources through
license, concession or lease which has been disallowed by Article XII, Sec 2 of the 1987
Philippine Constitution. By virtue of this constitutional mandate and its implementing laws,
the provisions dealing with license, concession or leases ceased to operate as the governing
laws. In other words, in all areas of administration and management of mineral lands, the
provision of PD463, as amended and other existing laws still govern.

Upon the effectivity of the 1987 Consti on 2 February 1987, the State assumed a more
dynamic role in EDU. Article XII, Section 2 explicitly ordains that EDU shall under the full
control and supervision of the State. Given these considerations, there is no clear showing
that the DENR Sec has transcended the bounds demarcated in the EO279 for the exercise of
his rule-making power.

Article XII, Sec 2 of the 1987 Consti does not apply retroactively to license, concession or
lease granted by the government under the 1973 Consti or before the effectivity of the
present Constitution. The intent to apply prospectively was stressed during the deliberations
in the Constitutional Commission.

AO No. 57 applies only to all existing mining leases or agreements which were granted after
the effectivity of the 1987 Consti pursuant to EO No. 211. It bears to mention that under the
text of EO211, there is a reservation clause which provides that the privileges as well as the
terms and conditions of all existing mining leases or agreements granted after the effectivity
of the present constitution shall be subject to any and all modification or alterations which
the Congress may adopt. Hence, the strictures of the non-impairment of contract clause do
not apply to the aforesaid mining leased or agreements after the effectivity of the 1987
Consti.

The State in the exercise of police power may not be precluded by the constitutional
restriction on non-impairment of contracts. Police power being co-extensive with the
necessities of the case and the demands of public interest.

Natre_Midterm SY2019-2020_1stSemester_ P a g e 52 | 68
5. REPUBLIC V. ROSEMOOR MINING
FACTS:
Four respondents were granted permission to look for marble deposits in the mountains of
Biak-na-Bato. When they discovered marble deposits in Mount Mabio, they applied for license
to exploit said marble deposits and they were issued such license. However, in a letter,
Ernest o Maceda (newly-appointed Minister of the Department of Energy and Natural
Resources) cancelled their license. Proclamation No. 84 was then issued, confirming the
cancellation of the license.

RULING OF THE TRIAL COURT:


The privilege granted under respondents’ license already became a property right, which is
protected under the due process clause. License cancellation, without notice and hearing was
unjust. Moreover, the proclamation, which confirmed the cancellation of the license was an
ex post facto law.

RULING OF THE COURT OF APPEALS:


CA affirmed the decision of the Trial Court.
It adds that the Constitution provides for the non-impairment of obligations and contracts,
which implies that the license of the respondents must be respected.

ISSUES:
1. WON the license was valid.
Petitioners: License was issued in violation of PD 463 – a quarry license should cover
not more than 100 hectares in any given province. The license was issued to Rosemoor
Mining and Development Corporation and covered a 330-hectare land.
Respondents: The license was embraced by 4 separate applications, each for an aread
of 81 hectares. Also, the issue has been mooted because PD 463 has already been
repealed by RA 7942 or the Philippine Mining Act of 1995.
2. WON Proclamation No. 84 – which confirmed the cancellation of the license, is valid.

Petitioners: The license was validly declared a nullity and terminated. Maceda’s letter did
not cancel or revoke the license, but merely declared its nullity. Also, the respondents waived
their right to notice and hearing in their license application.

Respondents: Their right to due process was violated because there was no notice and
hearing. Proclamation No. 84 is not valid because it violates the clause on non-impairment
of contracts, it is an ex post facto law and/or a bill of attainder, and it was issued by the
President after the effectivity of the 1987 Constitution.

RULING OF THE SUPREME COURT:


The Petition is GRANTED.

SC set aside the decision of the CA and TC.


1. THE LICENSE IS NOT VALID.
The issue has not been mooted because while RA 7942 has expressly repealed provisions of
mining laws that are inconsistent with its own, it respects previously issued valid and existing
licenses.

When the license was issued, the governing law was PD 463. Thus, it was subject to the
terms and conditions of PD 463, including the part where it says that the quarry license shall
cover an area of not more than 100 hectares in any one province and not more than 1000
hectares in the entire Philippines. The license in question was issued in the name of Rosemoor
Mining Development Corporation and not the 4 individual stockholders. It clearly violates PD
463 because the license covered an area of 330-hectares.

2. PROCLAMATION NO. 84, CONFIRMING THE CANCELLATION OF THE LICENSE, IS


VALID.
N a t r e _ M i d t e r m S Y 2 0 1 9 - 2 0 2 0 _ 1 s t S e m e s t e r _ P a g e 53 | 68
Respondents’ license may be revoked or rescinded by executive action when the national
interest so requires because it is not a contract, property or a property right protected by
the due process clause of the Constitution. The license itself provides such condition. The
license can also be validly revoked by the State in the exercise of police power, in accordance
with the Regalian doctrine.

Also, since the license is not a contract, the non-impairment clause may not be invoked.
Even if it were, the non-impairment clause must yield to the police power of the State.
The proclamation cannot also be said to be a bill-of-attainder, which is a legislative act which
inflicts punishment without judicial trial. The proclamation only declares the nullity of the
license. It does not declare guilt or impose punishment.

The proclamation can also be said to be an ex post facto law because it does not fall under
any of the six recognized instances when a law is considered as such. It is not even criminal
or penal in nature.

Lastly, when President Aquino issued Proclamation No. 84, she was still validly exercising
legislative powers under the Provisional Constitution of 1986.

6. YINLU BICOL MINING CORPORATION, Petitioner, v. TRANS-ASIA OIL AND


ENERGY DEVELOPMENT CORPORATION, Respondent.
[G.R. No. 207942, January 12, 2015, BERSAMIN, J.:]
TOPIC: PERSONS – EFFECT AND APPLICATION OF LAWS – REPEAL OF LAWS
DOCTRINE: Rights pertaining to mining patents issued pursuant to the Philippine Bill of
1902 and existing prior to November 15, 1935 are vested rights that cannot be impaired.

FACTS:
1. This case involves 13 mining claims over the area located in Barrio Larap, Municipality
of Jose Panganiban, Camarines Norte, a portion of which was owned and mined by
Philippine Iron Mines, Inc. (PIMI), which ceased operations in 1975 due to financial
losses.
a. PIMI’s portion (known as the PIMI Larap Mines) was sold in a foreclosure sale to
the Manila Banking Corporation (MBC) and Philippine Commercial and Industrial
Bank (PCIB, later Banco De Oro, or BDO).

2. The Government then opened the area for exploration.


a. Trans-Asia Oil and Energy Development Corporation (Trans-Asia) then explored
the area from 1986 onwards.
i. In 1996, it entered into an operating agreement with Philex Mining
Corporation over the area, their agreement being duly registered by the
Mining Recorder Section of Regional Office No. V of the Department of
Environment and Natural Resources (DENR).

3. 1997: Trans-Asia filed an application for the approval of Mineral Production Sharing
Agreement (MPSA) over the area in that Regional Office of the DENR, through the
Mines and Geosciences Bureau (MGB), in Daraga, Albay.
a. The application, which was amended in 1999, was granted on July 28, 2007
under MPSA No. 252-2007-V, by which Trans-Asia was given the exclusive
right to explore, develop and utilize the mineral deposits in the portion
of the mineral lands.

4. August 31 2007: Yinlu Bicol Mining Corporation (Yinlu) informed the DENR by
letter that it had acquired the mining patents of PIMI from MBC/BDO by way
of a deed of absolute sale, stating that the areas covered by its mining patents
were within the areas of Trans-Asia’s MPSA

Natre_Midterm SY2019-2020_1stSemester_ P a g e 54 | 68
5. September 14, 2007: Trans-Asia informed Yinlu by letter that it would commence
exploration works in Yinlu’s areas pursuant to the MPSA, and requested Yinlu to allow
its personnel to access the areas for the works to be undertaken.
a. Yinlu replied that Trans-Asia could proceed with its exploration works on its own
private property in the Calambayungan area, not in the areas covered by its
(Yinlu) mining patents.

6. TransAsia found out that the registration of its MPSA had been put on hold
because of Yinlu’s request to register the deed of absolute sale in its favor.

7. DENR Secretary directed MGB Regional Office V to verify the validity of the mining
patents of Yinlu.
a. MGB Regional Office V informed the Office of the DENR Secretary that there was
no record on file showing the existence of the mining patents of Yinlu.
Accordingly, the parties were required to submit their respective position papers

8. DENR Sec Atienza ordered the amendment of Trans-Asia’s MPSA by excluding


therefrom the mineral lands covered by Yinlu’s mining patents
a. DENR Sec. Jose L. Atienza, Jr in his order found that the mining patents had
been issued to PIMI in 1930 as evidenced by and indicated in PIMI’s certificates
of title submitted by Yinlu; and that the patents were validly transferred to and
were now owned by Yinlu.
b. He rejected Trans-Asia’s argument that Yinlu’s patents had no effect and were
deemed abandoned because Yinlu had failed to register them pursuant to Section
101 of Presidential Decree No. 463, as amended.
c. He refuted Trans-Asia’s contention that there was a continuing
requirement under the Philippine Bill of 1902 for the mining patent
holder to undertake improvements in order to have the patents subsist,
and that Yinlu failed to perform its obligation to register and to
undertake the improvement, observing that the requirement was not an
absolute imposition.
i. He noted that the suspension of PIMI’s operation in 1974 due to financial
losses and the foreclosure of its mortgaged properties by the creditor
banks (MBC/PCIB) constituted force majeure that justified PIMI’s failure in
1974 to comply with the registration requirement under P.D. No. 463;
ii. that the Philippine Bill of 1902, which was the basis for issuing the patents,
allowed the private ownership of minerals, rendering the minerals covered
by the patents to be segregated from the public domain and be considered
private property; and
iii. that the Regalian doctrine, under which the State owned all natural
resources, was adopted only by the 1935, 1973 and 1987 Constitutions

9. Office of the President affirmed the DENR Sec’s Order.


a. Under the Philippine Constitution, there is an absolute prohibition against
alienation of natural resources. Mining locations may only be subject to
concession or lease.
i. The only exception is where a location of a mining claim was perfected
prior to November 15, 1935, when the government under the 1935
Constitution was inaugurated, and according to the laws existing at that
time a valid location of a mining claim segregated the area from the public
domain, and the locator is entitled to a grant of the beneficial ownership
of the claim and the right to a patent therefore.
b. The right of the locator to the mining patent is a vested right, and the
Constitution recognizes such right as an exception to the prohibition
against alienation of natural resources.
i. The right of the appellee as the beneficial owner of the subject mining
patents in this case, therefore, is superior to the claims of appellant
N a t r e _ M i d t e r m S Y 2 0 1 9 - 2 0 2 0 _ 1 s t S e m e s t e r _ P a g e 55 | 68
ii. The existence of the TCT’s in the name of appellee further bolsters the
existence of the mining patents. Under PD 1529, also known as the
Property Registration Decree, once a title is cleared of all claims or where
none exists, the ownership over the real property covered by the Torrens
title becomes conclusive and indefeasible even as against the government.

10. CA: It agreed with the DENR Secretary and the OP that Yinlu held mining patents
over the disputed mining areas, but ruled that Yinlu was required to register the
patents under PD No. 463 in order for the patents to be recognized in its favor.
a. It found that Yinlu and its predecessors-in-interest did not register the patents
pursuant to PD No. 463; hence, the patents lapsed and had no more effect

11. Yinlu asserts the following:


a. The mining patents of Yinlu were registered pursuant to Act No. 496 (Land
Registration Act of 1902) in relation to the Philippine Bill of 1902 (Act of Congress
of July 1 , 1902), the governing law on the registration of mineral patents, were
valid, existing and indefeasible.
i. Section 21 of the Philippine Bill of 1902: allowed citizens of the United
States and of the Philippine Islands to explore, occupy and purchase
mineral lands
ii. Section 27 of the Philippine Bill of 1902: after the exploration and
claim of the mineral land, the owner of the claim and of the mineral patents
was entitled to all the minerals found in the area subject of the claim
iii. its registered mineral patents, being valid and existing, could not be
defeated by adverse, open and notorious possession and prescription;
b. substantive rights over mineral claims perfected under the Philippine
Bill of 1902 subsisted despite the changes of the Philippine Constitution
and of the mining laws
i. Constitution could not impair vested rights;
ii. Section 100 and Section 101 of PD No. 463 would impair its vested
rights under its mineral patents if said provisions were applied to
it;
iii. Section 99 of PD No. 463 expressly prohibited the application of
Section 100 and Section 101 to vested rights.

ISSUE: Whether Yinlu’s mining patents constitute vested rights and could not be
disregarded.

HELD: YES
1. A mining patent pertains to a title granted by the government for the said mining
claim.

2. Under the 1935 Constitution, which took effect on November 15 1935, the alienation
of natural resources, with the exception of public agricultural land, was expressly
prohibited.
a. The natural resources being referred therein included mineral lands of public
domain, but not mineral lands that at the time the 1935 Constitution took effect
no longer formed part of the public domain.
b. Prohibition against the alienation of natural resources did not apply to
a mining claim or patent existing prior to November 15, 1935.
c. McDaniel v. Apacible: A mining claim perfected under the law is property in the
highest sense, which may be sold and conveyed and will pass by descent. It has
the effect of a grant (patent) by the United States of the right of present and
exclusive possession of the lands located.
d. The owner of a perfected valid appropriation of public mineral lands is entitled
to the exclusive possession and enjoyment against everyone, including the
Government itself. Where there is a valid and perfected location of a mining
N a t r e _ M i d t e r m S Y 2 0 1 9 - 2 0 2 0 _ 1 s t S e m e s t e r _ P a g e 56 | 68
claim, the area becomes segregated from the public domain and the property of
the locator.
e. A valid and subsisting location of mineral land, made and kept up in accordance
with the provisions of the statutes of the United States, has the effect of a grant
by the United States of the present and exclusive possession of the lands located,
and this exclusive right of possession and enjoyment continues during the entire
life of the location. There is no provision for, nor suggestion of, a prior
termination thereof.

3. Even without a patent, the possessory right of a qualified locator after discovery of
minerals upon the claim is a property right in the fullest sense, unaffected by the fact
that the paramount title to the land is in the Government, and it is capable of transfer
by conveyance, inheritance, or devise.
a. the mining claim under consideration no longer formed part of the public
domain when the provisions of Article XII of the Constitution became
effective, it does not come within the prohibition against the alienation
of natural resources; and the petitioner has the right to a patent therefor
upon compliance with the terms and conditions prescribed by law.

4. Although Section 100 and Section 101 of PD No. 463 require registration and annual
work obligations, Section 99 of PD No. 463 nevertheless expressly provides that the
provisions of PD No. 463 shall not apply if their application will impair vested rights
under other mining laws

Section 99. Non-impairment of Vested or Acquired Substantive Rights. Changes made and
new provisions and rules laid down by this Decree which may prejudice or impair vested or
acquired rights in accordance with order mining laws previously in force shall have no
retroactive effect. Provided, That the provisions of this Decree which are procedural in nature
shall prevail.
a. A right is vested when the right to enjoyment has become the property of
some particular person or persons as a present interest.
i. It is “the privilege to enjoy property legally vested, to enforce contracts,
and enjoy the rights of property conferred by existing law” or “some right
or interest in property which has become fixed and established and is no
longer open to doubt or controversy”
b. The due process clause prohibits the annihilation of vested rights. ‘A state may not
impair vested rights by legislative enactment, by the enactment or by the
subsequent repeal of a municipal ordinance, or by a change in the constitution of
the State, except in a legitimate exercise of the police power’
c. It has been observed that, generally, the term “vested right” expresses the concept
of present fixed interest, which in right reason and natural justice should be
protected against arbitrary State action, or an innately just an imperative right
which an enlightened free society, sensitive to inherent and irrefragable individual
rights, cannot deny
d. Republic v. Court of Appeals: that mining rights acquired under the Philippine Bill
of 1902 and prior to the effectivity of the 1935 Constitution were vested rights that
could not be impaired even by the Government.
e. In the present case: the mining patents of Yinlu were issued pursuant to the
Philippine Bill of 1902 and were subsisting prior to the effectivity of the
1935 Constitution. Consequently, Yinlu and its predecessors-in-interest had
acquired vested rights in the disputed mineral lands that could not and should not
be impaired even in light of their past failure to comply with the requirement of
registration and annual work obligations.

Natre_Midterm SY2019-2020_1stSemester_ P a g e 57 | 68
G.R. No. L-49143 August 21, 1989
7. ZAMBALES CHROMITE MINING COMPANY, INC., petitioner,
vs.
HON. MINISTER OF NATURAL RESOURCES JOSE J. LEIDO JR. and DIRECTOR OF
MINES JUANITO C. FERNANDEZ, respondents. PHILEX MINING CORPORATION and
REGALIAN MINING EXPLORATION CORPORATION, intervenors.

This is a petition for certiorari and prohibition with preliminary injunction seeking to enjoin
the Minister (now Secretary) of Natural Resources and the Director of Mines from enforcing
Presidential Decree No. 1214 dated October 14,1977 requiring all locators under the Act of
Congress of July 1, 1902, as amended, to apply for mininglease contracts under the
provision of Presidential Decree No. 463 better known as the Mineral Development
Resources Decree of 1974 and to declare Presidential Decree No. 1214 unconstitutional since
its enforcement would deprive petitioners of its property without due process and without
just compensation.

Petitioner Zambales Chromite Mining Company, Inc. is a mining corporation duly organized
and existing under and by virtue of the laws of the Philippines.

Petitioner claims that it is the owner and holder of sixty (60) mineral claims which it acquired
through purchase in good faith and for value 43 years ago. Said claims situated at the
Municipality of Sta. Cruz, Zambales, were located and registered in 1934 under the Act of
U.S. Congress of July 1, 1902 (known as the Philippine Bill of 1902). (Petition, p. 2; Rollo,
p. 3); that from 1934 to 1977 it has to its credit a total investment of over Pl,222,640.00 for
the mining exploration, development and operation of its said sixty mining claims (Petition,
p. 3; Rollo, p. 4); that on June 14, 1977 it actually and duly flied its application for patent for
each claim of said sixty (60) mineral claims (Petition, p. 4; Rollo, p. 5); that respondent
Director of Mines issued an order dated July 13,1977 approving the application of petitioner
for availment of rights on said claims under Presidential Decree No. 463 (Petition, p. 5; Rollo,
p. 6); that the aforesaid sixty (60) lode mineral claims are already private property of
petitioner, following the doctrinal rule laid down in McDaniel v. Apacible and Cuisia (42 Phil.
749; 753-754) and Gold Creek Mining Corporation v. Rodriguez, et al. (66 Phil. 259) which
had already been segregated from the public domain to which petitioner is entitled to the
exclusive possession and enjoyment against everyone; that the issuance of Presidential
Decree No. 1214 on October 14, 1977 which declared open to lease subsisting and valid
patentable mining claims, lode or placer, located under the provisions of the Act of U.S.
Congress of July 1, 1902, as amended, already segregated from the public domain and
owned and held by it for over 43 years and requiring it without fail and against their will to
file a mining lease application with the Mines Regional Office concerned within a period of
one year from October 14, 1977 is a deprivation of petitioner's rights to the ownership of
said claims without due process of law nor or just compensation and therefore,
unconstitutional.

The Court in its resolution dated November 3,1978, gave due course to the petition and
required respondents to comment (Rollo, p. 33). The Solicitor General as counsel for public
respondent, flied his comment on March 26,1979 (Rollo, pp. 58-71-A).

On May 10, 1979, petitioner filed a reply (Rollo, p. 83) to the comment in compliance with
the resolution of April 10, 1979. But on May 9, 1979, Baguio Gold Mining Company, Philex
Mining and Regalian Mining Corporation filed with the Court two separate motions for leave
to intervene (Rollo, p. 120).

On February 10, 1981, Baguio Gold Mining Company, Philex Mining Company and Regalian
Mining Corporation filed with the court a Joint Petition for Intervention (Rollo, p. 171) raising
the same issues brought up by petitioner Zambales Chromite Mining Company regarding the
constitutionality of P.D. No. 1214 based on the doctrinal mandates of the ruling cases
Natre_Midterm SY2019-2020_1stSemester_ P a g e 58 | 68
of McDaniel v. Apacible, 42 Phil. 749 [1922] and Gold Creek Mining Corporation v. Rodriguez,
66 Phil. 259 (1939); Salazar Mining Co. v. Rodriguez, et al., 67 Phil. 97, insofar as it
invests inter alia, private ownership in patentable mining claims to have survived to date
due to a faithful compliance with the various requirements of applicable mining laws to
include the land surface of said mining claims. Petitionees memorandum was adopted by
intervenors as to the factual and legal showing of the unconstitutionality of Presidential
Decree No. 1214 (Rollo, pp. 455-456).

The Solicitor General as counsel for public respondent submitted his memorandum on
February 12,1982 (Rollo, pp. 468499) while petitioner filed its reply to said memorandum
on April 3, 1982 (Rollo, pp. 505-560).

Counsel for petitioner on August 20, 1982 filed a motion to refer this case to the Court En
Bane for action and decision (Rollo, p. 536) and on September 8,1982, the Court resolved
to issue a temporary restraining order, effective as of said date and continuing until otherwise
ordered by the Court (Rollo, p. 562).

On February 11, 1988 the Court acting on the motion for intervention filed by counsel for
intervenor Francisco N. Calinisan dated January 6,1988, and considering that this case has
long been submitted for decision, resolved to deny the aforesaid motion for having been filed
late (Reno, p. 597).

The principal issue raised by the petitioner and by the erstwhile intervenors, is: whether or
not under the provision of P.D. No. 1214 there was deprivation of property without due
process of law and just compensation which makes said decree unconstitutional. Their
contention that a perfected and valid appropriation of public mineral lands operates as a
withdrawal of the tract of land from the public domain and is deemed to be already private
property, is without basis in fact and in law (Comment, Rollo, p. 61)

This issue has been resolved in a recent decision of this Court in Sta. Rosa Mining Co., Inc.
vs. Leido Jr. (156 SCRA 1 [1987]) where it was held that while rulings in McDaniel v.
Apacible (42 Phil. 749 [1922]). and Gold Creek Mining Corp. v. Rodriguez (66 Phil. 259
[1938]) cited by the petitioner, true enough, recognize the right of a locator of a mining
claim as a property right; such right is not absolute. It is merely a possessory right more so
if petitioner's claims are still unpatented. It can be lost through abandonment or forfeiture
or they may be revoked on valid legal grounds.

In the case at bar, there is no showing that petitioner has complied with all the terms and
conditions prescribed by law prior to November 1, 1935; that there should be not only a
valid and subsisting location of the mineral land but that there should be, thereafter,
continuous compliance with all the requirements of law such as the performance of annual
assessment works and payment of real estate taxes. In fact, petitioner filed its application
only in 1977 for a patent, or 43 years after it allegedly located and registered the mining
claims (Rollo, p. 63).

As to the issue of constitutionality, the Court categorically stated that P.D. No. 1214 is
constitutional. The Court ruled:
...It is a valid exercise of the sovereign power of the State, as owner, over lands of the public
domain, of which petitioner's mining claims still form a part, and over the patrimony of the
nation, of which mineral deposits are a valuable asset. It may be underscored, in this
connection, that the Decree does not cover all mining claims located under the Phil. Bill of
1902, but only those claims over which their locators had failed to obtain a patent. And even
then, such locators may still avail of the renewable twenty-five year (25) lease prescribed
by Pres. Decree No. 463, the Mineral Development Resources Decree of 1974.

Mere location does not mean absolute ownership over the affected land or the mining claim.
It merely segregates the located land or area from the public domain by barring other would
N a t r e _ M i d t e r m S Y 2 0 1 9 - 2 0 2 0 _ 1 s t S e m e s t e r _ P a g e 59 | 68
be locators from locating the same and appropriate for themselves the minerals found
therein. To rule otherwise would imply that location is all that is needed to acquire and
maintain rights over a located mining claim. This, we cannot approve or sanction because it
is contrary to the intention of the lawmaker that the locator should faithfully and consistently
comply with the requirements for annual work and improvements in the located mining claim.
(Santa Rosa Mining Co., Inc. vs. Leido Jr., supra, pp. 8-9)

P.D. No. 1214 is in accord with Section 8, Article XIV of the 1973 Constitution and presently
in Section 2, Article XII of the 1987 Constitution where the same constitutional mandate is
restated. On June 2,1988, the Court granted a motion filed by counsel for petitioner dated
May 20,1988 to admit a manifestation and motion wherein petitioner prayed that the "Court
allow the petitioner to change the original prayer in its petition dated October 10, 1978 with
a new prayer directing public respondents to dispose of petitioner's application on its own
merit unaffected and without regard to the provision of P.D. 1214 . . ." (p. 631, Rollo)

Records show that petitioner Zambales Chromite filed its patent application over its 60
mining claims on June 14,1977 and to order such disposal of said "application on its own
merit" is not within the scope of the jurisdiction of the Court. For, even assuming claimant
to be a holder of a subsisting and valid patentable mining claim, this Court has held that it
can no longer proceed with the acquisition of a mining patent in view of P.D. No. 1214, issued
on October 14, 1977, directing holder of subsisting and patentable mining claims, lode or
placer, located under the provisions of the Act of Congress on July 1, 1902, as amended, to
file a mining lease application . . . within one year from the approval of the Decree and upon
the filing thereof, holders of said claims shall be considered to have waived their rights to
the issuance of mining patents therefor: Provided, however, that the non-filing of the
application for mining lease by the holders thereby within the period herein prescribed shall
cause the forfeiture of all his rights to the claim." (Director of Lands v. Kalahi Investments,
Inc., G.R. No. L-48066, January 31, 1989). (Emphasis supplied)

PREMISES CONSIDERED, the instant petition is DENIED for lack of merit.


SO ORDERED

8. PNOC-ENERGY DEVELOPMENT CORPORATION (PNOC-EDC), Petitioner, vs.


EMILIANO G. VENERACION, JR., Respondent.
DECISION
CHICO-NAZARIO, J.:
This case involves the conflicting claims of the petitioner Philippine National Oil Corporation
Energy Development Corporation and the respondent over the mining rights over Block 159
of the Malangas Coal Reservation, Alicia, Zamboanga del Sur.

FACTS
On 31 January 1989, respondent applied with the Mines and Geo-Sciences Development
Services, DENR, Region IX, Zamboanga City for a Declaration of Location (DOL) over Block
159 of the Malangas Coal Reservation, situated at Barangays Payongan and Kauswagan,
Alicia, Zamboanga del Sur. On 18 May 1989, the Office of the Regional Executive Director
(RED) of the DENR informed the respondent that his DOL cannot be registered since Block
159 was part of the Malangas Coal Reservation, as provided under Proclamation No. 284,
issued by the President on 19 July 1938. With the endorsement of the Office of Energy Affairs
(OEA) and the DENR Secretary, the respondent petitioned the Office of the President for the
withdrawal of Block 159 from the coal reservation and its conversion into a mineral
reservation.

The petitioner applied for a mineral prospecting permit over Block 159 (and Blocks 120 and
160) with the OEA, which the latter granted on 4 September 1989. The Malangas Coal
Reservation was, at that time, under the administration of the OEA. When it had initially
applied for a mineral prospecting permit over lands within the Malangas Coal Reservation,
N a t r e _ M i d t e r m S Y 2 0 1 9 - 2 0 2 0 _ 1 s t S e m e s t e r _ P a g e 60 | 68
the OEA advised it to obtain the permission of the Bureau of Mines and GeoSciences (BMGS).
On 18 October 1991, petitioner submitted to the DENR an application/proposal for a Mineral
Production Sharing Agreement (MPSA) over Blocks 120, 159 and 160 of the Malangas Coal
Reservation. On 21 February 1992, the Officer-In-Charge Regional Technical Director Dario
R. Miñoza of the Mines and Geo-Sciences Developmental Service (MGDS) advised the
petitioner to amend its application for MPSA by excluding Block 159 as the same is covered
by the application of the respondent. Nevertheless, the petitioner did not exclude Block 159
from its MPSA. Records also show that it had not applied for nor was it able to obtain an
Exploration Permit from the BMGS over Block 159.

On 13 April 1992, Presidential Proclamation No. 890 was issued, which effectively excluded
Block 159 from the operation of Proclamation No. 284, and declared Block No. 159 as
government mineral reservation open for disposition to qualified mining applicants, pursuant
to Executive Order No. 279. On 26 May 1992, petitioner’s application for MPSA covering
Coal Block Nos. 120, 159 and 160 was accepted for filing. Respondent immediately filed, on
28 May 1992, a protest to the petitioner’s inclusion of Block 159 in its application for MPSA
before the RED of the DENR Office in Zamboanga City. After the parties were heard, the
RED, in an Order, dated 12 April 1993, ruled in favor of the respondent and ordered the
petitioner to amend its MPSA by excluding therefrom Block 159. On 18 May 1993, petitioner
filed a Motion for Reconsideration of the Order dated 12 April 1993, which the RED denied in
an Order dated 5 July 1993. On 30 July 1993, petitioner filed an appeal with the DENR
Secretary questioning the Orders issued by the RED.

While the case was pending, respondent applied for a MPSA. On 31 July 1992, he paid the
processing fee for a MPSA covering Block 159 and was able to comply with all other
requirements of the MPSA application. On 4 October 1994, the Office of the Secretary
dismissed the appeal on the ground that petitioner’s right to appeal had already prescribed.
Section 50 of Presidential Decree No. 463 provides therefore for a five-day reglementary
period from the receipt of the order or decision of the Director. Petitioner received its copy
of the assailed Order dated 12 April 1993 on 7 May 1993, but filed its Motion for
Reconsideration only on 18 May 1993, or eleven days after its receipt thereof. Thereafter,
petitioner received a copy of the Order dated 5 July 1993 on 16 July 1993, but filed its appeal
only on 30 July 1993 or nine days after the allowable period to appeal.

On 25 October 1994, petitioner, through a letter addressed to the DENR Secretary, sought
the reconsideration of the Decision, dated 4 October 1994. In a Resolution, dated 21
December 1994, the then DENR Secretary Angel C. Alcala reversed the Decision, dated 4
October 1994, and gave due course to the MPSA of the petitioner.

On 1 February 1995, respondent filed a Motion for Reconsideration of the Resolution, dated
21 December 1994. The now DENR Secretary Victor O. Ramos issued an Order, dated 5
August 1996, reversing the Resolution, dated 21 December 1994 and reinstating the
Decision, dated 4 October 1994. It ruled that the Orders issued by the RED have already
become final and executory when the petitioner failed to file its appeal five days after it had
received the Orders. As a result, the DENR Secretary no longer had the jurisdiction to issue
the assailed Resolution, dated 21 December 1994. It added that after looking into the merits
of the case, the Orders of the RED were in accordance with the evidence on record and the
pertinent laws on the matter. On 20 August 1996, petitioner filed a Motion for
Reconsideration of the Order, dated 5 August 1996. On 21 May 1997, the MAB resolved the
motion in favor of the respondent and affirmed the assailed Order, dated 5 August 1996. It
took cognizance of the appeal filed by petitioner, in accordance with Section 78 of Republic
Act No 7942, otherwise known as The Philippine Mining Act of 1995. The MAB ruled that the
petitioner filed its appeal beyond the five-day prescriptive period provided under Presidential
Decree No. 463, which was then the governing law on the matter.

The MAB also decreed that the respondent had preferential mining rights over Block 159. It
ruled that the proper procedure with respect to the mining rights application over Block 159
N a t r e _ M i d t e r m S Y 2 0 1 9 - 2 0 2 0 _ 1 s t S e m e s t e r _ P a g e 61 | 68
when it was still part of the Malangas Coal Reservation required the following: (1) application
for prospecting permit with the OEA or other office having jurisdiction over said reservation;
(2) application for exploration permit; (3) application for exclusion of the land from such
reservation; (4) Presidential Declaration on exclusion as recommended by the Secretary;
and (5) application for Lease thereof with priority given to holder of exploration Permit.

The MAB noted that petitioner did not file for an exploration permit nor applied for the
exclusion of Block 159. Moreover, petitioner filed a MPSA on 18 October 1991, or almost six
(6) months prior to the issuance of Proclamation No. 890 excluding Block 159 from the
Malangas Coal Reservation and allowing its disposition. Thus, the application for a MPSA over
Block 159, while it was still part of a government reservation other than a mineral
reservation, was erroneous and improper and could not have been legally accepted. And,
since the records show that only one MPSA was filed after the issuance of Proclamation 890
– that of the respondent’s, the preferential right over Block 159 was acquired by the
respondent. The MAB, nevertheless, pointed out that the said preferential right does not
necessarily lead to the granting of the respondent’s MPSA, but merely consists of the right
to have his application evaluated and the prohibition against accepting other mining
applications over Block 159 pending the processing of his MPSA.

ISSUES:
There are two main issues that need to be resolved in this case: (1) whether or not the
petitioner has already lost its right to appeal the RED’s Order dated 12 April 1993; and (2)
whether or not the petitioner acquired a preferential right on mining rights over Block 159.

HELD: (kopyahon ko na lang ang sa SCRA)


Administrative Law; Appeals; Appeals from judgments and final orders of quasi-judicial
bodies are required to be brought to the CA, under the requirements and conditions set forth
in Rule 43 of the Rules of CivPro. With the enactment of Republic Act No. 7902, this Court
issued Circular No. 1-95 dated 16 May 1995 governing appeals from all quasi-judicial bodies
to the Court of Appeals by petition for review, regardless of the nature of the question raised.
Said circular was incorporated in Rule 43 of the Rules of Civil Procedure. In addition, this
Court held in a line of cases that appeals from judgments and final orders of quasi-judicial
bodies are required to be brought to the Court of Appeals, under the requirements and
conditions set forth in Rule 43 of the Rules of Civil Procedure. Nevertheless, this Court has
taken into account the fact that these cases were promulgated after the petitioner filed this
appeal on 4 August 1997, and decided to take cognizance of the present case.

Natural resources; mines, by providing a five-day period within which to file an appeal on
the decisions of the Dir. Of Mines and Geo-Sciences, PD 463 unquestionably repealed Section
61 of CA 137 - When Presidential Decree No. 463 was enacted in 1974, Section 50 of the
law had clearly intended to repeal the corresponding provision found in Section 61 of
Commonwealth Act No. 137, and to shorten the 30-day period within which to file an appeal
from the Decision of the Director of Mines and Geo-Sciences to five days. Section 61 of
Commonwealth Act No. 137, as amended, provides that: SEC. 61. - Conflicts and disputes
arising out of mining locations shall be submitted to the Director of Mines for decision:
Provided, That the decision or order of the Director of Mines may be appealed to the
Secretary of Agriculture and Natural Resources within thirty days from receipt of such
decision or order. In case any one of the parties should disagree from the decision or order
of the Secretary of Agriculture and Natural Resources, the matter may be taken to the Court
of Appeals or the Supreme Court, as the case may be, within thirty days from the receipt of
such decision or order, otherwise the said decision or order shall be final and binding upon
the parties concerned. x x x. Section 50 of Presidential Decree No. 463 reads: Sec. 50.
Appeals. - Any party not satisfied with the decision or order of the Director, may, within five
(5) days from receipt thereof, appeal to the Minister [now Secretary]. Decisions of the
Minister [now Secretary] are likewise appealable within five (5) days from receipt thereof by
the affected party to the President whose decision shall be final and executory.

Natre_Midterm SY2019-2020_1stSemester_ P a g e 62 | 68
Petitioner’s insistence that the 30-day reglementary period provided by Section 61 of
Commonwealth Act No. 137, as amended, applies, cannot be sustained by this Court. By
providing a five-day period within which to file an appeal on the decisions of the Director of
Mines and GeoSciences, Presidential Decree No. 463 unquestionably repealed Section 61 of
Commonwealth Act No. 137.

Rules of Procedure must be faithfully followed except only when for persuasive reasons, they
may be relaxed to relieve a litigant of an injustice not commensurate with his failure to
comply with the prescribed procedure - Petitioner invokes the judicial policy of allowing
appeals, although filed late, when the interest of justice so requires. Procedural law has its
own rationale in the orderly administration of justice, namely, to ensure the effective
enforcement of substantive rights by providing for a system that obviates arbitrariness,
caprice, despotism, or whimsicality in the settlement of disputes. Hence, rules of procedure
must be faithfully followed except only when for persuasive reasons, they may be relaxed to
relieve a litigant of an injustice not commensurate with his failure to comply with the
prescribed procedure. Concomitant to a liberal application of the rules of procedure should
be an effort on the part of the party invoking liberality to explain his failure to abide by the
rules.30 In the instant case, petitioner failed to state any compelling reason for not filing its
appeal within the mandated period. Instead, the records show that after failing to comply
with the period within which to file their motion for reconsideration on time, they again failed
to file their appeal before the Office of the DENR Secretary within the time provided by law.
Natural resources, mines: as a general rule, prospecting and exploration of minerals in a
government reservation is prohibited under Sec. 13 of PD 463 - However, the same rule
provides an exception involving instances when the government agency concerned allows it.
Section 13. Areas Closed to Mining Location. – No prospecting and exploration shall be
allowed: (a) In military, and other Government reservations except when authorized by the
proper Government agency concerned. Section 8 of Presidential Decree No. 463 reiterates
the rule and clarifies it further by stating that prospecting, exploration and exploitation of
minerals on reserved lands other than mineral reservations may be undertaken by the proper
government agency. As an exception to this rule, qualified persons may undertake the said
prospecting, exploration and exploitation when the said agencies cannot undertake them.
Petition denied!

G.R. No. 195580 April 21, 2014


9. NARRA NICKEL MINING AND DEVELOPMENT CORP., TESORO MINING AND
DEVELOPMENT, INC., and MCARTHUR MINING, INC., Petitioners, 
vs.
REDMONT
CONSOLIDATED MINES CORP., Respondent.
FACTS:
Sometime in December 2006, respondent Redmont Consolidated Mines Corp. (Redmont), a
domestic corporation organized and existing under Philippine laws, took interest in mining
and exploring certain areas of the province of Palawan. After inquiring with the Department
of Environment and Natural Resources (DENR), it learned that the areas where it wanted to
undertake exploration and mining activities where already covered by Mineral Production
Sharing Agreement (MPSA) applications of petitioners Narra, Tesoro and McArthur.
Petitioner McArthur Narra and Tesoro, filed an application for an MPSA and Exploration Permit
(EP) which was subsequently issued.

On January 2, 2007, Redmont filed before the Panel of Arbitrators (POA) of the DENR three
(3) separate petitions for the denial of petitioners’ applications for MPSA. Redmont alleged
that at least 60% of the capital stock of McArthur, Tesoro and Narra are owned and controlled
by MBMI Resources, Inc. (MBMI), a 100% Canadian corporation. Redmont reasoned that
since MBMI is a considerable stockholder of petitioners, it was the driving force behind
petitioners’ filing of the MPSAs over the areas covered by applications since it knows that it
can only participate in mining activities through corporations which are deemed Filipino
citizens. Redmont argued that given that petitioners’ capital stocks were mostly owned by

Natre_Midterm SY2019-2020_1stSemester_ P a g e 63 | 68
MBMI, they were likewise disqualified from engaging in mining activities through MPSAs,
which are reserved only for Filipino citizens.

Petitioners averred that they were qualified persons under Section 3(aq) of Republic Act No.
(RA) 7942 or the Philippine Mining Act of 1995. They stated that their nationality as
applicants is immaterial because they also applied for Financial or Technical Assistance
Agreements (FTAA) denominated as AFTA-IVB-09 for McArthur, AFTA-IVB-08 for Tesoro and
AFTA-IVB-07 for Narra, which are granted to foreign-owned corporations. Nevertheless, they
claimed that the issue on nationality should not be raised since McArthur, Tesoro and Narra
are in fact Philippine Nationals as 60% of their capital is owned by citizens of the Philippines.
On December 14, 2007, the POA issued a Resolution disqualifying petitioners from gaining
MPSAs. The POA considered petitioners as foreign corporations being "effectively controlled"
by MBMI, a 100% Canadian company and declared their MPSAs null and void.

Pending the resolution of the appeal filed by petitioners with the MAB, Redmont filed a
Complaint with the Securities and Exchange Commission (SEC), seeking the revocation of
the certificates for registration of petitioners on the ground that they are foreign-owned or
controlled corporations engaged in mining in violation of Philippine laws.

CA found that there was doubt as to the nationality of petitioners when it realized that
petitioners had a common major investor, MBMI, a corporation composed of 100%
Canadians. Pursuant to the first sentence of paragraph 7 of Department of Justice (DOJ)
Opinion No. 020, Series of 2005, adopting the 1967 SEC Rules which implemented the
requirement of the Constitution and other laws pertaining to the exploitation of natural
resources, the CA used the "grandfather rule" to determine the nationality of petitioners.

In determining the nationality of petitioners, the CA looked into their corporate structures
and their corresponding common shareholders. Using the grandfather rule, the CA
discovered that MBMI in effect owned majority of the common stocks of the
petitioners as well as at least 60% equity interest of other majority shareholders
of petitioners through joint venture agreements. The CA found that through a "web
of corporate layering, it is clear that one common controlling investor in all mining
corporations involved x x x is MBMI." Thus, it concluded that petitioners McArthur,
Tesoro and Narra are also in partnership with, or privies-in-interest of, MBMI.

ISSUE:
Whether or not the Court of Appeals’ ruling that Narra, Tesoro and McArthur are foreign
corporations based on the "Grandfather Rule" is contrary to law, particularly the express
mandate of the Foreign Investments Act of 1991, as amended, and the FIA Rules.

HELD:
No. There are two acknowledged tests in determining the nationality of a corporation: the
control test and the grandfather rule. Paragraph 7 of DOJ Opinion No. 020, Series of
2005, adopting the 1967 SEC Rules which implemented the requirement of the Constitution
and other laws pertaining to the controlling interests in enterprises engaged in the
exploitation of natural resources owned by Filipino citizens, provides:
Shares belonging to corporations or partnerships at least 60% of the capital of which
is owned by Filipino citizens shall be considered as of Philippine nationality (CONTROL
TEST), but if the percentage of Filipino ownership in the corporation or partnership is
less than 60%, only the number of shares corresponding to such percentage shall be
counted as of Philippine nationality (GRANDFATHER RULE). Thus, if 100,000 shares
are registered in the name of a corporation or partnership at least 60% of the capital
stock or capital, respectively, of which belong to Filipino citizens, all of the shares shall
be recorded as owned by Filipinos. But if less than 60%, or say, 50% of the capital
stock or capital of the corporation or partnership, respectively, belongs to Filipino
citizens, only 50,000 shares shall be counted as owned by Filipinos and the other
50,000 shall be recorded as belonging to aliens.
N a t r e _ M i d t e r m S Y 2 0 1 9 - 2 0 2 0 _ 1 s t S e m e s t e r _ P a g e 64 | 68
The grandfather rule, petitioners reasoned, has no leg to stand on in the instant case since
the definition of a "Philippine National" under Sec. 3 of the FIA does not provide for it. They
further claim that the grandfather rule "has been abandoned and is no longer the applicable
rule." They also opined that the last portion of Sec. 3 of the FIA admits the application of a
"corporate layering" scheme of corporations. Petitioners claim that the clear and
unambiguous wordings of the statute preclude the court from construing it and prevent the
court’s use of discretion in applying the law. They said that the plain, literal meaning of the
statute meant the application of the control test is obligatory.

SC disagreed. "Corporate layering" is admittedly allowed by the FIA; but if it is used to


circumvent the Constitution and pertinent laws, then it becomes illegal. Further, the
pronouncement of petitioners that the grandfather rule has already been abandoned must
be discredited for lack of basis.

Petitioners McArthur, Tesoro and Narra are not Filipino since MBMI, a 100% Canadian
corporation, owns 60% or more of their equity interests. Such conclusion is derived from
grandfathering petitioners’ corporate owners, namely: MMI, SMMI and PLMDC. The "control
test" is still the prevailing mode of determining whether or not a corporation is a Filipino
corporation, within the ambit of Sec. 2, Art. II of the 1987 Constitution, entitled to undertake
the exploration, development and utilization of the natural resources of the Philippines. When
in the mind of the Court there is doubt, based on the attendant facts and circumstances of
the case, in the 60-40 Filipino-equity ownership in the corporation, then it may apply the
"grandfather rule."

10. DIDIPIO V. GOZUN:*


Constitutionality of the Mining Act:
The Facts
On June 20, 1994, before the passage of Republic Act 7042 (The Philippine Mining Act of
1995), President Fidel V. Ramos executed a Financial and Technical Assistance Agreement
(FTAA) with Arimco Mining Corporation (AMC), over a total land area of 37,000 hectares
covering the provinces of Nueva Vizcaya and Quirino. Included in that area was Barangay
Didipio, Kasibu, Nueva Vizcaya.

Subsequently, AMC consolidated with Climax Mining Limited to form a single company that
did business under the new name of Climax-Arimco Mining Corporation (CAMC), of which the
controlling 99 percent of stockholders were Australian nationals.

On September 7, 2001, counsels for petitioners filed a demand letter addressed to then
Secretary Heherson Alvarez of the Department of Environment and Natural Resources
(DENR), seeking cancellation of the CAMC FTAA for the primary reason that Republic Act
7942 and its Implementing Rules and Regulations (IRR)-Department Administrative Order
(DAO) No. 96-40 were unconstitutional. This letter was referred to the panel of arbitrators
of the Mines and Geosciences Bureau (MGB), Regional Office No. 2, Tuguegarao, Cagayan,
for further action. The MGB eventually rejected the demand.

Petitioners thus filed the present Petition for Prohibition and Mandamus. They prayed that
the Court issue an order (1) enjoining public respondent from acting on any application for
an FTAA; (2) declaring the Philippine Mining Act of 1995 and its IRR unconstitutional; and
(3) canceling the FTAA issued to CAMC.

The Issues
The parties raised two sets of issues. The first was procedural: Was the issue of eminent
domain a justiciable controversy? Did petitioners have a legal standing to file the case?

The second set of issues was substantive in character: Did Section 76 of Republic Act 7942
allow the unlawful taking of private property for private purpose? Did the assailed law and
N a t r e _ M i d t e r m S Y 2 0 1 9 - 2 0 2 0 _ 1 s t S e m e s t e r _ P a g e 65 | 68
its IRR encroach on the power of the trial courts to determine just compensation, inasmuch
as that determination was cognizable only by the panel of arbitrators? Did the Mining Act
make it possible for FTAA contracts to cede over to a fully foreign-owned corporation full
control and management of mining enterprises, resulting in the alleged reduction of the State
to a passive regulator, dependent on submitted plans and reports and having only weak
review and audit powers? Were foreign corporations barred from making decisions on the
conduct of operations and the management of the mining project? Were service contracts
prohibited?

The Court’s Decision, penned by Justice Minita V. Chico-Nazario,[1] brushed aside the
procedural objections and answered all the substantive questions in the negative.

Procedural Issues
Respondents maintained that the issue of eminent domain was not ripe for adjudication,
because petitioners failed to allege that the latter’s properties had actually been taken by
CAMC. Neither did petitioners allege that their property rights had been endangered or were
in danger on account of the corporation’s FTAA. Dismissing this issue, the Court held that
there was a live controversy involving a clash of legal rights, because Republic Act 7942 had
been enacted, DAO 96-40 approved, and FTAAs entered into.

The FTAA holders were already operating in various provinces of the country. Among them
was CAMC, which operated in various provinces of Nueva Vizcaya and Quirino. As a result
of the CAMC FTAA in those places, numerous individuals, including petitioners, were in
danger of being ousted from their landholdings. In this light, the Court could not await the
adverse consequences of the law in order to consider the controversy actual and ripe for
judicial intervention. Actual eviction of the landowners and occupants did not have to happen
for this Court to intervene.

Also affirmed by the Court was the legal standing of petitioners to raise the constitutionality
of the questioned FTAA, as they had alleged personal and substantial injury from the mining
activities of CAMC. Likewise, they were under an imminent threat of being displaced from
their landholdings as a result of the implementation of the questioned FTAA.

Substantive Issues
Validity of Section 76 of
Republic Act 7942 and DAO 96-40
Petitioners claimed that Section 76 of Republic Act 7942 and DAO 96-40 were
unconstitutional for allowing the unlawful and unjust taking of private property for a private
purpose. The Court, however, rejected that claim. Underscoring the history of mining laws,
it explained that their evolution gave a positive indication that mining operators who were
qualified to own lands were granted the authority to exercise eminent domain for the entry,
acquisition, and use of private lands in areas open to mining operations.

Considering that Section 1 of PD 512 granted qualified mining operators that authority, which
was deemed incorporated in Section 76 of Republic Act 7942, the inescapable conclusion was
that the latter was a “taking” provision.

By this conclusion, the Court did not mean, though, that Section 76 was unconstitutional for
allowing the taking of private property without a determination of public use and the payment
of just compensation. Concededly, for a taking to be valid, it must be for public use. The
mining industry was deemed to play a pivotal role in the economic development of the
country and to be a vital tool in the government’s emphasis on accelerated recovery. It was
therefore an industry that yielded public benefit.

The Court rejected petitioners’ claim that the State’s discretion to decide when to take private
property had been contractually reduced by the CAMC FTAA. Moreover, it held that the
assailed agreement had laid down the ways and means by which a foreign contractor,
N a t r e _ M i d t e r m S Y 2 0 1 9 - 2 0 2 0 _ 1 s t S e m e s t e r _ P a g e 66 | 68
disqualified from owning land, could identify for the government which specific surface areas
within the FTAA contract area were to be acquired for the mine infrastructure. Through a
voluntary transaction, the government would acquire ownership of those areas on behalf of
the contractor, which would then be able to proceed with the full implementation of the
agreement.

Eminent domain was not yet called for at this stage, since there were still various avenues
by which surface rights could be acquired other than expropriation. The FTAA provision
under attack merely facilitated the implementation of the provision in question and thus
shielded CAMC from a charge of violating the Anti-Dummy Law. Also, the Mining Law and
its implementing rules provided for the payment of just compensation for private properties
to be expropriated.

The Power of Courts to


Determine Just Compensation
It was contended that the assailed law and order encroached on the power of the trial courts
to determine just compensation in eminent domain cases, inasmuch as that determination
was cognizable only by a panel of arbitrators.

The Mining Act did not prevent the courts from taking cognizance of expropriation cases. The
disagreement in Section 107 that petitioner referred to did not involve any exercise of
eminent domain. Rather, it contemplated a situation in which the permit holders were
allowed entry into the lands of surface owners, and in which disagreement ensued as regards
the proper compensation for the permitted entry and use of the private lands. Noticeably,
the provision pointed to a voluntary, not to an involuntary, sale or transaction.

The law conferred upon a panel of arbitrators the authority to decide cases in which permit
holders were refused entry by owners, occupants, and concessionaires, thus necessitating
an involuntary taking. The grant of authority did not imply, though, that the determination
of just compensation by the arbitrators or the mines adjudication board was final and
conclusive; the determination was only preliminary, unless accepted by all the parties
concerned. There was nothing wrong with this procedure, because the original and exclusive
jurisdiction of the courts to determine just compensation remained intact, despite the
preliminary determination made by the administrative agency.

Sufficient State Control


Over Mining Operations
Petitioners asserted that the law, the implementing regulations, and the CAMC FTAA had
ceded beneficial ownership of mineral resources to the foreign contractor. Holding that this
matter had already been settled in La Bugal-B’laan Tribal Association v. Ramos,[2] the Court
emphasized that the FTAA contractor was not free to do whatever it pleased and get away
with it; on the contrary, the contractor would have to follow the government line if it wanted
to stay in the enterprise. The law and its IRR had vested in the government more than a
sufficient degree of control and supervision over the conduct of mining operations.

Proper Interpretation of
“Agreements Involving Either
Technical or Financial Assistance”
Petitioners maintained that the Constitution barred aliens and foreign-owned corporations
from entering into direct arrangements with the government. Such arrangements included
agreements involving co-production, joint venture, and production sharing. Moreover, the
participation of foreign-owned corporations in large-scale exploration was limited to
agreements for either financial or technical assistance only.

In discrediting this argument, the Court again cited La Bugal B’laan. The use of the word
“involving” in the phrase “agreements involving either technical or financial assistance”
implied that the agreements with foreign corporations were not limited to mere financial and
N a t r e _ M i d t e r m S Y 2 0 1 9 - 2 0 2 0 _ 1 s t S e m e s t e r _ P a g e 67 | 68
technical assistance. If the real intention of the drafters was to confine foreign corporations
to financial or technical assistance and nothing more, their language would have certainly
been so unmistakably restrictive and stringent as to leave no doubt in anyone’s mind about
their true intent. Evidently, there was a conscious and deliberate decision on their part to
avoid the use of restrictive wording that bespoke an intent not to use the questioned
expression in an exclusionary and limiting manner.

Service Contracts Not Prohibited


Petitioner contended that the service contract regime under the previous Constitution was
expressly prohibited under the present one. “Service contracts,” the term found in the 1973
Constitution, was later deleted to avoid the circumvention of constitutional prohibitions
prevalent in the 1987 Charter.

The Court brushed aside this contention. The term used earlier had not been carried over to
the 1987 Constitution, which bore no categorical statement banning service contracts from
mining activities. This mere fact did not mean that service contracts, as understood in the
1973 Charter, were eradicated from the present Constitution. The latter still allowed the
continued use of these contracts with foreign corporations that would invest in and operate
and manage extractive enterprises, subject to the full control of the State. This time
however, safety measures would be put in place to prevent the abuses of the past regime. At
bottom, the Court reiterated the doctrinal pronouncements in La Bugal.

Natre_Midterm SY2019-2020_1stSemester_ P a g e 68 | 68

Das könnte Ihnen auch gefallen